Exam 3 RNSG PREP BRUNNER

Lakukan tugas rumah & ujian kamu dengan baik sekarang menggunakan Quizwiz!

A) Inflammatory bowel disease The use of a lavage solution is contraindicated in patients with intestinal obstruction or inflammatory bowel disease. It can safely be used with patients who have polyps, colon cancer, or diverticulitis.

1. A nurse is caring for a patient who is scheduled for a colonoscopy and whose bowel preparation will include polyethylene glycol electrolyte lavage prior to the procedure. The presence of what health problem would contraindicate the use of this form of bowel preparation? A) Inflammatory bowel disease B) Intestinal polyps C) Diverticulitis D) Colon cancer

B) Erosion of the lining of the stomach or intestine A peptic ulcer is erosion of the lining of the stomach or intestine. Peptic ulcers are often accompanied by bleeding and inflammation, but these are not the definitive characteristics.

1. A nurse is caring for a patient who just has been diagnosed with a peptic ulcer. When teaching the patient about his new diagnosis, how should the nurse best describe a peptic ulcer? A) Inflammation of the lining of the stomach B) Erosion of the lining of the stomach or intestine C) Bleeding from the mucosa in the stomach D) Viral invasion of the stomach wall

D) Brushing the patient's teeth with a toothbrush and small amount of toothpaste Application of mechanical friction is the most effective way to cleanse the patient's mouth. If the patient is unable to brush teeth, the nurse may brush them, taking precautions to prevent aspiration; or as a substitute, the nurse can achieve mechanical friction by wiping the teeth with a gauze pad. Bacteriocidal mouthwash does reduce plaque-causing bacteria; however, it is not as effective as application of mechanical friction. Water-soluble gel may be applied to lubricate dry lips, but it is not part of oral care.

1. A nurse is providing oral care to a patient who is comatose. What action best addresses the patient's risk of tooth decay and plaque accumulation? A) Irrigating the mouth using a syringe filled with a bacteriocidal mouthwash B) Applying a water-soluble gel to the teeth and gums C) Wiping the teeth and gums clean with a gauze pad D) Brushing the patient's teeth with a toothbrush and small amount of toothpaste

C) Consume high-residue, high-fiber foods. Goals for the patient include restoring or maintaining a regular pattern of elimination by responding to the urge to defecate, ensuring adequate intake of fluids and high-fiber foods, learning about methods to avoid constipation, relieving anxiety about bowel elimination patterns, and avoiding complications. Ongoing use of pharmacologic aids should not be promoted, due to the risk of dependence. Increased mobility helps to maintain a regular pattern of elimination. The urge to defecate should be heeded.

1. A nurse is working with a patient who has chronic constipation. What should be included in patient teaching to promote normal bowel function? A) Use glycerin suppositories on a regular basis. B) Limit physical activity in order to promote bowel peristalsis. C) Consume high-residue, high-fiber foods. D) Resist the urge to defecate until the urge becomes intense.

D) Inadequate nutrition and decreased saliva production can cause cavities Many ill patients do not eat adequate amounts of food and therefore produce less saliva, which in turn reduces the natural cleaning of the teeth. Stress response is not a factor, infections generally do not attack the enamel of the teeth, and the fluoride level of the patient is not significant in the development of dental caries in the ill patient.

10. A nurse is caring for a patient who is acutely ill and has included vigilant oral care in the patient's plan of care. Why are patients who are ill at increased risk for developing dental caries? A) Hormonal changes brought on by the stress response cause an acidic oral environment B) Systemic infections frequently migrate to the teeth C) Hydration that is received intravenously lacks fluoride D) Inadequate nutrition and decreased saliva production can cause cavities

B) Initiating the infusion slowly and monitoring the patient's fluid and glucose tolerance PN solutions are initiated slowly and advanced gradually each day to the desired rate as the patient's fluid and glucose tolerance permits. The formulation of the PN solutions is calculated carefully each day to meet the complete nutritional needs of the individual patient based on clinical findings and laboratory data. It is not infused more quickly at mealtimes.

10. A nurse is initiating parenteral nutrition (PN) to a postoperative patient who has developed complications. The nurse should initiate therapy by performing which of the following actions? A) Starting with a rapid infusion rate to meet the patient's nutritional needs as quickly as possible B) Initiating the infusion slowly and monitoring the patient's fluid and glucose tolerance C) Changing the rate of administration every 2 hours based on serum electrolyte values D) Increasing the rate of infusion at mealtimes to mimic the circadian rhythm of the body

B) A pregnant woman at 28 weeks' gestation Hemorrhoids commonly affect 50% of patients after the age of 50. Pregnancy may initiate hemorrhoids or aggravate existing ones. This is due to increased constipation during pregnancy. The significance of pregnancy is greater than that of standing, lifting, or stress in the development of hemorrhoids.

10. A nursing instructor is discussing hemorrhoids with the nursing class. Which patients would the nursing instructor identify as most likely to develop hemorrhoids? A) A 45-year-old teacher who stands for 6 hours per day B) A pregnant woman at 28 weeks' gestation C) A 37-year-old construction worker who does heavy lifting D) A 60-year-old professional who is under stress

A) Tachycardia, hypotension, and tachypnea Tachycardia, hypotension, and tachypnea are signs of recurrent bleeding. Patients who have had one GI bleed are at risk for recurrence. Tarry stools are expected short-term findings after a hemorrhage. Hemorrhage is not normally associated with sudden thirst or diaphoresis.

11. A patient who experienced an upper GI bleed due to gastritis has had the bleeding controlled and the patient's condition is now stable. For the next several hours, the nurse caring for this patient should assess for what signs and symptoms of recurrence? A) Tachycardia, hypotension, and tachypnea B) Tarry, foul-smelling stools C) Diaphoresis and sudden onset of abdominal pain D) Sudden thirst, unrelieved by oral fluid administration

C) Assess for a patent airway. Postoperatively, the nurse assesses for a patent airway. The patient's ability to manage secretions has a direct bearing on airway patency. However, airway patency is the overarching goal. This immediate physiologic need is prioritized over communication, though this is an important consideration. Infection is not normally a threat in the immediate postoperative period.

13. A nurse is assessing a patient who has just been admitted to the postsurgical unit following surgical resection for the treatment of oropharyngeal cancer. What assessment should the nurse prioritize? A) Assess ability to clear oral secretions. B) Assess for signs of infection. C) Assess for a patent airway. D) Assess for ability to communicate.

D) Maintain an open dialogue with the patient and facilitate a referral to the wound-ostomy-continence (WOC) nurse. A wound-ostomy-continence (WOC) nurse is a registered nurse who has received advanced education in an accredited program to care for patients with stomas. The enterostomal nurse therapist can assist with the selection of an appropriate stoma site, teach about stoma care, and provide emotional support. The surgeon is less likely to address the patient's psychosocial and learning needs. Reassurance does not address the patient's questions, and education may or may not alleviate anxiety.

14. A nurse is talking with a patient who is scheduled to have a hemicolectomy with the creation of a colostomy. The patient admits to being anxious, and has many questions concerning the surgery, the care of a stoma, and necessary lifestyle changes. Which of the following nursing actions is most appropriate? A) Reassure the patient that the procedure is relatively low risk and that patients are usually successful in adjusting to an ostomy. B) Provide the patient with educational materials that match the patient's learning style. C) Encourage the patient to write down these concerns and questions to bring forward to the surgeon. D) Maintain an open dialogue with the patient and facilitate a referral to the wound-ostomy-continence (WOC) nurse.

A) Risk for Aspiration Related to Inhalation of Gastric Contents Achalasia can result in the aspiration of gastric contents. It is not normally an acute risk to the patient's nutritional status and does not affect cardiac output or communication.

14. A patient has been diagnosed with achalasia based on his history and diagnostic imaging results. The nurse should identify what risk diagnosis when planning the patient's care? A) Risk for Aspiration Related to Inhalation of Gastric Contents B) Risk for Imbalanced Nutrition: Less than Body Requirements Related to Impaired Absorption C) Risk for Decreased Cardiac Output Related to Vasovagal Response D) Risk for Impaired Verbal Communication Related to Oral Trauma

A) Esophageal or pyloric obstruction related to scarring A severe form of acute gastritis is caused by the ingestion of strong acid or alkali, which may cause the mucosa to become gangrenous or to perforate. Scarring can occur, resulting in pyloric stenosis (narrowing or tightening) or obstruction. Chronic referred pain to the lower abdomen is a symptom of peptic ulcer disease, but would not be an expected finding for a patient who has ingested a corrosive substance. Bacterial proliferation and hyperacidity would not occur.

14. A patient was treated in the emergency department and critical care unit after ingesting bleach. What possible complication of the resulting gastritis should the nurse recognize? A) Esophageal or pyloric obstruction related to scarring B) Uncontrolled proliferation of H. pylori C) Gastric hyperacidity related to excessive gastrin secretion D) Chronic referred pain in the lower abdomen

A) Percussion Percussion is used to assess the size and density of the abdominal organs and to detect the presence of air-filled, fluid-filled, or solid masses. Percussion is used either independently or concurrently with palpation because it can validate palpation findings.

14. An advanced practice nurse is assessing the size and density of a patient's abdominal organs. If the results of palpation are unclear to the nurse, what assessment technique should be implemented? A) Percussion B) Auscultation C) Inspection D) Rectal examination

B) Below the right nipple Patients with referred abdominal pain associated with biliary colic complain of pain below the right nipple. Referred pain above the left nipple may be associated with the heart. Groin pain may be referred pain from ureteral colic.

15. A nurse is caring for a patient with biliary colic and is aware that the patient may experience referred abdominal pain. Where would the nurse most likely expect this patient to experience referred pain? A) Midline near the umbilicus B) Below the right nipple C) Left groin area D) Right lower abdominal quadrant

C) Frequent lung auscultation Aspiration is a risk associated with tube feeding; this risk may be exacerbated by the patient's cognitive deficits. Consequently, the nurse should auscultate the patient's lungs and monitor oxygen saturation closely. Bowel function is important, but the risk for aspiration is a priority. Hemorrhage is highly unlikely and the patient's abdominal girth is not a main focus of assessment.

15. A nurse is providing care for a patient with a diagnosis of late-stage Alzheimer's disease. The patient has just returned to the medical unit to begin supplemental feedings through an NG tube. Which of the nurse's assessments addresses this patient's most significant potential complication of feeding? A) Frequent assessment of the patient's abdominal girth B) Assessment for hemorrhage from the nasal insertion site C) Frequent lung auscultation D) Vigilant monitoring of the frequency and character of bowel movements

C) Avoid carbonated drinks. For a patient diagnosed with esophageal reflux disorder, the nurse should instruct the patient to keep the head of the bed elevated. Carbonated drinks, caffeine, and tobacco should be avoided. Protein limitation is not necessary.

15. A nurse is providing health promotion education to a patient diagnosed with an esophageal reflux disorder. What practice should the nurse encourage the patient to implement? A) Keep the head of the bed lowered. B) Drink a cup of hot tea before bedtime. C) Avoid carbonated drinks. D) Eat a low-protein diet.

A) Eating more slowly and chewing food more thoroughly Dysphagia may be prevented by educating patients to eat slowly, to chew food thoroughly, and to avoid eating tough foods such as steak or dry chicken or doughy bread. After bariatric procedures, patients should normally not drink beverages with meals. Medications or chewing gum will not alleviate this problem.

16. A patient is one month postoperative following restrictive bariatric surgery. The patient tells the clinic nurse that he has been having "trouble swallowing" for the past few days. What recommendation should the nurse make? A) Eating more slowly and chewing food more thoroughly B) Taking an OTC antacid or drinking a glass of milk prior to each meal C) Chewing gum to cause relaxation of the lower esophageal sphincter D) Drinking at least 12 ounces of liquid with each meal

B) Lower esophageal sphincter The lower esophageal sphincter, also called the gastroesophageal sphincter or cardiac sphincter, is located at the junction of the esophagus and the stomach. An incompetent lower esophageal sphincter allows reflux (backward flow) of gastric contents. The upper esophageal sphincter and the hypopharyngeal sphincter are synonymous and are not responsible for the manifestations of GERD. The pyloric sphincter exists between the stomach and the duodenum.

16. A staff educator is reviewing the causes of gastroesophageal reflux disease (GERD) with new staff nurses. What area of the GI tract should the educator identify as the cause of reduced pressure associated with GERD? A) Pyloric sphincter B) Lower esophageal sphincter C) Hypopharyngeal sphincter D) Upper esophageal sphincter

D) Report possible signs of aspiration pneumonia to the primary care provider. The patient should be assessed for further signs of aspiration pneumonia. It is unnecessary to remove the NG tube and chest physiotherapy is not indicated. A different feeding solution will not resolve this complication.

17. A nurse is caring for a patient with a nasogastric tube for feeding. During shift assessment, the nurse auscultates a new onset of bilateral lung crackles and notes a respiratory rate of 30 breaths per minute. The patient's oxygen saturation is 89% by pulse oximetry. After ensuring the patient's immediate safety, what is the nurse's most appropriate action? A) Perform chest physiotherapy. B) Reduce the height of the patient's bed and remove the NG tube. C) Liaise with the dietitian to obtain a feeding solution with lower osmolarity. D) Report possible signs of aspiration pneumonia to the primary care provider.

C) Promoting maximum shoulder function Shoulder drop occurs as a result of radical neck dissection. Shoulder function can be improved by rehabilitation exercises. Rehabilitation would not be initiated until the patient's neck incision and graft, if present, were sufficiently healed. Nerve paralysis in the cervical plexus and other variables affecting swallowing would be managed by a speech therapist rather than a physical therapist.

17. A patient who has had a radical neck dissection is being prepared for discharge. The discharge plan includes referral to an outpatient rehabilitation center for physical therapy. What would the goals of physical therapy for this patient include? A) Muscle training to relieve dysphagia B) Relieving nerve paralysis in the cervical plexus C) Promoting maximum shoulder function D) Alleviating achalasia by decreasing esophageal peristalsis

B) To reduce intestinal bacteria levels Antibiotics such a kanamycin (Kantrex), neomycin (Mycifradin), and cephalexin (Keflex) are administered orally the day before surgery to reduce intestinal bacterial. Preoperative antibiotics are not given to treat undiagnosed infections, reduce motility, or prevent abdominal distention.

18. A nurse caring for a patient with colorectal cancer is preparing the patient for upcoming surgery. The nurse administers cephalexin (Keflex) to the patient and explains what rationale? A) To treat any undiagnosed infections B) To reduce intestinal bacteria levels C) To reduce bowel motility D) To reduce abdominal distention postoperatively

C) Early diagnosis and treatment of gastroesophageal reflux disease There are numerous risk factors for esophageal cancer but chronic esophageal irritation or GERD is among the most significant. This is a more significant risk factor than dietary habits. Screening endoscopies are not recommended solely on the basis of family history.

18. A nurse is addressing the prevention of esophageal cancer in response to a question posed by a participant in a health promotion workshop. What action has the greatest potential to prevent esophageal cancer? A) Promotion of a nutrient-dense, low-fat diet B) Annual screening endoscopy for patients over 50 with a family history of esophageal cancer C) Early diagnosis and treatment of gastroesophageal reflux disease D) Adequate fluid intake and avoidance of spicy foods

D) A sudden release of peptides For many years, it had been theorized that the hypertonic gastric food boluses that quickly transit into the intestines drew extracellular fluid from the circulating blood volume into the small intestines to dilute the high concentration of electrolytes and sugars, resulting in symptoms. Now, it is thought that this rapid transit of the food bolus from the stomach into the small intestines instead causes a rapid and exuberant release of metabolic peptides that are responsible for the symptoms of dumping syndrome. It is not a result of phrenic nerve irritation, malabsorption, or bile reflux.

18. A patient has experienced symptoms of dumping syndrome following bariatric surgery. To what physiologic phenomenon does the nurse attribute this syndrome? A) Irritation of the phrenic nerve due to diaphragmatic pressure B) Chronic malabsorption of iron and vitamins A and C C) Reflux of bile into the distal esophagus D) A sudden release of peptides

D) Persistently low hemoglobin and hematocrit In the absence of intrinsic factor, vitamin B12 cannot be absorbed, and pernicious anemia results. This would result in a marked reduction in hemoglobin and hematocrit.

18. A patient is being assessed for a suspected deficit in intrinsic factor synthesis. What diagnostic or assessment finding is the most likely rationale for this examination of intrinsic factor production? A) Muscle wasting B) Chronic jaundice in the absence of liver disease C) The presence of fat in the patient's stool D) Persistently low hemoglobin and hematocrit

D) Change in bowel habits The most common presenting symptom associated with colorectal cancer is a change in bowel habits. The passage of blood is the second most common symptom. Symptoms may also include unexplained anemia, anorexia, weight loss, and fatigue. Hemorrhoids and bloating are atypical.

19. A nurse is teaching a group of adults about screening and prevention of colorectal cancer. The nurse should describe which of the following as the most common sign of possible colon cancer? A) Development of new hemorrhoids B) Abdominal bloating and flank pain C) Unexplained weight gain D) Change in bowel habits

In the course of a focused neurologic assessment, the nurse is palpating the patient's major muscle groups at rest and during passive movement. Data gleaned from this assessment will allow the nurse to describe which of the following aspects of neurologic function? A) Muscle dexterity B) Muscle tone C) Motor symmetry D) Deep tendon reflexes

Ans: B Feedback: Muscle tone (the tension present in a muscle at rest) is evaluated by palpating various muscle groups at rest and during passive movement. Data from this assessment do not allow the nurse to ascertain the patient's dexterity, reflexes, or motor symmetry

A patient who has sustained a nondepressed skull fracture is admitted to the acute medical unit. Nursing care should include which of the following? A) Preparation for emergency craniotomy B) Watchful waiting and close monitoring C) Administration of inotropic drugs D) Fluid resuscitation

Ans: B Feedback: Nondepressed skull fractures generally do not require surgical treatment; however, close observation of the patient is essential. A craniotomy would not likely be needed if the fracture is nondepressed. Even if treatment is warranted, it is unlikely to include inotropes or fluid resuscitation.

As a member of the stroke team, the nurse knows that thrombolytic therapy carries the potential for benefit and for harm. The nurse should be cognizant of what contraindications for thrombolytic therapy? Select all that apply. A) INR above 1.0 B) Recent intracranial pathology C) Sudden symptom onset D) Current anticoagulation therapy E) Symptom onset greater than 3 hours prior to admission

Ans: B, D, E Feedback: Some of the absolute contraindications for thrombolytic therapy include symptom onset greater than 3 hours before admission, a patient who is anticoagulated (with an INR above 1.7), or a patient who has recently had any type of intracranial pathology (e.g., previous stroke, head injury, trauma)

A patient is admitted to the neurologic ICU with a spinal cord injury. In writing the patient's care plan, the nurse specifies that contractures can best be prevented by what action? A) Repositioning the patient every 2 hours B) Initiating range-of-motion exercises (ROM) as soon as the patient initiates C) Initiating (ROM) exercises as soon as possible after the injury D) Performing ROM exercises once a day

Ans: C Feedback: Passive ROM exercises should be implemented as soon as possible after injury. It would be inappropriate to wait for the patient to first initiate exercises. Toes, metatarsals, ankles, knees, and hips should be put through a full ROM at least four, and ideally five, times daily. Repositioning alone will not prevent contractures.

24. A gerontologic nurse is assessing a patient who has numerous comorbid health problems. What assessment findings should prompt the nurse to suspect a UTI? Select all that apply. A) Food cravings B) Upper abdominal pain C) Insatiable thirst D) Uncharacteristic fatigue E) New onset of confusion

Ans: D Feedback: The most common subjective presenting symptom of UTI in older adults is generalized fatigue. The most common objective finding is a change in cognitive functioning. Food cravings, increased thirst, and upper abdominal pain necessitate further assessment and intervention, but none is directly suggestive of a UTI.

41. The nurse is collaborating with the wound-ostomy-continence (WOC) nurse to teach a patient how to manage her new ileal conduit in the home setting. To prevent leakage or skin breakdown, the nurse should encourage which of the following practices? A) Empty the collection bag when it is between one-half and two-thirds full. B) Limit fluid intake to prevent production of large volumes of dilute urine. C) Reinforce the appliance with tape if small leaks are detected. D) Avoid using moisturizing soaps and body washes when cleaning the peristomal area.

Ans: D Feedback: The patient is instructed to avoid moisturizing soaps and body washes when cleaning the area because they interfere with the adhesion of the pouch. To maintain skin integrity, a skin barrier or leaking pouch is never patched with tape to prevent accumulation of urine under the skin barrier or faceplate. Fluids should be encouraged, not limited, and the collection bag should not be allowed to become more than one-third full.

29. A patient has a flaccid bladder secondary to a spinal cord injury. The nurse recognizes this patient's high risk for urinary retention and should implement what intervention in the patient's plan of care? A) Relaxation techniques B) Sodium restriction C) Lower abdominal massage D) Double voiding

Ans: D Feedback: To enhance emptying of a flaccid bladder, the patient may be taught to double void. After each voiding, the patient is instructed to remain on the toilet, relax for 1 to 2 minutes, and then attempt to void again in an effort to further empty the bladder. Relaxation does not affect the neurologic etiology of a flaccid bladder. Sodium restriction and massage are similarly ineffective.

26. An adult patient has been hospitalized with pyelonephritis. The nurse's review of the patient's intake and output records reveals that the patient has been consuming between 3 L and 3.5 L of oral fluid each day since admission. How should the nurse best respond to this finding? A) Supplement the patient's fluid intake with a high-calorie diet. B) Emphasize the need to limit intake to 2 L of fluid daily. C) Obtain an order for a high-sodium diet to prevent dilutional hyponatremia. D) Encourage the patient to continue this pattern of fluid intake.

Ans: D Feedback: Unless contraindicated, 3 to 4 L of fluids per day is encouraged to dilute the urine, decrease burning on urination, and prevent dehydration. No need to supplement this fluid intake with additional calories or sodium.

patient diagnosed with Bell's palsy is being cared for on an outpatient basis. During health education, the nurse should promote which of the following actions

Applying a protective eye shield at night Corneal irritation and ulceration may occur if the eye is unprotected. While paralysis lasts, the involved eye must be protected.

The nurse caring for a patient in ICU diagnosed with Guillain-Barré syndrome should prioritize monitoring for what potential complication

Autonomic dysfunction potential complications that may develop include respiratory failure and autonomic dysfunction.

The nurse is discharging a patient home after surgery for trigeminal neuralgia. What advice should the nurse provide to this patient in order to reduce the risk of injury

Avoid rubbing the eye on the affected side of the face.

A nurse is caring for a patient who has been scheduled for endoscopic retrograde cholangiopancreatography (ERCP) the following day. When providing anticipatory guidance for this patient, the nurse should describe what aspect of this diagnostic procedure? A) The need to protect the incision postprocedure B) The use of moderate sedation C) The need to infuse 50% dextrose during the procedure D) The use of general anesthesia

B

A nurse is caring for a patient with gallstones who has been prescribed ursodeoxycholic acid (UDCA). The patient askshow this medicine is going to help his symptoms. The nurse should be aware of what aspect of this drugs pharmacodynamics? A) It inhibits the synthesis of bile. B) It inhibits the synthesis and secretion of cholesterol. C) It inhibits the secretion of bile. D) It inhibits the synthesis and secretion of amylase.

B

A nurse is preparing a plan of care for a patient with pancreatic cysts that have necessitated drainage through the abdominal wall. What nursing diagnosis should the nurse prioritize? A) Disturbed Body Image B) Impaired Skin Integrity C) Nausea D) Risk for Deficient Fluid Volume

B

A patient has been diagnosed with acute pancreatitis. The nurse is addressing the diagnosis of Acute Pain Related to Pancreatitis. What pharmacologic intervention is most likely to be ordered for this patient? A) Oral oxycodone B) IV hydromorphone (Dilaudid) C) IM meperidine (Demerol) D) Oral naproxen (Aleve)

B

A nurse is caring for a patient with cirrhosis secondary to heavy alcohol use. The nurse's most recent assessment reveals subtle changes in the patient's cognition and behavior. What is the nurse's most appropriate response? A. Ensure that the patient's sodium intake does not exceed recommended levels B. Report this finding to the primary care provider due to the possibility of hepatic encephalopathy C. Inform the primary care provider that the patient should be assessed for alcoholic hepatitis. D. Implement interventions aimed at ensuring a calm and therapeutic care environment.

B. Report this finding to the primary care provider due to the possibility of hepatic encephalopathy Rationale: Monitoring is an essential nursing function to identify early deterioration in mental status. The nurse monitors the patient's mental status closely and reports changes so that treatment of encephalopathy can be initiated promptly. This change in status is likely unrelated to sodium intake and would not signal the onset of hepatitis. A supportive care environment is beneficial, but does not address the patient's physiologic deterioration.

A community health nurse is caring for a patient whose multiple health problems include chronic pancreatitis. During the most recent visit, the nurse notes that the patient is experiencing severe abdominal pain and has vomited 3 times in the past several hours. What is the nurses most appropriate action? A) Administer a PRN dose of pancreatic enzymes as ordered. B) Teach the patient about the importance of abstaining from alcohol. C) Arrange for the patient to be transported to the hospital. D) Insert an NG tube, if available, and stay with the patient.

C

A nurse is aware of the high incidence and prevalence of fluid volume deficit among older adults. What related health education should the nurse provide to an older adult? A) "If possible, try to drink at least 4 liters of fluid daily." B) "Ensure that you avoid replacing water with other beverages." C) "Remember to drink frequently, even if you don't feel thirsty." D) "Make sure you eat plenty of salt in order to stimulate thirst."

C

A nurse is creating a care plan for a patient with acute pancreatitis. The care plan includes reduced activity. What rationale for this intervention should be cited in the care plan? A) Bed rest reduces the patients metabolism and reduces the risk of metabolic acidosis. B) Reduced activity protects the physical integrity of pancreatic cells. C) Bed rest lowers the metabolic rate and reduces enzyme production. D) Inactivity reduces caloric need and gastrointestinal motility

C

A nurse is providing discharge education to a patient who has undergone a laparoscopic cholecystectomy. During the immediate recovery period, the nurse should recommend what foods? A) High-fiber foods B) Low-purine, nutrient-dense foods C) Low-fat foods high in proteins and carbohydrates D) Foods that are low-residue and low in fat

C

A nurse who provides care in a walk-in clinic assesses a wide range of individuals. The nurse should identify which of the following patients as having the highest risk for chronic pancreatitis? A) A 45-year-old obese woman with a high-fat diet B) An 18-year-old man who is a weekend binge drinker C) A 39-year-old man with chronic alcoholism D) A 51-year-old woman who smokes one-and-a-half packs of cigarettes per day

C

A patient admitted to the medical unit with impaired renal function is complaining of severe, stabbing pain in the flank and lower abdomen. The patient is being assessed for renal calculi. The nurse recognizes that the stone is most likely in what anatomic location? A) Meatus B) Bladder C) Ureter D) Urethra

C

A nurse is caring for a patient with hepatic encephalopathy. The nurse's assessment reveals that the patient exhibits episodes of confusion, is difficult to arouse from sleep and has rigid extremities. Based on these clinical findings, the nurse should document what stage of hepatic encephalopathy? A. Stage 1 B. Stage 2 C. Stage 3 D. Stage 4

C. Stage 3 Rationale: Patients in the third stage of hepatic encephalopathy exhibit the following symptoms: stuporous, difficult to arouse, sleeps most of the time, exhibits marked confusion, incoherent in speech, asterixis, increased deep tendon reflexes, rigidity of extremities, marked EEG abnormalities. Patients in stages 1 and 2 exhibit clinical symptoms that are not as advanced as found in stage 3, and patients in stage 4 are comatose. In stage 4, there is an absence of asterixis, absence of deep tendon reflexes, flaccidity of extremities, and EEG abnormalities.

A patient is admitted through the ED with suspected St. Louis encephalitis. The unique clinical feature of St. Louis encephalitis will make what nursing action a priority

Close monitoring of fluid balance due to SIADH

A 55-year-old man has been newly diagnosed with acute pancreatitis and admitted to the acute medical unit. How should the nurse most likely explain the pathophysiology of this patients health problem? A) Toxins have accumulated and inflamed your pancreas. B) Bacteria likely migrated from your intestines and became lodged in your pancreas. C) A virus that was likely already present in your body has begun to attack your pancreatic cells. D) The enzymes that your pancreas produces have damaged the pancreas itself.

D

A geriatric nurse is performing an assessment of body systems on an 85-year-old patient. The nurse should be aware of what age-related change affecting the renal or urinary system? A) Increased ability to concentrate urine B) Increased bladder capacity C) Urinary incontinence D) Decreased glomerular filtration rate

D

A patient with liver cancer is being discharged home with a biliary drainage system in place. The nurse should teach the patient's family how to safely perform which of the following actions? A. Aspirating bile from the catheter using a syringe B. Removing the catheter when output is 15 mL in 24 hours C. Instilling antibiotics into the catheter D. Assessing the patency of the drainage catheter

D. Assessing the patency of the drainage catheter Rationale: Families should be taught to provide basic catheter care, including assessment of patency. Antibiotics are not instilled into the catheter and aspiration using a syringe is contraindicated. The family would not independently remove the catheter; this would be done by a member of the care team when deemed necessary.

A patient with liver cancer is being discharged home with a hepatic artery catheter in place. The nurse should be aware that this catheter will facilitate which of the following? A. Continuous monitoring for portal hypertension B. Administration of immunosuppressive drugs during the first weeks after transplantation C. Real-time monitoring of vascular changes in the hepatic system D. Delivery of a continuous chemotherapeutic dose

D. Delivery of a continuous chemotherapeutic dose Rationale: In most cases, the hepatic artery catheter has been inserted surgically and has a prefilled infusion pump that delivers a continuous chemotherapeutic dose until completed. The hepatic artery catheter dose not monitor portal hypertension, deliver immunosuppressive drugs, or monitor vascular changes in the hepatic system

mononeuropathy.

Damage to a single nerve

patient is being admitted to the neurologic ICU with suspected herpes simplex virus encephalitis. What nursing action best addresses the patient's complaints of headache

Dimming the lights and reducing stimulation

A patient with suspected Creutzfeldt-Jakob disease (CJD) is being admitted to the unit. The nurse would expect what diagnostic test to be ordered for this patient

EEG

nurse is caring for a patient who is hospitalized with an exacerbation of MS. To ensure the patient's safety, what nursing action should be performed

Ensure that suction apparatus is set up at the bedside.Because of the patient's risk of aspiration, it is important to have a suction apparatus at hand.

patient diagnosed with Bell's palsy is having decreased sensitivity to touch of the involved nerve. What should the nurse recommend to prevent atrophy of the muscles Whistling

Facial exercises, such as wrinkling the forehead, blowing out the cheeks, and whistling, may be performed with the aid of a mirror to prevent muscle atrophy.

nurse is developing a plan of care for a patient newly diagnosed with Bell's palsy. The nurse's plan of care should address what characteristic manifestation of this disease

Facial paralysis

nurse is creating a plan of care for a patient who has a recent diagnosis of MS. Which of the following should the nurse include in the patient's care plan

Instruct the patient on daily muscle stretching.

nurse is working with a patient who is newly diagnosed with MS. What basic information should the nurse provide to the patient

MS is a progressive demyelinating disease of the nervous system.

The critical care nurse is caring for 25-year-old man admitted to the ICU with a brain abscess. What is a priority nursing responsibility in the care of this patient

Monitoring neurologic status closely Vigilant neurologic monitoring is a key aspect of caring for a patient who has a brain abscess.

patient with possible bacterial meningitis is admitted to the ICU. What assessment finding would the nurse expect for a patient with this diagnosis

Neck flexion produces flexion of knees and hips a positive Brudzinski's sign.

patient with MS has developed dysphagia as a result of cranial nerve dysfunction. What nursing action should the nurse consequently perform

Position the patient upright during feeding.Correct, upright positioning is necessary to prevent aspiration in the patient with dysphagia.

patient with diabetes presents to the clinic and is diagnosed with a mononeuropathy. This patient's nursing care should involve which of the following

Protection of the affected limb from injury

critical care nurse is admitting a patient in myasthenic crisis to the ICU. The nurse should prioritize what nursing action in the immediate care of this patient

Providing ventilatory assistance

Plasmapheresis

Removal of plasma from withdrawn blood by centrifuge. Collected cells are retransfused back into the donor. Fresh-frozen plasma or salt solution is used to replace withdrawn plasma.

nurse is caring for a patient with multiple sclerosis . patient tells the nurse the hardest thing to deal with is the fatigue. teaching the patient how to reduce fatigue, nurse suggest

Resting in an air-conditioned room whenever possible Fatigue is a common symptom of patients with MS.

male patient presents to the clinic complaining of a headache.nurse notes that patient is guarding his neck and tells the nurse that he has stiffness in the neck area.

The nurse suspects the patient may have meningitis. What is another well-recognized sign of this infection Positive Kernig's sign

48-year-old patient has been diagnosed with trigeminal neuralgia following recent episodes of unilateral face pain. The nurse should recognize what implication of this diagnosis

The patient needs to be assessed for MS.

The major symptoms of peripheral nerve disorders are loss of sensation, muscle atrophy, weakness, diminished reflexes, pain, and paresthesia of the extremities.

Trigeminal neuralgia is a condition of the fifth cranial nerve that is characterized by paroxysms of pain in the area innervated by any of the three branches, but most

The nurse is developing a plan of care for a patient with Guillain-Barré syndrome. Which of the following interventions should the nurse prioritize for this patient

Using the incentive spirometer as prescribed

patient diagnosed with MS has been admitted to the medical unit for treatment of an MS exacerbation. Included in the admission orders is baclofen (Lioresal).

What should the nurse identify as an expected outcome of this treatment Decreased severity and duration of exacerbations

73-year-old man comes to the clinic complaining of weakness and loss of sensation in his feet and legs. Assessment of the patient shows decreased reflexes bilaterally.

Why would it be a challenge to diagnose a peripheral neuropathy in this patient Many symptoms can be the result of normal aging process.

A patient presents at the clinic complaining of pain and weakness in her hands. On assessment, the nurse notes diminished reflexes in the upper extremities bilaterally

and bilateral loss of sensation. The nurse knows that these findings are indicative of what Peripheral nerve disorder

A patient with MS has been admitted to the hospital following an acute exacerbation. When planning the patient's care, the nurse addresses the need to enhance the patient's

bladder control. What aspect of nursing care is most likely to meet this goal Establish a timed voiding schedule.

You are the clinic nurse caring for a patient with a recent diagnosis of myasthenia gravis. The patient has begun treatment with pyridostigmine bromide (Mestinon). What

change in status would most clearly suggest a therapeutic benefit of this medication Increased muscle strength

nurse caring for a patient diagnosed with Guillain-Barré syndrome is planning care with regard to the clinical manifestations associated this syndrome. The nurse's

communication with the patient should reflect the possibility of what sign or symptom of the disease Vocal paralysis Guillain-Barré syndrome is a disorder of the vagus nerve

The nurse is caring for a 77-year-old woman with MS. She states that she is very concerned about the progress of her disease and what the future holds. The nurse should know that

elderly patients with MS are known to be particularly concerned about what variables Possible nursing home placement, Increasing disability, Becoming a burden on the family

69-year-old patient is brought to the ED by ambulance because a family member found him lying on the floor disoriented and lethargic. The physician suspects bacterial

meningitis and admits the patient to the ICU. nurse knows that risk factors for an unfavorable outcome include what Heart rate greater than 120 bpm, Older age Low Glasgow Coma Scale

nurse is planning the care of a 28-year-old woman hospitalized with a diagnosis of myasthenia gravis. What approach would be most appropriate for the care and scheduling

of diagnostic procedures for this patient In the morning, with frequent rest periods

To alleviate pain associated with trigeminal neuralgia, a patient is taking Tegretol (carbamazepine). What education should the nurse provide to the patient before initiating

this treatment Blood levels of the drug must be monitored.Side effects of Tegretol include nausea, dizziness, drowsiness, and aplastic anemia.

C) It protects the stomach's lining Misoprostol is a synthetic prostaglandin that, like prostaglandin, protects the gastric mucosa. NSAIDs decrease prostaglandin production and predispose the patient to peptic ulceration. Misoprostol does not reduce gastric acidity, improve emptying of the stomach, or increase lower esophageal sphincter pressure.

20. A nurse is providing patient education for a patient with peptic ulcer disease secondary to chronic nonsteroidal anti-inflammatory drug (NSAID) use. The patient has recently been prescribed misoprostol (Cytotec). What would the nurse be most accurate in informing the patient about the drug? A) It reduces the stomach's volume of hydrochloric acid B) It increases the speed of gastric emptying C) It protects the stomach's lining D) It increases lower esophageal sphincter pressure

B) Atelectasis C) Pneumonia D) Metabolic imbalances After surgery, the nurse assesses the patient for complications secondary to the surgical intervention, such as pneumonia, atelectasis, or metabolic imbalances resulting from the GI disruption. Malignant hyperthermia is an intraoperative complication. Chronic gastritis is not a surgical complication.

23. A nurse is providing care for a patient who is postoperative day 2 following gastric surgery. The nurse's assessment should be planned in light of the possibility of what potential complications? Select all that apply. A) Malignant hyperthermia B) Atelectasis C) Pneumonia D) Metabolic imbalances E) Chronic gastritis

D) Assessing the patency of the ulnar artery If a radial graft is to be performed, an Allen test on the donor arm must be performed to ensure that the ulnar artery is patent and can provide blood flow to the hand after removal of the radial artery. The success of this surgery is not primarily dependent on CN function or the absence of GERD and atherosclerosis.

25. A radial graft is planned in the treatment of a patient's oropharyngeal cancer. In order to ensure that the surgery will be successful, the care team must perform what assessment prior to surgery? A) Assessing function of cranial nerves V, VI, and IX B) Assessing for a history of GERD C) Assessing for signs or symptoms of atherosclerosis D) Assessing the patency of the ulnar artery

A nurse is preparing a patient diagnosed with benign prostatic hypertrophy (BPH) for a lower urinary tract cystoscopic examination. The nurse informs the patient that the most common temporary complication experienced after this procedure is what? A) Urinary retention B) Bladder perforation C) Hemorrhage D) Nausea

A

A nurse is working with a patient who will undergo invasive urologic testing. The nurse has informed the patient that slight hematuria may occur after the testing is complete. The nurse should recommend what action to help resolve hematuria? A) Increased fluid intake following the test B) Use of an OTC diuretic after the test C) Gentle massage of the lower abdomen D) Activity limitation for the first 12 hours after the test

A

A patient has a recent diagnosis of chronic pancreatitis and is undergoing diagnostic testing to determine pancreatic islet cell function. The nurse should anticipate what diagnostic test? A) Glucose tolerance test B) ERCP C) Pancreatic biopsy D) Abdominal ultrasonography

A

nurse is preparing to provide care for a patient diagnosed with myasthenia gravis. nurse should know that the signs and symptoms of the disease are a result of what

A lower motor neuron lesion weakness of muscles, especially in the face and throat, caused by a lower neuron lesion at the myoneural junction.

D) Apply local anesthetic to the back of the patient's throat. Preparation for UGF includes spraying or gargling with a local anesthetic. A nasogastric tube or a micro Fleet enema is not required for this procedure. The patient should be positioned in a side-lying position in case of emesis.

A nurse is caring for a patient with recurrent hematemesis who is scheduled for upper gastrointestinal fibroscopy (UGF). How should the nurse in the radiology department prepare this patient? A) Insert a nasogastric tube. B) Administer a micro Fleet enema at least 3 hours before the procedure. C) Have the patient lie in a supine position for the procedure. D) Apply local anesthetic to the back of the patient's throat.

A) Metoclopramide (Reglan) Metoclopramide (Reglan) is useful in promoting gastric motility. Omeprazole and lansoprozole are proton pump inhibitors that reduce gastric acid secretion. Famotidine (Pepcid) is an H2receptor antagonist, which has a similar effect.

A patient with GERD has undergone diagnostic testing and it has been determined that increasing the pace of gastric emptying may help alleviate symptoms. The nurse should anticipate that the patient may be prescribed what drug? A) Metoclopramide (Reglan) B) Omeprazole (Prilosec) C) Lansoprazole (Prevacid) D) Famotidine (Pepcid)

B) Attach a syringe filled with warm water and attempt an in-and-out motion of instilling and aspirating. When a tube is first noted to be clogged, a 30- to 60-mL syringe should be attached to the end of the tube and any contents aspirated and discarded. Then the syringe should be filled with warm water, attached to the tube again, and a back-and-forth motion initiated to help loosen the clog. Removal is not warranted at this early stage and a flicking motion is not recommended. The tube should not be withdrawn, even a few centimeters.

A patient's NG tube has become clogged after the nurse instilled a medication that was insufficiently crushed. The nurse has attempted to aspirate with a large-bore syringe, with no success. What should the nurse do next? A) Withdraw the NG tube 3 to 5 cm and reattempt aspiration. B) Attach a syringe filled with warm water and attempt an in-and-out motion of instilling and aspirating. C) Withdraw the NG tube slightly and attempt to dislodge by flicking the tube with the fingers. D) Remove the NG tube promptly and obtain an order for reinsertion from the primary care provider.

The nurse is planning patient teaching for a patient with ESKD who is scheduled for the creation of a fistula. The nurse would include which of the following in teaching the patient about the fistula? The nurse is planning patient teaching for a patient with ESKD who is scheduled for the creation of a fistula. The nurse would include which of the following in teaching the patient about the fistula? A) A vein and an artery in your arm will be attached surgically. B) The arm should be immobilized for 4 to 6 days. C) One needle will be inserted into the fistula for each dialysis treatment. D) The fistula can be used 2 days after the surgery for dialysis treatment.

A) A vein and an artery in your arm will be attached surgically. The fistula joins an artery and a vein, either side-to-side or end-to-end. This access will need time, usually 2 to 3 months, to mature before it can be used. The patient is encouraged to perform exercises to increase the size of the affected vessels (e.g., squeezing a rubber ball for forearm fistulas). Two needles will be inserted into the fistula for each dialysis treatment.

A patient is being discharged after a liver transplant and the nurse is performing discharge education. When planning the patient's continuing care, the nurse should prioritize which of the following risk diagnoses? A. Risk for infection related to immunosuppressant use B. Risk for injury related to decreased hemostasis C. Risk for unstable blood glucose related to impaired gluconeogensis D. Risk for contamination related to accumulation of ammonia

A. Risk for infection related to immunosuppressant use Rationale: Infection is the leading cause of death after liver transplantation. Pulmonary and fungal infections are common; susceptibility to infection is increased by the immunosuppressive therapy that is needed to prevent rejection. This risk exceeds the threats of injury and unstable blood glucose. The diagnosis of risk for contamination related to environmental toxin exposure.

A patient has developed hepatic encephalopathy secondary to cirrhosis and is receiving care on the medical unit. The patient's current medication regimen includes lactulose (Cephulac) four times daily. What desired outcome should the nurse relate to this pharmacologic intervention? A. Two to 3 soft bowel movements daily B. Significant increase in appetite and food intake C. Absence of nausea and vomiting D. Absence of blood or mucus in stool

A. Two to 3 soft bowel movements daily Rationale: Lactulose (Cephulac) is administered to reduce serum ammonia levels. Two or three soft stools per day are desirable; this indicates that lactulose is performing as intended. Lactulose does not address the patient's appetite, symptoms of nausea and vomiting, or the development of blood and mucus in the stool.

patient with herpes simplex virus encephalitis has been admitted to the ICU. What medication would the nurse expect the physician to order for the treatment of this disease process

Acyclovir (Zovirax)

D) Streaks of blood present in the stool Barium has a high osmolarity and may draw fluid into the bowel, thus increasing the intraluminal contents and resulting in greater output (large stools). The barium will give the stools a milky white appearance, and it is not uncommon for the patient to experience an increase in the number of bowel movements. Blood in fecal matter is not an expected finding and the nurse should notify the physician.

An inpatient has returned to the medical unit after a barium enema. When assessing the patient's subsequent bowel patterns and stools, what finding should the nurse report to the physician? A) Large, wide stools B) Milky white stools C) Three stools during an 8-hour period of time D) Streaks of blood present in the stool

A patient is scheduled for a myelogram and the nurse explains to the patient that this is an invasive procedure, which assesses for any lesions in the spinal cord. The nurse should explain that the preparation is similar to which of the following neurologic tests? A) Lumbar puncture B) MRI C) Cerebral angiography D) EEG

Ans: A Feedback: A myelogram is an x-ray of the spinal subarachnoid space taken after the injection of a contrast agent into the spinal subarachnoid space through a lumbar puncture. Patient preparation for a myelogram would be similar to that for lumbar puncture. The other listed diagnostic tests do not involve lumbar puncture.

A patient is postoperative day 1 following intracranial surgery. The nurse's assessment reveals that the patient's LOC is slightly decreased compared with the day of surgery. What is the nurse's best response to this assessment finding? A) Recognize that this may represent the peak of post-surgical cerebral edema. B) Alert the surgeon to the possibility of an intracranial hemorrhage. C) Understand that the surgery may have been unsuccessful. D) Recognize the need to refer the patient to the palliative care team.

Ans: A Feedback: Some degree of cerebral edema occurs after brain surgery; it tends to peak 24 to 36 hours after surgery, producing decreased responsiveness on the second postoperative day. As such, there is not necessarily any need to deem the surgery unsuccessful or to refer the patient to palliative care. A decrease in LOC is not evidence of an intracranial hemorrhage.

A patient is admitted to the medical unit with an exacerbation of multiple sclerosis. When assessing this patient, the nurse has the patient stick out her tongue and move it back and forth. What is the nurse assessing? A) Function of the hypoglossal nerve B) Function of the vagus nerve C) Function of the spinal nerve D) Function of the trochlear nerve

Ans: A Feedback: The hypoglossal nerve is the 12th cranial nerve. It is responsible for movement of the tongue. None of the other listed nerves affects motor function in the tongue.

A patient with elevated BUN and creatinine values has been referred by her primary physician for further evaluation. The nurse should anticipate the use of what initial diagnostic test? A) Ultrasound B) X-ray C) Computed tomography (CT) D) Nuclear scan

A

A patient with a diagnosis of respiratory acidosis is experiencing renal compensation. What function does the kidney perform to assist in restoring acid-base balance? A) Sequestering free hydrogen ions in the nephrons B) Returning bicarbonate to the body's circulation C) Returning acid to the body's circulation D) Excreting bicarbonate in the urine

B

D) Monthly administration of injections of vitamin B12 Since vitamin B12 is absorbed in the stomach, the patient requires vitamin B12replacement to prevent pernicious anemia. A gastrectomy precludes the use of a G tube. Since the stomach is absent, a nasogastric tube would not be indicated. As well, this is not possible in the home setting. Since there is no stomach to act as a reservoir and fluids and nutrients are passing directly into the jejunum, distension is unlikely.

9. A community health nurse is preparing for an initial home visit to a patient discharged following a total gastrectomy for treatment of gastric cancer. What would the nurse anticipate that the plan of care is most likely to include? A) Enteral feeding via gastrostomy tube (G tube) B) Gastrointestinal decompression by nasogastric tube C) Periodic assessment for esophageal distension D) Monthly administration of injections of vitamin B12

A nurse is assisting with serving dinner trays on the unit. Upon receiving the dinner tray for a patient admitted with acute gallbladder inflammation, the nurse will question which of the following foods on the tray? A) Fried chicken B) Mashed potatoes C) Dinner roll D) Tapioca pudding

A

A patient with end-stage liver disease has developed hypervolemia. What nursing interventions would be most appropriate when addressing the patient's fluid volume excess? Select all that apply. A. Administering diuretics B. Administering calcium channel blockers C. Implementing fluid restrictions D. Implementing a 1500 kcal/day restriction E. Enhancing patient positioning

A. Administering diuretics B. Administering calcium channel blockers E. Enhancing patient positioning Rationale: Administering diuretics, implementing fluid restrictions, and enhancing patient positioning can optimize the management of fluid volume excess. Calcium channel blockers and calorie restriction do not address this problem.

A 55-year-old female patient with hepatocellular carcinoma (HCC) is undergoing radiofrequency ablation. The nurse should recognize what goal of this treatment? A. Destruction of the patient's liver tumor B. Restoration of portal vein patency C. Destruction of a liver abscess D. Reversal of metastasis

A. Destruction of the patient's liver tumor Rationale: Using radiofrequency ablation, a tumor up to 5 cm in size can be destroyed in one treatment session. This technique does not address circulatory function or abscess formation. It does not allow for the reversal of metastasis.

A nurse on a solid organ transplant unit is planning the care of a patient who will soon be admitted upon immediate recovery following liver transplantation. What aspect of nursing care is the nurse's priority? A. Implementation of infection-control measures B. Close monitoring of skin integrity and color C. Frequent assessment of the patient's psychosocial status D. Administration of antiretroviral medications

A. Implementation of infection-control measures Rationale: Infection control is paramount following liver transplantation. This is a priority over skin integrity and psychosocial status, even though these are valid areas of assessment and intervention. Antiretrovirals are not indicated.

A patient with a liver mass is undergoing a percutaneous liver biopsy. What action should the nurse perform when assisting with this procedure? A. Position the patient on the right side with a pillow under the costal margin after the procedure B. Administer 1 unit of albumin 90 minutes before the procedure as ordered C. Administer at least 1 unit of packed red blood cells as ordered the day before the scheduled procedure D. Confirm that the patient's electrolyte levels have been assessed prior to the procedure

A. Position the patient on the right side with a pillow under the costal margin after the procedure. Rationale: Immediately after a percutaneous liver biopsy, assist the patient to turn onto the right side and place a pillow under the costal margin. Prior administration of albumin or PRBC's is unnecessary. Coagulation tests should be performed, but electrolyte analysis is not necessary.

A patient with chronic pancreatitis had a pancreaticojejunostomy created 3 months ago for relief of pain and to restore drainage of pancreatic secretions. The patient has come to the office for a routine postsurgical appointment. The patient is frustrated that the pain has not decreased. What is the most appropriate initial response by the nurse? A) The majority of patients who have a pancreaticojejunostomy have their normal digestion restored but do not achieve pain relief. B) Pain relief occurs by 6 months in most patients who undergo this procedure, but some people experience a recurrence of their pain. C) Your physician will likely want to discuss the removal of your gallbladder to achieve pain relief. D) You are probably not appropriately taking the medications for your pancreatitis and pain, so we will need to discuss your medication regimen in detail.

B

A patient with gallstones has been prescribed ursodeoxycholic acid (UDCA). The nurse understands that additional teaching is needed regarding this medication when the patient states: A) It is important that I see my physician for scheduled follow-up appointments while taking this medication. B) I will take this medication for 2 weeks and then gradually stop taking it. C) If I lose weight, the dose of the medication may need to be changed. D) This medication will help dissolve small gallstones made of cholesterol.

B

A patient with renal failure secondary to diabetic nephropathy has been admitted to the medical unit. What is the most life-threatening effect of renal failure for which the nurse should monitor the patient? A) Accumulation of wastes B) Retention of potassium C) Depletion of calcium D) Lack of BP control

B

The nurse is reviewing the electronic health record of a patient with a history of incontinence. The nurse reads that the physician assessed the patient's deep tendon reflexes. What condition of the urinary/renal system does this assessment address? A) Renal calculi B) Bladder dysfunction C) Benign prostatic hyperplasia (BPH) D) Recurrent urinary tract infections (UTIs)

B

The nurse caring for a patient with suspected renal dysfunction calculates that the patient's weight has increased by 5 pounds in the past 24 hours. The nurse estimates that the patient has retained approximately how much fluid? A) 1,300 mL of fluid in 24 hours B) 2,300 mL of fluid in 24 hours C) 3,100 mL of fluid in 24 hours D) 5,000 mL of fluid in 24 hours

B (An increase in body weight commonly accompanies edema. To calculate the approximate weight gain from fluid retention, remember that 1 kg of weight gain equals approximately 1,000 mL of fluid. Five lbs = 2.27 kg = 2,270 mL.)

The nurse coming on shift on the medical unit is taking a report on four patients. What patient does the nurse know is at the greatest risk of developing ESKD? A) A patient with a history of polycystic kidney disease B) A patient with diabetes mellitus and poorly controlled hypertension C) A patient who is morbidly obese with a history of vascular disorders D) A patient with severe chronic obstructive pulmonary disease

B) A patient with diabetes mellitus and poorly controlled hypertension Systemic diseases, such as diabetes mellitus (leading cause); hypertension; chronic glomerulonephritis; pyelonephritis; obstruction of the urinary tract; hereditary lesions, such as in polycystic kidney disease; vascular disorders; infections; medications; or toxic agents may cause ESKD. A patient with more than one of these risk factors is at the greatest risk for developing ESKD. Therefore, the patient with diabetes and hypertension is likely at highest risk for ESKD.

A patient is brought to the renal unit from the PACU status post resection of a renal tumor. Which of the following nursing actions should the nurse prioritize in the care of this patient? A) Increasing oral intake B) Managing postoperative pain C) Managing dialysis D) Increasing mobility

B) Managing postoperative pain The patient requires frequent analgesia during the postoperative period and assistance with turning, coughing, use of incentive spirometry, and deep breathing to prevent atelectasis and other pulmonary complications. Increasing oral intake and mobility are not priority nursing actions in the immediate postoperative care of this patient. Dialysis is not necessary following kidney surgery.

A patient is scheduled for a CT scan of the abdomen with contrast. The patient has a baseline creatinine level of 2.3 mg/dL. In preparing this patient for the procedure, the nurse anticipates what orders? A) Monitor the patients electrolyte values every hour before the procedure. B) Preprocedure hydration and administration of acetylcysteine C) Hemodialysis immediately prior to the CT scan D) Obtain a creatinine clearance by collecting a 24-hour urine specimen.

B) Preprocedure hydration and administration of acetylcysteine Radiocontrast-induced nephropathy is a major cause of hospital-acquired acute kidney injury. Baseline levels of creatinine greater than 2 mg/dL identify the patient as being high risk. Preprocedure hydration and prescription of acetylcysteine (Mucomyst) the day prior to the test is effective in prevention. The nurse would not monitor the patients electrolytes every hour preprocedure. Nothing in the scenario indicates the need for hemodialysis. A creatinine clearance is not necessary prior to a CT scan with contrast.

A patient has been admitted to the critical care unit with a diagnosis of toxic hepatitis. When planning the patient's care, the nurse should be aware of what potential clinical course of this health problem? Place the following events in the correct sequence. 1. Fever rises 2. Hematemesis 3. Clotting abnormalities 4. Vascular collapse 5. Coma A. 1,2,5,4,3 B. 1,2,3,4,5 C. 2,3,1,4,5 D. 3,1,2,5,4

B. 1,2,3,4,5 Rationale: Recovery from acute toxic hepatitis is rapid if the hepatotoxin is identified early and removed or if exposure to the agent has been limited. Recovery is unlikely if there is a prolonged period between exposure and onset of symptoms. There are no effective antidotes. The fever rises; the patient becomes toxic and prostrated. Vomiting may be persistent, with the emesis containing blood. Clotting abnormalities may be severe, and hemorrhages may appear under the skin. The severe GI symptoms may lead to vascular collapse. Delirium, coma, and seizures develop, and within a few days the patient may die of fulminant hepatic failure unless he or she receives a liver transplant.

The nurse is caring for a patient who had a brush biopsy 12 hours ago. The presence of what assessment finding should prompt the nurse to notify the physician? A) Scant hematuria B) Renal colic C) Temperature 100.2°F orally D) Infiltration of the patient's intravenous catheter

C

A patient with a diagnosis of esophageal varices has undergone endoscopy to gauge the progression of this complication of liver disease. Following the completion of this diagnostic test, what nursing intervention should the nurse perform? A. Keep patient NPO until the results of test are known B. Keep patient NPO until the patient's gag reflex returns C. Administer analgesia until post-procedure tenderness is relieved D. Give the patient a cold beverage to promote swallowing ability

B. Keep patient NPO until the patient's gag reflex returns Rationale: After the examination, fluids are not given until the patient's gag reflex returns. Lozenges and gargles may be used to relieve throat discomfort if the patient's physical condition and mental status permit. The result of the test is known immediately. Food and fluids are contraindicated until the gag reflex returns.

A nurse is caring for a patient with a blocked bile duct from a tumor. What manifestation of obstructive jaundice should the nurse anticipate? A. Watery, blood-streaked diarrhea B. Orange and foamy urine C. Increased abdominal girth D. Decreased cognition

B. Orange and foamy urine Rationale: If the bile duct is obstructed, the bile will be reabsorbed into the blood and carried throughout the entire body. It is excreted in the urine, which becomes deep orange and foamy. Bloody diarrhea, ascites, and cognitive changes are not associated with obstructive jaundice.

A patient has been admitted to the hospital for the treatment of chronic pancreatitis. The patient has been stabilized and the nurse is now planning health promotion and educational interventions. Which of the following should the nurse prioritize? A) Educating the patient about expectations and care following surgery B) Educating the patient about the management of blood glucose after discharge C) Educating the patient about postdischarge lifestyle modifications D) Educating the patient about the potential benefits of pancreatic transplantation

C

A patient has just been diagnosed with chronic pancreatitis. The patient is underweight and in severe pain and diagnostic testing indicates that over 80% of the patients pancreas has been destroyed. The patient asks the nurse why the diagnosis was not made earlier in the disease process. What would be the nurses best response? A) The symptoms of pancreatitis mimic those of much less serious illnesses. B) Your body doesnt require pancreatic function until it is under great stress, so it is easy to go unnoticed. C) Chronic pancreatitis often goes undetected until a large majority of pancreatic function is lost. D) Its likely that your other organs were compensating for your decreased pancreatic function.

C

The staff educator is giving a class for a group of nurses new to the renal unit. The educator is discussing renal biopsies. In what patient would the educator tell the new nurses that renal biopsies are contraindicated? A) A 64-year-old patient with chronic glomerulonephritis B) A 57-year-old patient with proteinuria C) A 42-year-old patient with morbid obesity D) A 16-year-old patient with signs of kidney transplant rejection

C

The nurse is caring for a patient in acute kidney injury. Which of the following complications would most clearly warrant the administration of polystyrene sulfonate (Kayexalate)? A) Hypernatremia B) Hypomagnesemia C) Hyperkalemia D) Hypercalcemia

C) Hyperkalemia Hyperkalemia, a common complication of acute kidney injury, is life-threatening if immediate action is not taken to reverse it. The administration of polystyrene sulfonate reduces serum potassium levels.

A patient has a glomerular filtration rate (GFR) of 43 mL/min/1.73 m2. Based on this GFR, the nurse interprets that the patients chronic kidney disease is at what stage? A) Stage 1 B) Stage 2 C) Stage 3 D) Stage 4

C) Stage 3 Stages of chronic renal failure are based on the GFR. Stage 3 is defined by a GFR in the range of 30 to 59 mL/min/1.73 m2. This is considered a moderate decrease in GFR.

The nurse is caring for acutely ill patient. What assessment finding should prompt the nurse to inform the physician that the patient may be exhibiting signs of acute kidney injury (AKI)? A) The patient is complains of an inability to initiate voiding. B) The patients urine is cloudy with a foul odor. C) The patients average urine output has been 10 mL/hr for several hours. D) The patient complains of acute flank pain.

C) The patients average urine output has been 10 mL/hr for several hours. Oliguria (<500 mL/d of urine) is the most common clinical situation seen in AKI. Flank pain and inability to initiate voiding are not characteristic of AKI. Cloudy, foul-smelling urine is suggestive of a urinary tract infection.

A nurse is caring for a patient with severe hemolytic jaundice. Laboratory tests show free bilirubin to be 24 mg/dL. For what complication is this patient at risk? A. Chronic jaundice B. Pigment stones in portal circulation C. Central nervous system damage D. Hepatomegaly

C. Central nervous system damage Rationale: Prolonged jaundice, even if mild, predisposes to the formation of pigment stones in the gallbladder, and extremely severe jaundice (levels of free bilirubin exceeding 20 to 25 mg/dL) poses a risk for CNS damage. There are not specific risks of hepatomegaly or chronic jaundice resulting from high bilirubin

A patient with portal hypertension has been admitted to the medical floor. The nurse should prioritize which of the following assessments related to the manifestations of this health problem? A. Assessment of blood pressure and assessment for headaches and visual changes. B. Assessments for signs and symptoms of venous thromboembolism. C. Daily weights and abdominal girth measurement. D. Blood glucose monitoring q4h.

C. Daily weights and abdominal girth measurement Rationale: Obstruction to blood flow through the damaged liver results in increased blood pressure (portal hypertension) throughout the portal venous system. This can result in varices and ascites in the abdominal cavity. Assessments related to ascites are daily weights and abdominal girths. Portal hypertension is not synonymous with cardiovascular hypertension and does not create a risk for unstable blood glucose of VTE.

A patient with a diagnosis of cirrhosis has developed variceal bleeding and will imminently undergo variceal banding. What psychosocial nursing diagnosis should the nurse most likely prioritize during this phase of the patient's treatment? A. Decisional Conflict B. Deficient Knowledge C. Death Anxiety D. Disturbed Thought Processes

C. Death Anxiety Rationale: The sudden hemorrhage that accompanies variceal bleeding is intensely anxiety-provoking. The nurse must address the patient's likely fear of death, which is a realistic possibility. For most patients, anxiety is likely to be a more acute concern than lack of knowledge or decisional conflict. The patient may or may not experience disturbances in thought processes.

A nurse has entered the room of a patient with cirrhosis and found the patient on the floor. The patient stated that she fell when transferring to the commode. The patient's vital signs are within reference ranges and the nurse observes no apparent injuries. What is the nurse's most appropriate action? A. Remove the patient's commode and supply a bedpan B. Complete an incident report and submit it to the unit supervisor C. Have the patient assessed by the physician due to the risk of internal bleeding D. Perform a focused abdominal assessment in order to rule out injury.

C. Have the patient assessed by the physician due to the risk of internal bleeding Rationale: A fall would necessitate thorough medical assessment due to the patient's risk of bleeding. The nurse's abdominal assessment is an appropriate action, but is not wholly sufficient to rule out internal injury. Medical assessment is a priority over removing the commode or filling out an incident report, even though these actions are appropriate.

A nurse is performing an admission assessment of a patient with a diagnosis of cirrhosis. What technique should the nurse use to palpate the patient's liver? A. Place hand under the right lower abdominal quadrant and press down lightly with the other hand. B. Place the left hand over the abdomen and behind the left side at the 11th rib. C. Place hand under right lower rib cage and press down lightly with the other hand. D. Hold hand 90 degree to right side of the abdomen and push down firmly.

C. Place hand under right lower rib cage and press down lightly with the other hand. Rationale: To palpate the liver, the examiner places one hand under the right lower rib cage and presses downward with light pressure with the other hand. The liver is not on the left side or in the right lower abdominal quadrant.

A home health nurse is caring for a patient discharged home after pancreatic surgery. The nurse documents the nursing diagnosis Risk for Imbalanced Nutrition: Less than Body Requirements on the care plan based on the potential complications that may occur after surgery. What are the most likely complications for the patient who has had pancreatic surgery? A) Proteinuria and hyperkalemia B) Hemorrhage and hypercalcemia C) Weight loss and hypoglycemia D) Malabsorption and hyperglycemia

D

A kidney biopsy has been scheduled for a patient with a history of acute renal failure. The patient asks the nurse why this test has been scheduled. What is the nurse's best response? A) "A biopsy is routinely ordered for all patients with renal disorders." B) "A biopsy is generally ordered following abnormal x-ray findings of the renal pelvis." C) "A biopsy is often ordered for patients before they have a kidney transplant." D) "A biopsy is sometimes necessary for diagnosing and evaluating the extent of kidney disease."

D

A nurse is assessing a patient who has been diagnosed with cholecystitis, and is experiencing localized abdominal pain. When assessing the characteristics of the patients pain, the nurse should anticipate that it may radiate to what region? A) Left upper chest B) Inguinal region C) Neck or jaw D) Right shoulder

D

A nurse is assessing an elderly patient with gallstones. The nurse is aware that the patient may not exhibit typical symptoms, and that particular symptoms that may be exhibited in the elderly patient may include what? A) Fever and pain B) Chills and jaundice C) Nausea and vomiting D) Signs and symptoms of septic shock

D

A patient has been diagnosed with pancreatic cancer and has been admitted for care. Following initial treatment, the nurse should be aware that the patient is most likely to require which of the following? A) Inpatient rehabilitation B) Rehabilitation in the home setting C) Intensive physical therapy D) Hospice care

D

A patient has been treated in the hospital for an episode of acute pancreatitis. The patient has acknowledged the role that his alcohol use played in the development of his health problem, but has not expressed specific plans for lifestyle changes after discharge. What is the nurses most appropriate response? A) Educate the patient about the link between alcohol use and pancreatitis. B) Ensure that the patient knows the importance of attending follow-up appointments. C) Refer the patient to social work or spiritual care. D) Encourage the patient to connect with a community-based support group.

D

A patient is being treated on the acute medical unit for acute pancreatitis. The nurse has identified a diagnosis of Ineffective Breathing Pattern Related to Pain. What intervention should the nurse perform in order to best address this diagnosis? A) Position the patient supine to facilitate diaphragm movement. B) Administer corticosteroids by nebulizer as ordered. C) Perform oral suctioning as needed to remove secretions. D) Maintain the patient in a semi-Fowlers position whenever possible.

D

A patient with a history of incontinence will undergo urodynamic testing in the physician's office. Because voiding in the presence of others can cause situational anxiety, the nurse should perform what action?' A) Administer diuretics as ordered. B) Push fluids for several hours prior to the test. C) Discuss possible test results as the patient voids. D) Help the patient to relax before and during the test.

D

A patient's most recent laboratory findings indicate a glomerular filtration rate (GFR) of 58 mL/min. The nurse should recognize what implication of this diagnostic finding? A) The patient is likely to have a decreased level of blood urea nitrogen (BUN). B) The patient is at risk for hypokalemia. C) The patient is likely to have irregular voiding patterns. D) The patient is likely to have increased serum creatinine levels.

D

The nurse is caring for a patient who has a fluid volume deficit. When evaluating this patient's urinalysis results, what should the nurse anticipate? A) A fluctuating urine specific gravity B) A fixed urine specific gravity C) A decreased urine specific gravity D) An increased urine specific gravity

D

The nurse is caring for a patient who is going to have an open renal biopsy. What would be an important nursing action in preparing this patient for the procedure? A) Discuss the patient's diagnosis with the family. B) Bathe the patient before the procedure with antiseptic skin wash. C) Administer antivirals before sending the patient for the procedure. D) Keep the patient NPO prior to the procedure.

D

What nursing action should the nurse perform when caring for a patient undergoing diagnostic testing of the renal-urologic system? A) Withhold medications until 12 hours post-testing. B) Ensure that the patient knows the importance of temporary fluid restriction after testing. C) Inform the patient of his or her medical diagnosis after reviewing the results. D) Assess the patient's understanding of the test results after their completion.

D

A patient admitted with nephrotic syndrome is being cared for on the medical unit. When writing this patients care plan, based on the major clinical manifestation of nephrotic syndrome, what nursing diagnosis should the nurse include? A) Constipation related to immobility B) Risk for injury related to altered thought processes C) Hyperthermia related to the inflammatory process D) Excess fluid volume related to generalized edema

D) Excess fluid volume related to generalized edema The major clinical manifestation of nephrotic syndrome is edema, so the appropriate nursing diagnosis is Excess fluid volume related to generalized edema. Edema is usually soft, pitting, and commonly occurs around the eyes, in dependent areas, and in the abdomen.

The nurse performing the health interview of a patient with a new onset of periorbital edema has completed a genogram, noting the health history of the patients siblings, parents, and grandparents. This assessment addresses the patients risk of what kidney disorder? A) Nephritic syndrome B) Acute glomerulonephritis C) Nephrotic syndrome D) Polycystic kidney disease (PKD)

D) Polycystic kidney disease (PKD) PKD is a genetic disorder characterized by the growth of numerous cysts in the kidneys. Nephritic syndrome, acute glomerulonephritis, and nephrotic syndrome are not genetic disorders.

A 15-year-old is admitted to the renal unit with a diagnosis of postinfectious glomerular disease. The nurse should recognize that this form of kidney disease may have been precipitated by what event? A) Psychosocial stress B) Hypersensitivity to an immunization C) Menarche D) Streptococcal infection

D) Streptococcal infection Postinfectious causes of postinfectious glomerular disease are group A beta-hemolytic streptococcal infection of the throat that precedes the onset of glomerulonephritis by 2 to 3 weeks. Menarche, stress, and hypersensitivity are not typical causes.

The care team is considering the use of dialysis in a patient whose renal function is progressively declining. Renal replacement therapy is indicated in which of the following situations?

D) When about 80% of the nephrons are no longer functioning

A local public health nurse is informed that a cook in a local restaurant has been diagnosed with hepatitis A. What should the nurse advise individuals to obtain who ate at this restaurant and have never received the hepatitis A vaccine? A. The hepatitis A vaccine B. Albumin infusion C. The hepatitis A and B vaccines D. An immune globulin injection

D. An immune globulin injection Rationale: For people who have not been previously vaccinated, hepatitis A can be prevented by the intramuscular administration of immune globulin during the incubation period, if given within 2 weeks of exposure. Administration of the hepatitis A vaccine will not protect the patient exposed to hepatitis A, as protection will take a few weeks to develop after the first dose of the vaccine. The hepatitis B vaccine provides protection against the hepatitis B virus, but plays no role in protection for the patient exposed to hepatitis A. Albumin confers no therapeutic benefit.

A nurse is caring for a patient with liver failure and is performing an assessment in the knowledge of the patient's increased risk of bleeding. The nurse recognizes that this risk is related to the patient's inability to synthesize prothrombin in the liver. What factor most likely contributes to this loss of function? A. Alterations in glucose metabolism B. Retention of bile salts C. Inadequate production of albumin by hepatocytes D. Inability of the liver to use vitamin K.

D. Inability of the liver to use vitamin K. Rationale: Decreased production of several clotting factors may be partially due to deficient absorption of vitamin K from the GI tract. This probably is caused by the inability of liver cells to use vitamin K to make prothrombin. This bleeding risk is unrelated to the roles of glucose, bile salts, or albumin.

A patient with esophageal varices is being cared for in the ICU. The varices have begun to bleed and the patient is at risk for hypovolemia. The patient has Ringer's Lactate at 150 cc/hr infusing. What else might the nurse expect to have ordered to maintain volume for this patient? A. Arterial line B. Diuretics C. Foley catheter D. Volume expanders

D. Volume expanders Rationale: Because patients with bleeding esophageal varices have intravascular volume depletion and are subject to electrolyte imbalance, IV fluids with electrolytes and volume expanders are provided to restore fluid volume and replace electrolytes. Diuretics would reduce vascular volume. An arterial line and Foley catheter are likely to be ordered, but neither actively maintains the patient's volume.

An adult patient has been admitted to the medical unit for the treatment of acute pancreatitis. What nursing action should be included in this patients plan of care? A) Measure the patients abdominal girth daily. B) Limit the use of opioid analgesics. C) Monitor the patient for signs of dysphagia. D) Encourage activity as tolerated.

A

The nurse is assessing a patient's bladder by percussion. The nurse elicits dullness after the patient has voided. How should the nurse interpret this assessment finding? A) The patient's bladder is not completely empty. B) The patient has kidney enlargement. C) The patient has a ureteral obstruction. D) The patient has a fluid volume deficit.

A

The nurse is providing pre-procedure teaching about an ultrasound. The nurse informs the patient that in preparation for an ultrasound of the lower urinary tract the patient will require what? A) Increased fluid intake to produce a full bladder B) IV administration of radiopaque contrast agent C) Sedation and intubation D) Injection of a radioisotope

A

The nurse is assessing a patient with a suspected stroke. What assessment finding is most suggestive of a stroke? A) Facial droop B) Dysrhythmias C) Periorbital edema D) Projectile vomiting

Ans: A Feedback: Facial drooping or asymmetry is a classic abnormal finding on a physical assessment that may be associated with a stroke. Facial edema is not suggestive of a stroke and patients less commonly experience dysrhythmias or vomiting.

A) Place distal tip to nose, then ear tip and end of xiphoid process. Tube length is traditionally determined by (1) measuring the distance from the tip of the nose to the earlobe and from the earlobe to the xiphoid process, and (2) adding up to 6 inches for NG placement or at least 8 to 10 inches or more for intestinal placement, although studies do not necessarily confirm that this is a reliable technique. The physician would not prescribe a specific length and the umbilicus is not a landmark for this process. Length is not determined by aspirating from the tube.

1. A nurse is preparing to place a patient's ordered nasogastric tube. How should the nurse best determine the correct length of the nasogastric tube? A) Place distal tip to nose, then ear tip and end of xiphoid process. B) Instruct the patient to lie prone and measure tip of nose to umbilical area. C) Insert the tube into the patient's nose until secretions can be aspirated. D) Obtain an order from the physician for the length of tube to insert.

A 33-year-old patient presents at the clinic with complaints of weakness, incoordination, dizziness, and loss of balance. The patient is hospitalized and diagnosed with MS. What

sign or symptom, revealed during the initial assessment, is typical of MS Blurred vision, intention tremor, and urinary hesitancy

B) Infection typically occurs due to ingestion of contaminated food and water. Most peptic ulcers result from infection with the gram-negative bacteria H. pylori, which may be acquired through ingestion of food and water. The organism is endemic to all areas of the United States. Genetic factors have not been identified.

10. A nurse is assessing a patient who has peptic ulcer disease. The patient requests more information about the typical causes of Helicobacter pylori infection. What would it be appropriate for the nurse to instruct the patient? A) Most affected patients acquired the infection during international travel. B) Infection typically occurs due to ingestion of contaminated food and water. C) Many people possess genetic factors causing a predisposition to H. pylori infection. D) The H. pylori microorganism is endemic in warm, moist climates.

A) Inspection, auscultation, percussion, and palpation Feedback: When performing a focused assessment of the patient's abdomen, auscultation should always precede percussion and palpation because they may alter bowel sounds. The traditional sequence for all other focused assessments is inspection, palpation, percussion, and auscultatio

10. The nurse is preparing to perform a patient's abdominal assessment. What examination sequence should the nurse follow? A) Inspection, auscultation, percussion, and palpation B) Inspection, palpation, auscultation, and percussion C) Inspection, percussion, palpation, and auscultation D) Inspection, palpation, percussion, and auscultation

C) The family's ability to provide emotional support Emotional support from the family is key to the patient's coping after discharge. A 21-year-old would be expected to self-manage the prescribed medication regimen and the family would not be primarily responsible for monitoring the patient's health status. It is highly beneficial if the family is willing and able to accommodate the patient's dietary needs, but emotional support is paramount and cannot be solely provided by the patient alone.

11. A nurse is planning discharge teaching for a 21-year-old patient with a new diagnosis of ulcerative colitis. When planning family assessment, the nurse should recognize that which of the following factors will likely have the greatest impact on the patient's coping after discharge? A) The family's ability to take care of the patient's special diet needs B) The family's ability to monitor the patient's changing health status C) The family's ability to provide emotional support D) The family's ability to manage the patient's medication regimen

A) A 65-year-old man with alcoholism who smokes Oral cancers are often associated with the use of alcohol and tobacco, which when used together have a synergistic carcinogenic effect. Most cases of oral cancers occur in people over the age of 60 and a disproportionate number of cases occur in men. Diabetes, dentures, dental caries, and GERD are not risk factors for oral cancer.

11. A nurse who provides care in an ambulatory clinic integrates basic cancer screening into admission assessments. What patient most likely faces the highest immediate risk of oral cancer? A) A 65-year-old man with alcoholism who smokes B) A 45-year-old woman who has type 1 diabetes and who wears dentures C) A 32-year-old man who is obese and uses smokeless tobacco D) A 57-year-old man with GERD and dental caries

C) Increase fluid intake to evacuate the barium. Adequate fluid intake is necessary to rid the GI tract of barium. The patient must not remain NPO after the test and enemas are not used to cleanse the bowel of barium. There is no need to avoid dairy products.

11. A patient who has been experiencing changes in his bowel function is scheduled for a barium enema. What instruction should the nurse provide for postprocedure recovery? A) Remain NPO for 6 hours postprocedure. B) Administer a Fleet enema to cleanse the bowel of the barium. C) Increase fluid intake to evacuate the barium. D) Avoid dairy products for 24 hours postprocedure.

B) Nontunneled central catheter Nontunneled central catheters are used for short-term (less than 6 weeks) IV therapy. A peripheral catheter can be used for the administration of peripheral parenteral nutrition for 5 to 7 days. Implantable ports and tunneled central catheters are for long-term use and may remain in place for many years. Peripherally inserted central catheters (PICCs) are another potential option.

11. A patient's physician has determined that for the next 3 to 4 weeks the patient will require parenteral nutrition (PN). The nurse should anticipate the placement of what type of venous access device? A) Peripheral catheter B) Nontunneled central catheter C) Implantable port D) Tunneled central catheter

B) Upper GI tract Blood shed in sufficient quantities in the upper GI tract will produce a tarry-black color (melena). Blood entering the lower portion of the GI tract or passing rapidly through it will appear bright or dark red. Lower rectal or anal bleeding is suspected if there is streaking of blood on the surface of the stool or if blood is noted on toilet tissue.

12. A nurse is caring for a newly admitted patient with a suspected GI bleed. The nurse assesses the patient's stool after a bowel movement and notes it to be a tarry-black color. This finding is suggestive of bleeding from what location? A) Sigmoid colon B) Upper GI tract C) Large intestine D) Anus or rectum

A) Administer an isotonic dextrose solution for 1 to 2 hours after discontinuing the PN After administration of the PN solution is gradually discontinued, an isotonic dextrose solution is administered for 1 to 2 hours to protect against rebound hypoglycemia. The other listed actions would likely cause hyperglycemia.

12. A nurse is caring for a patient who has an order to discontinue the administration of parenteral nutrition. What should the nurse do to prevent the occurrence of rebound hypoglycemia in the patient? A) Administer an isotonic dextrose solution for 1 to 2 hours after discontinuing the PN. B) Administer a hypertonic dextrose solution for 1 to 2 hours after discontinuing the PN. C) Administer 3 ampules of dextrose 50% immediately prior to discontinuing the PN. D) Administer 3 ampules of dextrose 50% 1 hour after discontinuing the PN.

B) Promptly report these indications of venous congestion. A graft that is blue with mottling may indicate venous congestion. This finding constitutes a risk for tissue ischemia and necrosis; prompt referral is necessary.

12. A nurse is caring for a patient who has undergone neck resection with a radial forearm free flap. The nurse's most recent assessment of the graft reveals that it has a bluish color and that mottling is visible. What is the nurse's most appropriate action? A) Document the findings as being consistent with a viable graft. B) Promptly report these indications of venous congestion. C) Closely monitor the patient and reassess in 30 minutes. D) Reposition the patient to promote peripheral circulation.

A) Infection with Helicobacter pylori H. pylori infection may be determined by endoscopy and histologic examination of a tissue specimen obtained by biopsy, or a rapid urease test of the biopsy specimen. Excessive stomach acid secretion leads to gastritis; however, peptic ulcers are caused by colonization of the stomach by H. pylori. Sphincter dysfunction and acid-base imbalances do not cause peptic ulcer disease.

12. A patient presents to the walk-in clinic complaining of vomiting and burning in her mid-epigastria. The nurse knows that in the process of confirming peptic ulcer disease, the physician is likely to order a diagnostic test to detect the presence of what? A) Infection with Helicobacter pylori B) Excessive stomach acid secretion C) An incompetent pyloric sphincter D) A metabolic acid-base imbalance

B) Assess the patient's food and fluid intake. The nurse should follow the nursing process and perform an assessment prior to interventions. The patient's food and fluid intake is more likely to affect bowel function than surgery.

12. An older adult who resides in an assisted living facility has sought care from the nurse because of recurrent episodes of constipation. Which of the following actions should the nurse first perform? A) Encourage the patient to take stool softener daily. B) Assess the patient's food and fluid intake. C) Assess the patient's surgical history. D) Encourage the patient to take fiber supplements.

B) Risk for Infection Related to Possible Rupture of Appendix The patient with a diagnosis of appendicitis has an acute risk of infection related to the possibility of rupture. This immediate physiologic risk is a priority over nutrition and constipation, though each of these concerns should be addressed by the nurse. The pain associated with appendicitis is acute, not chronic.

13. A 16-year-old presents at the emergency department complaining of right lower quadrant pain and is subsequently diagnosed with appendicitis. When planning this patient's nursing care, the nurse should prioritize what nursing diagnosis? A) Imbalanced Nutrition: Less Than Body Requirements Related to Decreased Oral Intake B) Risk for Infection Related to Possible Rupture of Appendix C) Constipation Related to Decreased Bowel Motility and Decreased Fluid Intake D) Chronic Pain Related to Appendicitis

B) Risk for Infection Related to the Presence of a Subclavian Catheter The high glucose content of PN solutions makes the solutions an idea culture media for bacterial and fungal growth, and the central venous access devices provide a port of entry. Prevention of infection is consequently a high priority. The patient will experience some inconveniences with regard to toileting, activity, and sleep, but the infection risk is a priority over each of these.

13. A nurse is caring for a patient with a subclavian central line who is receiving parenteral nutrition (PN). In preparing a care plan for this patient, what nursing diagnosis should the nurse prioritize? A) Risk for Activity Intolerance Related to the Presence of a Subclavian Catheter B) Risk for Infection Related to the Presence of a Subclavian Catheter C) Risk for Functional Urinary Incontinence Related to the Presence of a Subclavian Catheter D) Risk for Sleep Deprivation Related to the presence of a Subclavian Catheter

B) Hypoactive Documenting bowel sounds is based on assessment findings. The terms normal (sounds heard about every 5 to 20 seconds), hypoactive (one or two sounds in 2 minutes), hyperactive (5 to 6 sounds heard in less than 30 seconds), or absent (no sounds in 3 to 5 minutes) are frequently used in documentation. Paralytic ileus is a medical diagnosis that may cause absent or hypoactive bowel sounds, but the nurse would not independently document this diagnosis.

13. A nursing student has auscultated a patient's abdomen and noted one or two bowel sounds in a 2-minute period of time. How would you tell the student to document the patient's bowel sounds? A) Normal B) Hypoactive C) Hyperactive D) Paralytic ileus

D) Avoid drinking alcohol while taking the drug. Alcohol must be avoided when taking Flagyl and the medication should be taken with food. This drug does not cause drowsiness and the dose should not be adjusted by the patient.

13. A patient with a peptic ulcer disease has had metronidazole (Flagyl) added to his current medication regimen. What health education related to this medication should the nurse provide? A) Take the medication on an empty stomach. B) Take up to one extra dose per day if stomach pain persists. C) Take at bedtime to mitigate the effects of drowsiness. D) Avoid drinking alcohol while taking the drug.

A) Chemical phlebitis Formulations with dextrose concentrations of more than 10% should not be administered through peripheral veins because they irritate the intima (innermost walls) of small veins, causing chemical phlebitis. Hyperglycemia and line sepsis are risks with both peripheral and central administration of PN. PN is not associated with dumping syndrome.

14. A patient's health decline necessitates the use of total parenteral nutrition. The patient has questioned the need for insertion of a central venous catheter, expressing a preference for a "normal IV." The nurse should know that peripheral administration of high-concentration PN formulas is contraindicated because of the risk for what complication? A) Chemical phlebitis B) Hyperglycemia C) Dumping syndrome D) Line sepsis

B) "Avoid taking the drug on a long-term basis." Laxatives should not be taken on an ongoing basis in order to reduce the risk of dependence. Fluid should be increased, not limited, and there is no need to take each dose with a multivitamin. Senna does not need to be taken on an empty stomach.

15. A nurse is caring for a patient with constipation whose primary care provider has recommended senna (Senokot) for the management of this condition. The nurse should provide which of the following education points? A) "Limit your fluid intake temporarily so you don't get diarrhea." B) "Avoid taking the drug on a long-term basis." C) "Make sure to take a multivitamin with each dose." D) "Take this on an empty stomach to ensure maximum effect."

C) Monitor the patient closely for further signs of dumping syndrome. The patient's symptoms are characteristic of dumping syndrome, which results in a sensation of fullness, weakness, faintness, dizziness, palpitations, diaphoresis, cramping pains, and diarrhea. Aspiration is a less likely cause for the patient's symptoms. Supine positioning will likely exacerbate the symptoms and insertion of an NG tube is contraindicated due to the nature of the patient's surgery.

15. A patient who underwent gastric banding 3 days ago is having her diet progressed on a daily basis. Following her latest meal, the patient complains of dizziness and palpitations. Inspection reveals that the patient is diaphoretic. What is the nurse's best action? A) Insert a nasogastric tube promptly. B) Reposition the patient supine. C) Monitor the patient closely for further signs of dumping syndrome. D) Assess the patient for signs and symptoms of aspiration.

C) "I flush my tube with water before and after each of my medications." Frequent flushing is needed to prevent occlusion, and should not just be limited to times of medication administration. Alcohol will irritate skin surrounding the insertion site and activity should be maintained as much as possible.

16. The management of the patient's gastrostomy is an assessment priority for the home care nurse. What statement would indicate that the patient is managing the tube correctly? A) "I clean my stoma twice a day with alcohol." B) "The only time I flush my tube is when I'm putting in medications." C) "I flush my tube with water before and after each of my medications." D) "I try to stay still most of the time to avoid dislodging my tube."

B) Foul-smelling diarrhea that contains fat The hallmarks of malabsorption syndrome from any cause are diarrhea or frequent, loose, bulky, foul-smelling stools that have increased fat content and are often grayish (steatorrhea). Constipation and bloody bowel movements are not suggestive of malabsorption syndromes. Fever and a tender, rigid abdomen are associated with peritonitis.

16. The nurse is caring for a patient who is undergoing diagnostic testing for suspected malabsorption. When taking this patient's health history and performing the physical assessment, the nurse should recognize what finding as most consistent with this diagnosis? A) Recurrent constipation coupled with weight loss B) Foul-smelling diarrhea that contains fat C) Fever accompanied by a rigid, tender abdomen D) Bloody bowel movements accompanied by fecal incontinence

B) Medulla oblongata Swallowing is a voluntary act that is regulated by a swallowing center in the medulla oblongata of the central nervous system. Swallowing is not regulated by the temporal lobe, cerebellum, or pons.

17. A nurse in a stroke rehabilitation facility recognizes that the brain regulates swallowing. Damage to what area of the brain will most affect the patient's ability to swallow? A) Temporal lobe B) Medulla oblongata C) Cerebellum D) Pons

A) Acyclovir (Zovirax) Acyclovir (Zovirax) is often given to patients with viral anorectal infections. Doxycycline (Vibramycin) and penicillin (penicillin G) are drugs of choice for bacterial infections. Metronidazole (Flagyl) is used for other infections with a bacterial etiology; it is ineffective against viruses.

17. A nurse is caring for a patient admitted with symptoms of an anorectal infection; cultures indicate that the patient has a viral infection. The nurse should anticipate the administration of what drug? A) Acyclovir (Zovirax) B) Doxycycline (Vibramycin) C) Penicillin (penicillin D) Metronidazole (Flagyl)

C) Diarrhea and feelings of fullness Following a Billroth I, the patient may have problems with feelings of fullness, dumping syndrome, and diarrhea. Hunger and thirst, constipation, and gastric reflux are not adverse effects associated with this procedure.

17. A patient is receiving education about his upcoming Billroth I procedure (gastroduodenostomy). This patient should be informed that he may experience which of the following adverse effects associated with this procedure? A) Persistent feelings of hunger and thirst B) Constipation or bowel incontinence C) Diarrhea and feelings of fullness D) Gastric reflux and belching

D) Use a combination of at least two accepted methods for confirming placement. There are a variety of methods to check tube placement. The safest way to confirm placement is to utilize a combination of assessment methods.

18. A nurse is creating a care plan for a patient with a nasogastric tube. How should the nurse direct other members of the care team to check correct placement of the tube? A) Auscultate the patient's abdomen after injecting air through the tube. B) Assess the color and pH of aspirate. C) Locate the marking made after the initial x-ray confirming placement. D) Use a combination of at least two accepted methods for confirming placement.

B) Adequate understanding of required lifestyle changes Patients seeking bariatric surgery should be free of serious mental disorders and motivated to comply with lifestyle changes related to eating patterns, dietary choices, and elimination. While assessment of knowledge about causes of obesity and its associated risks as well as insight into the reasons why previous diets have been ineffective are included in the client's plan of care, these do not predict positive client outcomes following bariatric surgery. Most obese patients have an impaired body image and alteration in self-esteem. An obese patient with a positive body image would be unlikely to seek this surgery unless he or she was experiencing significant comorbidities.

19. A patient comes to the bariatric clinic to obtain information about bariatric surgery. The nurse assesses the obese patient knowing that in addition to meeting the criterion of morbid obesity, a candidate for bariatric surgery must also demonstrate what? A) Knowledge of the causes of obesity and its associated risks B) Adequate understanding of required lifestyle changes C) Positive body image and high self-esteem D) Insight into why past weight loss efforts failed

D) The test is noninvasive. Capsule endoscopy allows the noninvasive visualization of the mucosa throughout the entire small intestine. Bowel preparation is necessary and biopsies cannot be collected. This procedure allows visualization of the entire GI tract, but not the peritoneal cavity.

19. A patient with a recent history of intermittent bleeding is undergoing capsule endoscopy to determine the source of the bleeding. When explaining this diagnostic test to the patient, what advantage should the nurse describe? A) The test allows visualization of the entire peritoneal cavity. B) The test allows for painless biopsy collection. C) The test does not require fasting. D) The test is noninvasive.

D) Glucagon Glucagon is administered prior to removal of a foreign body because it relaxes the smooth muscle of the esophagus, facilitating insertion of the endoscope. Haloperidol is an antipsychotic drug and is not indicated. Prostigmine is prescribed for patients with myastheniagravis. It increases muscular contraction, an effect opposite that which is desired to facilitate removal of the foreign body. Epinephrine is indicated in asthma attack and bronchospasm.

19. An emergency department nurse is admitting a 3-year-old brought in after swallowing a piece from a wooden puzzle. The nurse should anticipate the administration of what medication in order to relax the esophagus to facilitate removal of the foreign body? A) Haloperidol B) Prostigmine C) Epinephrine D) Glucagon

B) Insertion of an NG tube for decompression In treating the patient with gastric outlet obstruction, the first consideration is to insert an NG tube to decompress the stomach. This is a priority over fluid or medication administration.

26. A patient has been admitted to the hospital after diagnostic imaging revealed the presence of a gastric outlet obstruction (GOO). What is the nurse's priority intervention? A) Administration of antiemetics B) Insertion of an NG tube for decompression C) Infusion of hypotonic IV solution D) Administration of proton pump inhibitors as ordered

C) Leave the tube in its present position. The patient's aspirate is from the gastric area when the nurse observes that the color of the aspirate is green. Further confirmation of placement is necessary, but there is likely no need for repositioning. Pleural secretions are pale yellow.

19. The nurse is assessing placement of a nasogastric tube that the patient has had in place for 2 days. The tube is draining green aspirate. What is the nurse's most appropriate action? A) Inform the physician that the tube may be in the patient's pleural space. B) Withdraw the tube 2 to 4 cm. C) Leave the tube in its present position. D) Advance the tube up to 8 cm.

A) Watery with blood and mucus The predominant symptoms of ulcerative colitis are diarrhea and abdominal pain. Stools may be bloody and contain mucus. Stools are not hard, dry, tarry, black or fatty in patients who have ulcerative colitis.

2. A nurse is preparing to provide care for a patient whose exacerbation of ulcerative colitis has required hospital admission. During an exacerbation of this health problem, the nurse would anticipate that the patient's stools will have what characteristics? A) Watery with blood and mucus B) Hard and black or tarry C) Dry and streaked with blood D) Loose with visible fatty streaks

A) Pepsin The enzyme that initiates the digestion of protein is pepsin. Intrinsic factor combines with vitamin B12 for absorption by the ileum. Lipase aids in the digestion of fats and amylase aids in the digestion of starch.

2. A nurse is promoting increased protein intake to enhance a patient's wound healing. The nurse knows that enzymes are essential in the digestion of nutrients such as protein. What is the enzyme that initiates the digestion of protein? A) Pepsin B) Intrinsic factor C) Lipase D) Amylase

A) Does your pain resolve when you have something to eat? Pain relief after eating is associated with duodenal ulcers. The pain of peptic ulcers is generally unrelated to activity or bowel function and may or may not respond to analgesics.

2. A patient comes to the clinic complaining of pain in the epigastric region. What assessment question during the health interview would most help the nurse determine if the patient has a peptic ulcer? A) Does your pain resolve when you have something to eat? B) Do over-the-counter pain medications help your pain? C) Does your pain get worse if you get up and do some exercise? D) Do you find that your pain is worse when you need to have a bowel movement?

B) Keep the vent lumen above the patient's waist. The blue vent lumen should be kept above the patient's waist to prevent reflux of gastric contents through it; otherwise it acts as a siphon. A one-way anti-reflux valve seated in the blue pigtail can prevent the reflux of gastric contents out the vent lumen. To prevent reflux, the nurse does not prime the tubing, maintain the patient in a high Fowler's position, or have the patient pin the tube to the thigh.

2. A patient is concerned about leakage of gastric contents out of the gastric sump tube the nurse has just inserted. What would the nurse do to prevent reflux gastric contents from coming through the blue vent of a gastric sump tube? A) Prime the tubing with 20 mL of normal saline. B) Keep the vent lumen above the patient's waist. C) Maintain the patient in a high Fowler's position. D) Have the patient pin the tube to the thigh.

A) Palpate the patient's parotid glands to detect swelling and tenderness. Older adults and debilitated patients of any age who are dehydrated or taking medications that reduce saliva production are at risk for parotitis. Symptoms include fever and tenderness, as well as swelling of the parotid glands. Pain radiates to the ear. Pain associated with malocclusion of the temporomandibular joint may also radiate to the ears; however, a temperature elevation would not be associated with malocclusion. The 12th cranial nerve is not associated with the auditory system. Bleeding and hyperpigmented gums may be caused by pyorrhea or gingivitis. These conditions do not cause earache; fever would not be present unless the teeth were abscessed.

2. An elderly patient comes into the emergency department complaining of an earache. The patient and has an oral temperature of 100.2ºF and otoscopic assessment of the ear reveals a pearly gray tympanic membrane with no evidence of discharge or inflammation. Which action should the triage nurse take next? A) Palpate the patient's parotid glands to detect swelling and tenderness. B) Assess the temporomandibular joint for evidence of a malocclusion. C) Test the integrity of cranial nerve XII by asking the patient to protrude the tongue. D) Inspect the patient's gums for bleeding and hyperpigmentation.

B) Report signs and symptoms of obstruction to the physician. It is important to report nausea and abdominal distention, which may indicate intestinal obstruction. This requires prompt medical intervention. Referral to the WOC nurse is not an appropriate short-term response, since medical treatment is necessary. Physical mobility will not normally resolve an obstruction. There is no need to collect a culture from the stoma, because infection is unrelated to this problem.

20. A nurse caring for a patient with a newly created ileostomy assesses the patient and notes that the patient has had not ostomy output for the past 12 hours. The patient also complains of worsening nausea. What is the nurse's priority action? A) Facilitate a referral to the wound-ostomy-continence (WOC) nurse. B) Report signs and symptoms of obstruction to the physician. C) Encourage the patient to mobilize in order to enhance motility. D) Contact the physician and obtain a swab of the stoma for culture.

B) Use warm saline to rinse the mouth as needed The patient should be assessed for bleeding after the tooth is extracted. The mouth can be rinsed with warm saline to keep the area clean. A firm toothbrush or toothpick could injure the tissues around the extracted area. Alcohol would injure tissues that are healing.

20. A nurse in an oral surgery practice is working with a patient scheduled for removal of an abscessed tooth. When providing discharge education, the nurse should recommend which of the following actions? A) Rinse the mouth with alcohol before bedtime for the next 7 days. B) Use warm saline to rinse the mouth as needed. C) Brush around the area with a firm toothbrush to prevent infection. D) Use a toothpick to dislodge any debris that gets lodged in the socket.

B) Lipase C) Amylase D) Trypsin Digestive enzymes secreted by the pancreas include trypsin, which aids in digesting protein; amylase, which aids in digesting starch; and lipase, which aids in digesting fats. Pepsin is secreted by the stomach and ptyalin is secreted in the saliva.

20. A nurse is caring for a patient admitted with a suspected malabsorption disorder. The nurse knows that one of the accessory organs of the digestive system is the pancreas. What digestive enzymes does the pancreas secrete? Select all that apply. A) Pepsin B) Lipase C) Amylase D) Trypsin E) Ptyalin

A) Confirm placement of the tube prior to each medication administration. Each time liquids or medications are administered, and once a shift for continuous feedings, the tube must be checked to ensure that it remains properly placed. If the NG tube is used for decompression, it is attached to intermittent low suction. During the placement of a nasogastric tube the patient should be positioned in a Fowler's position. Oral fluid administration is contraindicated by the patient's dysphagia.

20. A patient's new onset of dysphagia has required insertion of an NG tube for feeding; the nurse has modified the patient's care plan accordingly. What intervention should the nurse include in the patient's plan of care? A) Confirm placement of the tube prior to each medication administration. B) Have the patient sip cool water to stimulate saliva production. C) Keep the patient in a low Fowler's position when at rest. D) Connect the tube to continuous wall suction when not in use.

C) Facilitate the patient's contact with a support group. Support groups can be highly beneficial in relieving preoperative and postoperative anxiety and in promoting healthy coping. This is preferable to antianxiety medications. Downplaying the risks of surgery or focusing solely on the benefits is a simplistic and patronizing approach

21. A nurse is providing anticipator guidance to a patient who is preparing for bariatric surgery. The nurse learns that the patient is anxious about numerous aspects of the surgery. What intervention is most appropriate to alleviate the patient's anxiety? A) Emphasize the fact that bariatric surgery has a low risk of complications. B) Encourage the patient to focus on the benefits of the surgery. C) Facilitate the patient's contact with a support group. D) Obtain an order for a PRN benzodiazepine.

C) Insert the catheter approximately 5 cm into the pouch. To empty a Kock pouch, the catheter is gently inserted approximately 5 cm to the point of the valve or nipple. The length of time between drainage periods is gradually increased until the reservoir needs to be drained only every 4 to 6 hours and irrigated once each day. It is not appropriate to wait until the pouch is full, and this would not be visible. The contents of the pouch are not aspirated.

21. A nurse is working with a patient who is learning to care for a continent ileostomy (Kock pouch). Following the initial period of healing, the nurse is teaching the patient how to independently empty the ileostomy. The nurse should teach the patient to do which of the following actions? A) Aim to eventually empty the pouch every 90 minutes. B) Avoid emptying the pouch until it is visibly full. C) Insert the catheter approximately 5 cm into the pouch. D) Aspirate the contents of the pouch using a 60 mL piston syringe.

D) Orogastric tube An orogastric tube is a large-bore tube inserted through the mouth with a wide outlet for removal of gastric contents; it is used primarily in the emergency department or an intensive care setting. Nasogastric, Levin, and gastric sump tubes are not used for this specific purpose.

21. A patient has been brought to the emergency department by EMS after telling a family member that he deliberately took an overdose of NSAIDs a few minutes earlier. If lavage is ordered, the nurse should prepare to assist with the insertion of what type of tube? A) Nasogastric tube B) Levin tube C) Gastric sump D) Orogastric tube

D) Metastases are common and respond poorly to treatment. Deaths from malignancies of the head and neck are primarily attributable to local-regional metastasis to the cervical lymph nodes in the neck. This often occurs by way of the lymphatics before the primary lesion has been treated. This local-regional metastasis is not amenable to surgical resection and responds poorly to chemotherapy and radiation therapy. This high mortality rate is not related to surgical complications, late diagnosis, or the development of brain tumors.

21. A patient has been diagnosed with a malignancy of the oral cavity and is undergoing oncologic treatment. The oncologic nurse is aware that the prognosis for recovery from head and neck cancers is often poor because of what characteristic of these malignancies? A) Radiation therapy often results in secondary brain tumors. B) Surgical complications are exceedingly common. C) Diagnosis rarely occurs until the cancer is endstage. D) Metastases are common and respond poorly to treatment.

C) Secretion of mucus D) Absorption of nutrients E) Movement of nutrients into the bloodstream The small intestine folds back and forth on itself, providing approximately 7000 cm2(70 m2) of surface area for secretion and absorption, the process by which nutrients enter the bloodstream through the intestinal walls. Water reabsorption primarily takes place in the large bowel. HCl is secreted by the stomach.

21. The nurse is caring for a patient with a duodenal ulcer and is relating the patient's symptoms to the physiologic functions of the small intestine. What do these functions include? Select all that apply. A) Secretion of hydrochloric acid (HCl) B) Reabsorption of water C) Secretion of mucus D) Absorption of nutrients E) Movement of nutrients into the bloodstream

D) Decreased mucus secretion Older adults tend to secrete less mucus than younger adults. Gastric motility slows with age and gastric pH rises due to decreased secretion of gastric acids. Older adults tend to have a blunted gag reflex compared to younger adults.

22. A nurse is performing an abdominal assessment of an older adult patient. When collecting and analyzing data, the nurse should be cognizant of what age-related change in gastrointestinal structure and function? A) Increased gastric motility B) Decreased gastric pH C) Increased gag reflex D) Decreased mucus secretion

A) Patient will accurately identify foods that trigger symptoms. A major focus of nursing care for the patient with IBS is to identify factors that exacerbate symptoms. Surgery is not used to treat this health problem and infection control is not a concern that is specific to this diagnosis. Establishing causation likely is more important to the patient than managing physical activity.

22. A nurse is providing care for a patient who has a diagnosis of irritable bowel syndrome (IBS). When planning this patient's care, the nurse should collaborate with the patient and prioritize what goal? A) Patient will accurately identify foods that trigger symptoms. B) Patient will demonstrate appropriate care of his ileostomy. C) Patient will demonstrate appropriate use of standard infection control precautions. D) Patient will adhere to recommended guidelines for mobility and activity.

D) Maintaining fluid and electrolyte balance All of the listed focuses of care are important for the patient with a small bowel obstruction. However, the patient's risk of fluid and electrolyte imbalances is an immediate threat to safety, and is a priority in nursing assessment and interventions.

34. A patient has been diagnosed with a small bowel obstruction and has been admitted to the medical unit. The nurse's care should prioritize which of the following outcomes? A) Preventing infection B) Maintaining skin and tissue integrity C) Preventing nausea and vomiting D) Maintaining fluid and electrolyte balance

D) Providing the patient with physical and emotional support For acute gastritis, the nurse provides physical and emotional support and helps the patient manage the symptoms, which may include nausea, vomiting, heartburn, and fatigue. The scenario describes a newly diagnosed patient; teaching about the etiology of the disease, lifestyle modifications, or various treatment options would be best provided at a later time.

22. A patient has just been diagnosed with acute gastritis after presenting in distress to the emergency department with abdominal symptoms. What would be the nursing care most needed by the patient at this time? A) Teaching the patient about necessary nutritional modification B) Helping the patient weigh treatment options C) Teaching the patient about the etiology of gastritis D) Providing the patient with physical and emotional support

C) Placing the patient in Fowler's position After the endotracheal tube or airway has been removed and the effects of the anesthesia have worn off, the patient may be placed in Fowler's position to facilitate breathing and promote comfort. Lateral positioning does not facilitate oxygenation or comfort. Medications do not facilitate spontaneous breathing.

22. A patient has undergone surgery for oral cancer and has just been extubated in postanesthetic recovery. What nursing action best promotes comfort and facilitates spontaneous breathing for this patient? A) Placing the patient in a left lateral position B) Administering opioids as ordered C) Placing the patient in Fowler's position D) Teaching the patient to use the patient-controlled analgesia (PCA) system

D) Report this finding to the patient's primary care provider If the tube does not come out easily, force should not be used, and the problem should be reported to the primary provider. Enzymes are used to resolve obstructions, not to aid removal. For safety reasons, hot water is never instilled into a tube. Twisting could cause damage to the mucosa.

23. A nurse has obtained an order to remove a patient's NG tube and has prepared the patient accordingly. After flushing the tube and removing the nasal tape, the nurse attempts removal but is met with resistance. Because the nurse is unable to overcome this resistance, what is the most appropriate action? A) Gently twist the tube before pulling. B) Instill a digestive enzyme solution and reattempt removal in 10 to 15 minutes. C) Flush the tube with hot tap water and reattempt removal. D) Report this finding to the patient's primary care provider.

D) Imbalanced Nutrition: Less Than Body Requirements Because digestion normally begins in the mouth, adequate nutrition is related to good dental health and the general condition of the mouth. Any discomfort or adverse condition in the oral cavity can affect a person's nutritional status. Dental caries do not typically affect the patient's tissue perfusion or skin integrity. Aspiration is not a likely consequence of dental caries.

23. A nurse is performing health education with a patient who has a history of frequent, serious dental caries. When planning educational interventions, the nurse should identify a risk for what nursing diagnosis? A) Ineffective Tissue Perfusion B) Impaired Skin Integrity C) Aspiration D) Imbalanced Nutrition: Less Than Body Requirements

D) Frequent screening for osteoporosis Persons with lactose intolerance often experience hypocalcemia and a consequent risk of osteoporosis related to malabsorption of calcium. Lactose intolerance does not create an increased need for screening for colorectal cancer, immunizations, or blood pressure monitoring.

23. A patient has been experiencing disconcerting GI symptoms that have been worsening in severity. Following medical assessment, the patient has been diagnosed with lactose intolerance. The nurse should recognize an increased need for what form of health promotion? A) Annual screening colonoscopies B) Adherence to recommended immunization schedules C) Regular blood pressure monitoring D) Frequent screening for osteoporosis

A) Splenic vein B) Inferior mesenteric vein C) Gastric vein This portal venous system is composed of five large veins: the superior mesenteric, inferior mesenteric, gastric, splenic, and cystic veins, which eventually form the vena portae that enters the liver. The inferior vena cava is not part of the portal system. The saphenous vein is located in the leg.

23. The nurse educator is reviewing the blood supply of the GI tract with a group of medical nurses. The nurse is explaining the fact that the veins that return blood from the digestive organs and the spleen form the portal venous system. What large veins will the nurse list when describing this system? Select all that apply. A) Splenic vein B) Inferior mesenteric vein C) Gastric vein D) Inferior vena cava E) Saphenous vein

B) Risk for Impaired Skin Integrity Related to the Presence of NG Tube NG tubes can easily damage the delicate mucosa of the nose, sinuses, and upper airway. An NG tube does not preclude verbal communication. This patient's NG tube is in place for decompression, so complications of enteral feeding do not apply.

24. A nurse is writing a care plan for a patient with a nasogastric tube in place for gastric decompression. What risk nursing diagnosis is the most appropriate component of the care plan? A) Risk for Excess Fluid Volume Related to Enteral Feedings B) Risk for Impaired Skin Integrity Related to the Presence of NG Tube C) Risk for Unstable Blood Glucose Related to Enteral Feedings D) Risk for Impaired Verbal Communication Related to Presence of NG Tube

C) Promotion of adequate nutrition The patient who has had rigid fixation should be instructed not to chew food in the first 1 to 4 weeks after surgery. A liquid diet is recommended, and dietary counseling should be obtained to ensure optimal caloric and protein intake. The nature of this surgery threatens the patient's nutritional status; this physiologic need would likely supersede the resumption of ADLs. Pain should be under control prior to discharge and communication is not precluded by this surgery.

24. A patient has undergone rigid fixation for the correction of a mandibular fracture suffered in a fight. What area of care should the nurse prioritize when planning this patient's discharge education? A) Resumption of activities of daily living B) Pain control C) Promotion of adequate nutrition D) Strategies for promoting communication

C) Intermittent pain and bloody stool When the patient is symptomatic from a tumor of the small intestine, benign tumors often present with intermittent pain. The next most common presentation is occult bleeding. The other listed signs and symptoms are not normally associated with the presentation of small intestinal tumors.

24. A patient is undergoing diagnostic testing for a tumor of the small intestine. What are the most likely symptoms that prompted the patient to first seek care? A) Hematemesis and persistent sensation of fullness B) Abdominal bloating and recurrent constipation C) Intermittent pain and bloody stool D) Unexplained bowel incontinence and fatty stools

C) Toilet the patient on a frequent, scheduled basis. Because the patient's fecal incontinence is most likely attributable to cognitive decline, frequent toileting is an appropriate intervention. Loperamide is unnecessary in the absence of diarrhea. Specific foods are not likely to be a cause of, or solution to, this patient's health problem.

24. An older adult has a diagnosis of Alzheimer's disease and has recently been experiencing fecal incontinence. However, the nurse has observed no recent change in the character of the patient's stools. What is the nurse's most appropriate intervention? A) Keep a food diary to determine the foods that exacerbate the patient's symptoms. B) Provide the patient with a bland, low-residue diet. C) Toilet the patient on a frequent, scheduled basis. D) Liaise with the primary care provider to obtain an order for loperamide.

C) The absorption into the bloodstream of nutrient molecules produced by digestion Primary functions of the GI tract include the breakdown of food particles into molecular form for digestion; the absorption into the bloodstream of small nutrient molecules produced by digestion; and the elimination of undigested unabsorbed food stuffs and other waste products. Nutrients must be broken down into molecular form, not cell form. Fluid, electrolyte, and acid-base balance are primarily under the control of the kidneys.

24. The physiology instructor is discussing the GI system with the pre-nursing class. What should the instructor describe as a major function of the GI tract? A) The breakdown of food particles into cell form for digestion B) The maintenance of fluid and acid-base balance C) The absorption into the bloodstream of nutrient molecules produced by digestion D) The control of absorption and elimination of electrolytes

C) "You'll need to have enemas the day before the test." Preparation of the patient includes emptying and cleansing the lower bowel. This often necessitates a low-residue diet 1 to 2 days before the test; a clear liquid diet and a laxative the evening before; NPO after midnight; and cleansing enemas until returns are clear the following morning.

25. A nurse is providing preprocedure education for a patient who will undergo a lower GI tract study the following week. What should the nurse teach the patient about bowel preparation? A) "You'll need to fast for at least 18 hours prior to your test." B) "Starting today, take over-the-counter stool softeners twice daily." C) "You'll need to have enemas the day before the test." D) "For 24 hours before the test, insert a glycerin suppository every 4 hours."

D) Make appropriate referrals to services that provide psychosocial support.. The nurse should enlist the services of clergy, psychiatric clinical nurse specialists, psychologists, social workers, and psychiatrists, if needed. This is preferable to delegating care, since the patient has become angry with other care providers as well. It is impractical and inappropriate to expect the primary care provider to act as a liaison. It would be inappropriate and unsafe to simply limit contact with the patient.

25. A patient is recovering in the hospital following gastrectomy. The nurse notes that the patient has become increasingly difficult to engage and has had several angry outbursts at various staff members in recent days. The nurse's attempts at therapeutic dialogue have been rebuffed. What is the nurse's most appropriate action? A) Ask the patient's primary care provider to liaise between the nurse and the patient. B) Delegate care of the patient to a colleague. C) Limit contact with the patient in order to provide privacy. D) Make appropriate referrals to services that provide psychosocial support.

B) Entry of large amounts of water into the small intestine because of osmotic pressure When a concentrated solution of high osmolality entering the intestines is taken in quickly or in large amounts, water moves rapidly into the intestinal lumen from fluid surrounding the organs and the vascular compartment. This results in dumping syndrome. Dumping syndrome is not the result of changes in HCl or gastrin levels. It is not caused by an acid-base imbalance or direct irritation of the GI mucosa.

25. A patient's enteral feedings have been determined to be too concentrated based on the patient's development of dumping syndrome. What physiologic phenomenon caused this patient's complication of enteral feeding? A) Increased gastric secretion of HCl and gastrin because of high osmolality of feeds B) Entry of large amounts of water into the small intestine because of osmotic pressure C) Mucosal irritation of the stomach and small intestine by the high concentration of the feed D) Acid-base imbalance resulting from the high volume of solutes in the feed

B) Increased fiber intake D) Reduced fat intake Patients whose diverticular disease does not warrant hospital treatment often benefit from a high-fiber, low-fat diet. Neither enemas nor anticholinergics are indicated, and fluid intake is encouraged.

25. An adult patient has been diagnosed with diverticular disease after ongoing challenges with constipation. The patient will be treated on an outpatient basis. What components of treatment should the nurse anticipate? Select all that apply. A) Anticholinergic medications B) Increased fiber intake C) Enemas on alternating days D) Reduced fat intake E) Fluid reduction

D) Report this finding promptly to the physician and remain with the patient. In the immediate postoperative period, the nurse assesses for stridor (coarse, high-pitched sound on inspiration) by listening frequently over the trachea with a stethoscope. This finding must be reported immediately because it indicates obstruction of the airway. The patient's current status does not warrant activation of the emergency response system, and encouraging deep breathing and repositioning the patient are inadequate responses.

26. A nurse is caring for a patient who is postoperative day 1 following neck dissection surgery. The nurse is performing an assessment of the patient and notes the presence of high-pitched adventitious sounds over the patient's trachea on auscultation. The patient's oxygen saturation is 90% by pulse oximetry with a respiratory rate of 31 breaths per minute. What is the nurse's most appropriate action? A) Encourage the patient to perform deep breathing and coughing exercises hourly. B) Reposition the patient into a prone or semi-Fowler's position and apply supplementary oxygen by nasal cannula. C) Activate the emergency response system. D) Report this finding promptly to the physician and remain with the patient.

B) Clotted or displaced catheter C) Pneumothorax D) Hyperglycemia E) Line sepsis Common complications of PN include a clotted or displaced catheter, pneumothorax, hyperglycemia, and infection from the venous access device (line sepsis). Dumping syndrome applies to enteral nutrition, not PN.

26. A nurse is creating a care plan for a patient who is receiving parenteral nutrition. The patient's care plan should include nursing actions relevant to what potential complications? Select all that apply. A) Dumping syndrome B) Clotted or displaced catheter C) Pneumothorax D) Hyperglycemia E) Line sepsis

A) "Hydrochloric acid is secreted by glands in the stomach in response to the actual or anticipated presence of food." The stomach, which stores and mixes food with secretions, secretes a highly acidic fluid in response to the presence or anticipated ingestion of food. The stomach does not turn food directly into acid and the esophagus is not highly alkaline. Pancreatic enzymes are not synthesized in a highly acidic environment.

26. A patient presents at the walk-in clinic complaining of recurrent sharp stomach pain that is relieved by eating. The nurse suspects that the patient may have an ulcer. How would the nurse explain the formation and role of acid in the stomach to the patient? A) "Hydrochloric acid is secreted by glands in the stomach in response to the actual or anticipated presence of food." B) "As digestion occurs in the stomach, the stomach combines free hydrogen ions from the food to form acid." C) "The body requires an acidic environment in order to synthesize pancreatic digestive enzymes; the stomach provides this environment." D) "The acidic environment in the stomach exists to buffer the highly alkaline environment in the esophagus."

C) An absence of blood in stool Bloody stool is far more common in cases of UC than in Crohn's. Rectal involvement is nearly 100% in cases of UC (versus 20% in Crohn's) and patients with UC typically experience severe diarrhea. UC is also characterized by a pattern of remissions and exacerbations, while Crohn's often has a more prolonged and variable course.

26. A patient's health history is suggestive of inflammatory bowel disease. Which of the following would suggest Crohn's disease, rather that ulcerative colitis, as the cause of the patient's signs and symptoms? A) A pattern of distinct exacerbations and remissions B) Severe diarrhea C) An absence of blood in stool D) Involvement of the rectal mucosa

A) To remove gas from the stomach C) To remove toxins from the stomach E) To diagnose GI motility disorders GI intubation may be performed to decompress the stomach and remove gas and fluid, lavage (flush with water or other fluids) the stomach and remove ingested toxins or other harmful materials, diagnose disorders of GI motility and other disorders, administer medications and feedings, compress a bleeding site, and aspirate gastric contents for analysis. GI intubation is not used for opening sphincters that are not functional or for administering clotting factors.

27. A nurse is caring for a patient who has a gastrointestinal tube in place. Which of the following are indications for gastrointestinal intubation? Select all that apply. A) To remove gas from the stomach B) To administer clotting factors to treat a GI bleed C) To remove toxins from the stomach D) To open sphincters that are closed E) To diagnose GI motility disorders

B) Avoiding chewing food for the specified number of weeks after surgery The patient who has had rigid fixation should be instructed not to chew food in the first 1 to 4 weeks after surgery. A liquid diet is recommended, and dietary counseling should be obtained to ensure optimal caloric and protein intake. Increased calcium intake will not have an appreciable effect on healing. Enteral and parenteral nutrition are rarely necessary.

27. A nurse is caring for a patient who has just had a rigid fixation of a mandibular fracture. When planning the discharge teaching for this patient, what would the nurse be sure to include? A) Increasing calcium intake to promote bone healing B) Avoiding chewing food for the specified number of weeks after surgery C) Techniques for managing parenteral nutrition in the home setting D) Techniques for managing a gastrostomy

B) The patient has a rigid, "boardlike" abdomen that is tender. An extremely tender and rigid (boardlike) abdomen is suggestive of a perforated ulcer. None of the other listed signs and symptoms is suggestive of a perforated ulcer.

27. A patient with a history of peptic ulcer disease has presented to the emergency department (ED) in distress. What assessment finding would lead the ED nurse to suspect that the patient has a perforated ulcer? A) The patient has abdominal bloating that developed rapidly. B) The patient has a rigid, "boardlike" abdomen that is tender. C) The patient is experiencing intense lower right quadrant pain. D) The patient is experiencing dizziness and confusion with no apparent hemodynamic changes.

B) Increased fluid and fiber intake Hemorrhoid symptoms and discomfort can be relieved by good personal hygiene and by avoiding excessive straining during defecation. A high-residue diet that contains fruit and bran along with an increased fluid intake may be all the treatment that is necessary to promote the passage of soft, bulky stools to prevent straining. Antibiotics, regular use of suppositories, and NSAIDs are not recommended, as they do not address the etiology of the health problem.

27. During a patient's scheduled home visit, an older adult patient has stated to the community health nurse that she has been experiencing hemorrhoids of increasing severity in recent months. The nurse should recommend which of the following? A) Regular application of an OTC antibiotic ointment B) Increased fluid and fiber intake C) Daily use of OTC glycerin suppositories D) Use of an NSAID to reduce inflammation

B) Prepare to meet the patient's psychosocial needs. CEA is a protein that is normally not detected in the blood of a healthy person; therefore, when detected it indicates that cancer is present, but not what type of cancer is present. The patient would likely be learning that he or she has cancer, so the nurse must prioritize the patient's immediate psychosocial needs, not abdominal assessment. Future screening is not a high priority in the short term.

27. Results of a patient's preliminary assessment prompted an examination of the patient's carcinoembryonic antigen (CEA) levels, which have come back positive. What is the nurse's most appropriate response to this finding? A) Perform a focused abdominal assessment. B) Prepare to meet the patient's psychosocial needs. C) Liaise with the nurse practitioner to perform an anorectal examination. D) Encourage the patient to adhere to recommended screening protocols.

C) Hemorrhoids FOBT should not be performed when there is hemorrhoidal bleeding. GERD, peptic ulcers and nausea and vomiting do not contraindicate the use of FOBT as a diagnostic tool.

28. A clinic patient has described recent dark-colored stools;the nurse recognizes the need for fecal occult blood testing (FOBT). What aspect of the patient's current health status would contraindicate FOBT? A) Gastroesophageal reflux disease (GERD) B) Peptic ulcers C) Hemorrhoids D) Recurrent nausea and vomiting

A) Colonoscopy During colonoscopy, tissue biopsies can be obtained as needed, and polyps can be removed and evaluated. This is not possible during a barium enema, ERCP, or gastroscopy.

6. A patient has come to the outpatient radiology department for diagnostic testing. Which of the following diagnostic procedures will allow the care team to evaluate and remove polyps? A) Colonoscopy B) Barium enema C) ERCP D) Upper gastrointestinal fibroscopy

C) An older adult whose medication regimen includes an anticholinergic Elderly, acutely ill, or debilitated people with decreased salivary flow from general dehydration or medications are at high risk for parotitis. Anticholinergic medications inhibit saliva production. Antibiotics, diabetes, and wounds are not risk factors for parotitis.

28. A community health nurse serves a diverse population. What individual would likely face the highest risk for parotitis? A) A patient who is receiving intravenous antibiotic therapy in the home setting B) A patient who has a chronic venous ulcer C) An older adult whose medication regimen includes an anticholinergic D) A patient with poorly controlled diabetes who receives weekly wound care

B) Engage the patient in the care of the ostomy to the extent that the patient is willing. For some patients, becoming involved in the care of the ostomy helps to normalize it and enhance familiarity. Emphasizing the benefits of the intervention is unlikely to improve the patient's body image, since the benefits are likely already known. Online research is not likely to enhance the patient's body image and some ostomies are permanent.

28. A nurse is providing care for a patient whose recent colostomy has contributed to a nursing diagnosis of Disturbed Body Image Related to Colostomy. What intervention best addresses this diagnosis? A) Encourage the patient to conduct online research into colostomies. B) Engage the patient in the care of the ostomy to the extent that the patient is willing. C) Emphasize the fact that the colostomy was needed to alleviate a much more serious health problem. D) Emphasize the fact that the colostomy is temporary measure and is not permanent.

D) An internal retention disc secures the tube against the stomach wall. A PEG tube is held in place by an internal retention disc (flange) that holds it against the stomach wall. It is not held in place by stitches or adhesives

28. A patient with dysphagia is scheduled for PEG tube insertion and asks the nurse how the tube will stay in place. What is the nurse's best response? A) Adhesive holds a flange in place against the abdominal skin. B) A stitch holds the tube in place externally. C) The tube is stitched to the abdominal skin externally and the stomach wall internally. D) An internal retention disc secures the tube against the stomach wall.

A) Peritonitis Perforation is the erosion of the ulcer through the gastric serosa into the peritoneal cavity without warning. Chemical peritonitis develops within a few hours of perforation and is followed by bacterial peritonitis. Gastritis, reflux, and pancreatitis are not acute complications of a perforated ulcer.

28. Diagnostic imaging and physical assessment have revealed that a patient with peptic ulcer disease has suffered a perforated ulcer. The nurse recognizes that emergency interventions must be performed as soon as possible in order to prevent the development of what complication? A) Peritonitis B) Gastritis C) Gastroesophageal reflux D) Acute pancreatitis

A) Acute Pain Related to Increased Peristalsis and GI Inflammation B) Activity Intolerance Related to Generalized Weakness D) Deficient Fluid Volume Related to Anorexia, Nausea, and Diarrhea Patients with diverticulitis are likely to experience pain and decreased activity levels, and are at risk of fluid volume deficit. The patient is unlikely to experience fecal incontinence and urinary function is not directly influenced.

29. A nurse is caring for a patient who has been admitted to the hospital with diverticulitis. Which of the following would be appropriate nursing diagnoses for this patient? Select all that apply. A) Acute Pain Related to Increased Peristalsis and GI Inflammation B) Activity Intolerance Related to Generalized Weakness C) Bowel Incontinence Related to Increased Intestinal Peristalsis D) Deficient Fluid Volume Related to Anorexia, Nausea, and Diarrhea E) Impaired Urinary Elimination Related to GI Pressure on the Bladder

A) Examine one's own attitudes towards obesity in general and the patient in particular. Studies suggest that health care providers, including nurses, harbor negative attitudes towards obese patients. Nurses have a responsibility to examine these attitudes and change them accordingly. This is foundational to all other areas of assessing this patient.

29. A nurse is performing the admission assessment of a patient whose high body mass index (BMI) corresponds to class III obesity. In order to ensure empathic and patient-centered care, the nurse should do which of the following? A) Examine one's own attitudes towards obesity in general and the patient in particular. B) Dialogue with the patient about the lifestyle and psychosocial factors that resulted in obesity. C) Describe one's own struggles with weight gain and weight loss to the patient. D) Elicit the patient's short-term and long-term goals for weight loss.

D) Ineffective Tissue Perfusion Grafted skin is highly vulnerable to inadequate perfusion and subsequent ischemia and necrosis. This is a priority over chronic pain, which is unlikely to be a long-term challenge. Neglect and disuse are not risks related to the graft site.

29. A nurse is providing care for a patient whose neck dissection surgery involved the use of a graft. When assessing the graft, the nurse should prioritize data related to what nursing diagnosis? A) Risk for Disuse Syndrome B) Unilateral Neglect C) Risk for Trauma D) Ineffective Tissue Perfusion

B) Wash the area around the tube with soap and water daily. Infection can be prevented by keeping the skin near the insertion site clean using soap and water. Hydrogen peroxide is not used, due to associated skin irritation. The skin around the site is not irrigated with normal saline and antibiotics are not administered to prevent site infection.

29. A patient is postoperative day 1 following gastrostomy. The nurse is planning interventions to address the nursing diagnosis of Risk for Infection Related to Presence of Wound and Tube. What intervention is most appropriate? A) Administer antibiotics via the tube as ordered. B) Wash the area around the tube with soap and water daily. C) Cleanse the skin within 2 cm of the insertion site with hydrogen peroxide once per shift. D) Irrigate the skin surrounding the insertion site with normal saline before each use.

A) The patient's BUN and creatinine levels are within reference range following the CT. Both sodium bicarbonate and Mucomyst are free radical scavengers that sequester the contrast byproducts that are destructive to renal cells. Kidney damage would be evident by increased BUN and creatinine levels. These medications are unrelated to electrolyte or fluid balance and they play no role in the results of the CT.

29. A patient will be undergoing abdominal computed tomography (CT) with contrast. The nurse has administered IV sodium bicarbonate and oral acetylcysteine (Mucomyst) before the study as ordered. What would indicate that these medications have had the desired therapeutic effect? A) The patient's BUN and creatinine levels are within reference range following the CT. B) The CT yields high-quality images. C) The patient's electrolytes are stable in the 48 hours following the CT. D) The patient's intake and output are in balance on the day after the CT.

A) "Your appendix doesn't play a major role, so you won't notice any difference after you recovery from surgery." The appendix is an appendage of the cecum (not the large intestine) that has little or no physiologic function. Its absence does not affect digestion or absorption.

3. A patient has been brought to the emergency department with abdominal pain and is subsequently diagnosed with appendicitis. The patient is scheduled for an appendectomy but questions the nurse about how his health will be affected by the absence of an appendix. How should the nurse best respond? A) Your appendix doesn't play a major role, so you won't notice any difference after you recovery from surgery. B) The surgeon will encourage you to limit your fat intake for a few weeks after the surgery, but your body will then begin to compensate. C) Your body will absorb slightly fewer nutrients from the food you eat, but you won't be aware of this. D) Your large intestine will adapt over time to the absence of your appendix.

B) Apply a skin barrier to the peristomal skin prior to applying the pouch Guidelines for changing an ileostomy appliance are as follows. Skin should be washed with soap and water, and dried. A skin barrier should be applied to the peristomal skin prior to applying the pouch. Clamps are supplied one per box and should be reused with each bag change. Topical antibiotics are not utilized, but an antifungal spray or powder may be used.

3. A patient has had an ileostomy created for the treatment of irritable bowel disease and the patient is now preparing for discharge. What should the patient be taught about changing this device in the home setting? A) Apply antibiotic ointment as ordered after cleaning the stoma. B) Apply a skin barrier to the peristomal skin prior to applying the pouch. C) Dispose of the clamp with each bag change. D) Cleanse the area surrounding the stoma with alcohol or chlorhexidine.

C) Dilute the concentration of the feeding solution. Dumping syndrome can generally be alleviated by starting with a dilute solution and then increasing the concentration of the solution over several days. Fluid replacement may be necessary but does not prevent or treat dumping syndrome. There is no need to aspirate stomach contents. Increasing the rate will exacerbate the problem.

3. A patient receiving tube feedings is experiencing diarrhea. The nurse and the physician suspect that the patient is experiencing dumping syndrome. What intervention is most appropriate? A) Stop the tube feed and aspirate stomach contents. B) Increase the hourly feed rate so it finishes earlier. C) Dilute the concentration of the feeding solution. D) Administer fluid replacement by IV.

B) An effective means of communicating with the nurse Verbal communication may be impaired by radical surgery for oral cancer. It is therefore vital to assess the patient's ability to communicate in writing before surgery. Emotional support and dietary teaching are critical aspects of the plan of care; however, the patient's ability to communicate would be essential for both. Referral to a speech therapist will be required as part of the patient's rehabilitation; however, it is not a priority at this particular time. Communication with the nurse is crucial for the delivery of safe and effective care.

3. A patient who had a hemiglossectomy earlier in the day is assessed postoperatively, revealing a patent airway, stable vital signs, and no bleeding or drainage from the operative site. The nurse notes the patient is alert. What is the patient's priority need at this time? A) Emotional support from visitors and staff B) An effective means of communicating with the nurse C) Referral to a speech therapist D) Dietary teaching focused on consistency of food and frequency of feedings

A) "This medication will reduce the amount of acid secreted in your stomach." Proton pump inhibitors like Prilosec inhibit the synthesis of stomach acid. PPIs do not increase the durability of the stomach lining, relieve pain, or stimulate tissue repair.

3. A patient with a diagnosis of peptic ulcer disease has just been prescribed omeprazole (Prilosec). How should the nurse best describe this medication's therapeutic action? A) This medication will reduce the amount of acid secreted in your stomach. B) This medication will make the lining of your stomach more resistant to damage. C) This medication will specifically address the pain that accompanies peptic ulcer disease. D) This medication will help your stomach lining to repair itself.

A) The patient may have cancer, but other GI disease must be ruled out. CA 19-9 levels are elevated in most patients with advanced pancreatic cancer, but they may also be elevated in other conditions such as colorectal, lung, and gallbladder cancers; gallstones; pancreatitis; cystic fibrosis; and liver disease. A cancer diagnosis cannot be made solely on CA 19-9 results.

30. A medical patient's CA 19-9 levels have become available and they are significantly elevated. How should the nurse best interpret this diagnostic finding? A) The patient may have cancer, but other GI disease must be ruled out. B) The patient most likely has early-stage colorectal cancer. C) The patient has a genetic predisposition to gastric cancer. D) The patient has cancer, but the site is unknown.

B) Flatus with oily discharge Side effects of orlistat include increased frequency of bowel movements, gas with oily discharge, decreased food absorption, decreased bile flow, and decreased absorption of some vitamins. This drug does not cause bowel incontinence, abdominal pain, or heat intolerance.

30. A patient has been prescribed orlistat (Xenical) for the treatment of obesity. When providing relevant health education for this patient, the nurse should ensure the patient is aware of what potential adverse effect of treatment? A) Bowel incontinence B) Flatus with oily discharge C) Abdominal pain D) Heat intolerance

A) The patient's swallowing ability If the superior laryngeal nerve is damaged, the patient may have difficulty swallowing liquids and food because of the partial lack of sensation of the glottis. Damage to this particular nerve does not inhibit speech and only affects management of secretions and airway patency indirectly.

30. A patient's neck dissection surgery resulted in damage to the patient's superior laryngeal nerve. What area of assessment should the nurse consequently prioritize? A) The patient's swallowing ability B) The patient's ability to speak C) The patient's management of secretions D) The patient's airway patency

B) The patient is agitated. Inappropriate placement may occur in patients with decreased levels of consciousness, confused mental states, poor or absent cough and gag reflexes, or agitation during insertion. A "short neck," GERD, and pneumonia are not linked to incorrect placement.

30. The nurse is preparing to insert a patient's ordered NG tube. What factor should the nurse recognize as a risk for incorrect placement? A) The patient is obese and has a "short neck." B) The patient is agitated. C) The patient has a history of gastroesophageal reflux disease (GERD). D) The patient is being treated for pneumonia.

A) Anticholinergic medications 30 minutes before a meal The nurse administers anticholinergic medications 30 minutes before a meal as prescribed to decrease intestinal motility and administers analgesics as prescribed for pain. Antiemetics, vitamin B12 injections and beta blockers do not address the signs, symptoms, or etiology of inflammatory bowel disease.

30. The nurse is providing care for a patient whose inflammatory bowel disease has necessitated hospital treatment. Which of the following would most likely be included in the patient's medication regimen? A) Anticholinergic medications 30 minutes before a meal B) Antiemetics on a PRN basis C) Vitamin B12 injections to prevent pernicious anemia D) Beta adrenergic blockers to reduce bowel motility

B) A quantitative fecal immunochemical test Quantitative fecal immunochemical tests may be more accurate than guaiac testing and useful for patients who refuse invasive testing. CT or MRI cannot detect blood in stool. Laparoscopic intestinal mucosa biopsy is not performed.

31. A patient has come to the clinic complaining of blood in his stool. A FOBT test is performed but is negative. Based on the patient's history, the physician suggests a colonoscopy, but the patient refuses, citing a strong aversion to the invasive nature of the test. What other test might the physician order to check for blood in the stool? A) A laparoscopic intestinal mucosa biopsy B) A quantitative fecal immunochemical test C) Computed tomography (CT) D) Magnetic resonance imagery (MRI)

C) "Medications can be helpful, but few people achieve and maintain their desired weight loss with medications alone." Though antiobesity drugs help some patients lose weight, their use rarely results in loss of more than 10% of total body weight. Patients are consequently unlikely to attain their desired weight through medication alone. They are not predicted to disappear from the market and results are not attributed to a placebo effect.

31. A patient who is obese has been unable to lose weight successfully using lifestyle modifications and has mentioned the possibility of using weight-loss medications. What should the nurse teach the patient about pharmacologic interventions for the treatment of obesity? A) "Weight loss drugs have many side effects, and most doctors think they'll all be off the market in a few years." B) "There used to be a lot of hope that medications would help people lose weight, but it's been shown to be mostly a placebo effect." C) "Medications can be helpful, but few people achieve and maintain their desired weight loss with medications alone." D) "Medications are rapidly become the preferred method of weight loss in people for whom diet and exercise have not worked."

C) Positioning the patient to prevent gastric reflux After recovering from the effects of anesthesia, the patient is placed in a low Fowler's position, and later in a Fowler's position, to help prevent reflux of gastric secretions. The patient is observed carefully for regurgitation and dyspnea because a common postoperative complication is aspiration pneumonia. In this period of recovery, self-suctioning is also not likely realistic or safe. Chest physiotherapy is contraindicated because of the risk of aspiration. Nutrition is prioritized, but a regular diet is contraindicated in the immediate recovery from esophageal surgery.

31. A patient who underwent surgery for esophageal cancer is admitted to the critical care unit following postanesthetic recovery. Which of the following should be included in the patient's immediate postoperative plan of care? A) Teaching the patient to self-suction B) Performing chest physiotherapy to promote oxygenation C) Positioning the patient to prevent gastric reflux D) Providing a regular diet as tolerated

C) Acknowledge the patient's reluctance and initiate discussion of the factors underlying it. If the patient is reluctant to participate in ostomy care, the nurse should attempt to dialogue about this with the patient and explore the factors that underlie it. It is presumptive to assume that the patient's behavior is motivated by fear. Assessment must precede referrals and emphasizing the patient's responsibilities may or may not motivate the patient.

31. A patient's colorectal cancer has necessitated a hemicolectomy with the creation of a colostomy. In the 4 days since the surgery, the patient has been unwilling to look at the ostomy or participate in any aspects of ostomy care. What is the nurse's most appropriate response to this observation? A) Ensure that the patient knows that he or she will be responsible for care after discharge. B) Reassure the patient that many people are fearful after the creation of an ostomy. C) Acknowledge the patient's reluctance and initiate discussion of the factors underlying it. D) Arrange for the patient to be seen by a social worker or spiritual advisor.

C) Determining the patient's ability to understand and cooperate with the procedure The major focus of the preoperative assessment is to determine the patient's ability both to understand and cooperate with the procedure. Body image, nutritional needs, and postoperative care are all important variables, but they are not the main focuses of assessment during the immediate preoperative period.

31. Prior to a patient's scheduled jejunostomy, the nurse is performing the preoperative assessment. What goal should the nurse prioritize during the preoperative assessment? A) Determining the patient's nutritional needs B) Determining that the patient fully understands the postoperative care required C) Determining the patient's ability to understand and cooperate with the procedure D) Determining the patient's ability to cope with an altered body image

C) GI diseases often produce skin changes. Abdominal lesions are of particular importance, because GI diseases often produce skin changes. Skin problems do not normally cause GI disorders. Age-related skin changes do not have a pronounced effect on the skin of the abdomen when compared to other skin surfaces. Self-harm is a less likely explanation for skin lesions on the abdomen.

32. A nurse is assessing the abdomen of a patient just admitted to the unit with a suspected GI disease. Inspection reveals several diverse lesions on the patient's abdomen. How should the nurse best interpret this assessment finding? A) Abdominal lesions are usually due to age-related skin changes. B) Integumentary diseases often cause GI disorders. C) GI diseases often produce skin changes. D) The patient needs to be assessed for self-harm.

D) Potassium level In elderly patients, it is important to monitor the patient's serum electrolyte levels closely. Diarrhea is less likely to cause an alteration in white blood cell, creatinine, and hemoglobin levels.

32. A nurse is caring for an older adult who has been experiencing severe Clostridium difficile-related diarrhea. When reviewing the patient's most recent laboratory tests, the nurse should prioritize which of the following? A) White blood cell level B) Creatinine level C) Hemoglobin level D) Potassium level

D) Antibiotics, proton pump inhibitors, and bismuth salts Currently, the most commonly used therapy for peptic ulcers is a combination of antibiotics, proton pump inhibitors, and bismuth salts that suppress or eradicate H. pylori. H2 receptor antagonists are used to treat NSAID-induced ulcers and other ulcers not associated with H. pylori infection, but they are not the drug of choice. Bicarbonate salts are not used. ZES is the Zollinger-Ellison syndrome and not a drug.

32. A patient has been diagnosed with peptic ulcer disease and the nurse is reviewing his prescribed medication regimen with him. What is currently the most commonly used drug regimen for peptic ulcers? A) Bismuth salts, antivirals, and histamine-2 (H2) antagonists B) H2 antagonists, antibiotics, and bicarbonate salts C) Bicarbonate salts, antibiotics, and ZES D) Antibiotics, proton pump inhibitors, and bismuth salts

A) Ensure that none of the patient's visitors has an infection. Leukopenia reduces defense mechanisms, increasing the risk of infections. Visitors who might transmit microorganisms are prohibited if the patient's immunologic system is depressed. Changes in diet, CSFs, and the use of chemotherapy do not resolve leukopenia.

32. A patient has received treatment for oral cancer. The combination of medications and radiotherapy has resulted in leukopenia. Which of the following is an appropriate response to this change in health status? A) Ensure that none of the patient's visitors has an infection. B) Arrange for a diet that is high in protein and low in fat. C) Administer colony stimulating factors (CSFs) as ordered. D) Prepare to administer chemotherapeutics as ordered.

C) 2 to 3 months An alternative to the PEG device is a low-profile gastrostomy device (LPGD). LPGDs may be inserted 2 to 3 months after initial gastrostomy tube placement.

32. You are caring for a patient who was admitted to have a low-profile gastrostomy device (LPGD) placed. How soon after the original gastrostomy tube placement can an LPGD be placed? A) 2 weeks B) 4 to 6 weeks C) 2 to 3 months D) 4 to 6 months

C) Document that the stoma appears healthy and well perfused. A healthy, viable stoma should be shiny and pink to bright red. This finding does not indicate that the stoma is blocked or that skin integrity is compromised.

33. A nurse is assessing a patient's stoma on postoperative day 3. The nurse notes that the stoma has a shiny appearance and a bright red color. How should the nurse best respond to this assessment finding? A) Irrigate the ostomy to clear a possible obstruction. B) Contact the primary care provider to report this finding. C) Document that the stoma appears healthy and well perfused. D) Document a nursing diagnosis of Impaired Skin Integrity.

A) Enhancement of verbal communication Verbal communication may be impaired by radical surgery for oral cancer. Addressing this impairment often requires a long-term commitment. Immune function, social support, and fluid balance are all necessary, but communication is a priority issue for patients recovering from this type of surgery.

33. A nurse is caring for a patient who has had surgery for oral cancer. When addressing the patient's long-term needs, the nurse should prioritize interventions and referrals with what goal? A) Enhancement of verbal communication B) Enhancement of immune function C) Maintenance of adequate social support D) Maintenance of fluid balance

B) Ineffective Role Performance Related to Parenteral Nutrition The limitations associated with PN can make it difficult for patients to maintain their usual roles. PN does not normally cause bowel incontinence and catheters are not associated with chronic pain or neurovascular dysfunction.

33. A nurse is caring for a patient who is receiving parenteral nutrition. When writing this patient's plan of care, which of the following nursing diagnoses should be included? A) Risk for Peripheral Neurovascular Dysfunction Related to Catheter Placement B) Ineffective Role Performance Related to Parenteral Nutrition C) Bowel Incontinence Related to Parenteral Nutrition D) Chronic Pain Related to Catheter Placement

B) Deficient Knowledge Related to Risks and Expectations of Surgery It is expected that patients seeking bariatric surgery may have challenges with body image and self-esteem related to their obesity. Anxiety is also expected when facing surgery. However, if the patient's knowledge remains deficient regarding the risks and realistic expectations for surgery, this may show that the patient is not an appropriate surgical candidate.

33. A patient who is obese is exploring bariatric surgery options and presented to a bariatric clinic for preliminary investigation. The nurse interviews the patient, analyzing and documenting the data. Which of the following nursing diagnoses may be a contraindication for bariatric surgery? A) Disturbed Body Image Related to Obesity B) Deficient Knowledge Related to Risks and Expectations of Surgery C) Anxiety Related to Surgery D) Chronic Low Self-Esteem Related to Obesity

D) "Avoid vitamin C for 72 hours before you start the test." Red meats, aspirin, nonsteroidal anti-inflammatory drugs, turnips, and horseradish should be avoided for 72 hours prior to the study, because they may cause a false-positive result. Also, ingestion of vitamin C from supplements or foods can cause a false-negative result. Acidic foods do not need to be avoided.

33. Probably the most widely used in-office or at-home occult blood test is the Hemoccult II. The patient has come to the clinic because he thinks there is blood in his stool. When you reviewed his medications, you noted he is on antihypertensive drugs and NSAIDs for early arthritic pain. You are sending the patient home with the supplies necessary to perform 2 hemoccult tests on his stool and mail the samples back to the clinic. What instruction would you give this patient? A) "Take all your medications as usual." B) "Take all your medications except the antihypertensive medications." C) "Don't eat highly acidic foods 72 hours before you start the test." D) "Avoid vitamin C for 72 hours before you start the test."

B) Change the dressing no more than weekly. The CDC (2011) recommends changing CVAD dressings not more than every 7 days unless the dressing is damp, bloody, loose, or soiled. Sterile technique (not clean technique) is used. Irrigation and antibiotic ointments are not used.

34. A nurse is aware of the high incidence of catheter-related bloodstream infections in patients receiving parenteral nutrition. What nursing action has the greatest potential to reduce catheter-related bloodstream infections? A) Use clean technique and wear a mask during dressing changes. B) Change the dressing no more than weekly. C) Apply antibiotic ointment around the site with each dressing change. D) Irrigate the insertion site with sterile water during each dressing change.

nurse is planning discharge education for a patient with trigeminal neuralgia. nurse knows to include information about factors that precipitate an attack.the nurse correct in teaching the patient to avoid

teaching the patient to avoid Washing his face

B) The patient is encouraged to express fears openly. Encouraging the patient to discuss his or her fears and anxieties is usually the best way to assess a patient's anxiety. Presenting hypothetical situations is a surreptitious and possibly inaccurate way of assessing anxiety. Observing body language is part of assessment, but it is not the complete assessment. Presenting information may alleviate anxiety for some patients, but it is not an assessment.

34. A patient has recently received a diagnosis of gastric cancer; the nurse is aware of the importance of assessing the patient's level of anxiety. Which of the following actions is most likely to accomplish this? A) The nurse gauges the patient's response to hypothetical outcomes. B) The patient is encouraged to express fears openly. C) The nurse provides detailed and accurate information about the disease. D) The nurse closely observes the patient's body language.

C) Respiratory status and airway clearance Postoperatively, the patient is assessed for complications such as altered respiratory status, wound infection, and hemorrhage. The other assessments are part of the plan of care for a patient who has had a radical neck dissection, but are not the nurse's chief priority.

34. A patient with cancer of the tongue has had a radical neck dissection. What nursing assessment would be a priority for this patient? A) Presence of acute pain and anxiety B) Tissue integrity and color of the operative site C) Respiratory status and airway clearance D) Self-esteem and body image

B) The patient can resume a normal routine immediately. Following sigmoidoscopy, patients can resume their regular activities and diet. There is no need to push fluids and neither fecal urgency nor rectal bleeding is expected.

34. A patient's sigmoidoscopy has been successfully completed and the patient is preparing to return home. Which of the following teaching points should the nurse include in the patient's discharge education? A) The patient should drink at least 2 liters of fluid in the next 12 hours. B) The patient can resume a normal routine immediately. C) The patient should expect fecal urgency for several hours. D) The patient can expect some scant rectal bleeding.

A) Stomach emptying takes place more slowly. Delayed gastric emptying occurs in older adults and may contribute to nausea. Changes to the small intestine and decreased saliva production would be less likely to contribute to nausea. Loss of esophageal sphincter function is pathologic and is not considered an age-related change.

35. A nurse is caring for an 83-year-old patient who is being assessed for recurrent and intractable nausea. What age-related change to the GI system may be a contributor to the patient's health complaint? A) Stomach emptying takes place more slowly. B) The villi and epithelium of the small intestine become thinner. C) The esophageal sphincter becomes incompetent. D) Saliva production decreases.

A) Eat small, frequent meals with high calorie and vitamin content. The nurse encourages the patient to eat small, frequent portions of nonirritating foods to decrease gastric irritation. Food supplements should be high in calories, as well as vitamins A and C and iron, to enhance tissue repair.

35. A patient has received a diagnosis of gastric cancer and is awaiting a surgical date. During the preoperative period, the patient should adopt what dietary guidelines? A) Eat small, frequent meals with high calorie and vitamin content. B) Eat frequent meals with an equal balance of fat, carbohydrates, and protein. C) Eat frequent, low-fat meals with high protein content. D) Try to maintain the pre-diagnosis pattern of eating.

B) 60 mL of milky or cloudy drainage Between 80 and 120 mL of serosanguineous secretions may drain over the first 24 hours. Milky drainage is indicative of a chyle fistula, which requires prompt treatment.

35. A patient returns to the unit after a neck dissection. The surgeon placed a Jackson Pratt drain in the wound. When assessing the wound drainage over the first 24 postoperative hours the nurse would notify the physician immediately for what? A) Presence of small blood clots in the drainage B) 60 mL of milky or cloudy drainage C) Spots of drainage on the dressings surrounding the drain D) 120 mL of serosanguinous drainage

C) Regurgitation and aspiration are less likely. Gastrostomy is preferred over NG feedings in the patient who is comatose because the gastroesophageal sphincter remains intact, making regurgitation and aspiration less likely than with NG feedings. Both tubes carry a risk for infection; this change in care is not motivated by the possibility of faster infusion or easier personal care.

35. A patient who suffered a stroke had an NG tube inserted to facilitate feeding shortly after admission. The patient has since become comatose and the patient's family asks the nurse why the physician is recommending the removal of the patient's NG tube and the insertion of a gastrostomy tube. What is the nurse's best response? A) It eliminates the risk for infection. B) Feeds can be infused at a faster rate. C) Regurgitation and aspiration are less likely. D) It allows caregivers to provide personal hygiene more easily.

D) Preparing the patient for surgical bowel resection The usual treatment for a large bowel obstruction is surgical resection to remove the obstructing lesion. Administration of laxatives or bowel stimulants are contraindicated if the bowel is obstructed. Palpation would be painful and has no therapeutic benefit.

35. A patient's large bowel obstruction has failed to resolve spontaneously and the patient's worsening condition has warranted admission to the medical unit. Which of the following aspects of nursing care is most appropriate for this patient? A) Administering bowel stimulants as ordered B) Administering bulk-forming laxatives as ordered C) Performing deep palpation as ordered to promote peristalsis D) Preparing the patient for surgical bowel resection

D) Notify the physician. The nurse must always be alert for any indicators of hemorrhagic gastritis, which include hematemesis (vomiting of blood), tachycardia, and hypotension. If these occur, the physician is notified and the patient's vital signs are monitored as the patient's condition warrants. Putting the patient in a prone position could lead to aspiration. Giving ice water is contraindicated as it would stimulate more vomiting.

36. A nurse is caring for a patient who has a diagnosis of GI bleed. During shift assessment, the nurse finds the patient to betachycardic and hypotensive, and the patient has an episode of hematemesis while the nurse is in the room. In addition to monitoring the patient's vital signs and level of conscious, what would be a priority nursing action for this patient? A) Place the patient in a prone position. B) Provide the patient with ice water to slow any GI bleeding. C) Prepare for the insertion of an NG tube. D) Notify the physician.

A) Encourage the family to bring in the patient's favored foods. Family involvement and home-cooked favorite foods may help the patient to eat. Having visitors at mealtimes may make eating more pleasant and increase the patient's appetite. The nurse should not place the complete onus for initiating meals on the patient. Oral care after meals is necessary, but does not influence appetite.

36. A nurse is caring for a patient who is postoperative from a neck dissection. What would be the most appropriate nursing action to enhance the patient's appetite? A) Encourage the family to bring in the patient's favored foods. B) Limit visitors at mealtimes so that the patient is not distracted. C) Avoid offering food unless the patient initiates. D) Provide thorough oral care immediately after the patient eats.

A) Changes in lifestyle B) Loss of eating as a social behavior D) Sleep disturbances related to frequent urination during nighttime infusions Patients must cope with the loss of eating as a social behavior and with changes in lifestyle brought on by sleep disturbances related to frequent urination during night time infusions. PN is not associated with bowel incontinence and the patient does not select or adjust the formulation of PN.

36. A patient has been discharged home on parenteral nutrition (PN). Much of the nurse's discharge education focused on coping. What must a patient on PN likely learn to cope with? Select all that apply. A) Changes in lifestyle B) Loss of eating as a social behavior C) Chronic bowel incontinence from GI changes D) Sleep disturbances related to frequent urination during nighttime infusions E) Stress of choosing the correct PN formulation

D) Psyllium hydrophilic mucilloid (Metamucil) Psyllium hydrophilic mucilloid (Metamucil) is a bulk-forming laxative that is safe for ongoing use. None of the other listed laxatives should be used on an ongoing basis because of the risk of dependence.

36. A patient has been experiencing occasional episodes of constipation and has been unable to achieve consistent relief by increasing physical activity and improving his diet. What pharmacologic intervention should the nurse recommend to the patient for ongoing use? A) Mineral oil enemas B) Bisacodyl (Dulcolax) C) Senna (Senokot) D) Psyllium hydrophilic mucilloid (Metamucil)

A) Decreased motility Norepinephrine generally decreases GI motility and secretions, but increases muscle tone of sphincters. Norepinephrine does not increase the release of enzymes.

36. A patient has been experiencing significant psychosocial stress in recent weeks. The nurse is aware of the hormonal effects of stress, including norepinephrine release. Release of this substance would have what effect on the patient's gastrointestinal function? Select all that apply. A) Decreased motility B) Increased sphincter tone C) Increased enzyme release D) Inhibition of secretions E) Increased peristalsis

D) Strategies for avoiding irritating foods and beverages Measures to help relieve pain include instructing the patient to avoid foods and beverages that may be irritating to the gastric mucosa and instructing the patient about the correct use of medications to relieve chronic gastritis. An alkaline gastric environment is neither possible nor desirable. There is no plausible need for self-suctioning. Positioning does not have a significant effect on the presence or absence of gastric healing.

37. A nurse is caring for a patient hospitalized with an exacerbation of chronic gastritis. What health promotion topic should the nurse emphasize? A) Strategies for maintaining an alkaline gastric environment B) Safe technique for self-suctioning C) Techniques for positioning correctly to promote gastric healing D) Strategies for avoiding irritating foods and beverages

A) Measure and record drainage. This drainage should be measured and recorded because it is a significant indicator of GI function. The nurse should indeed monitor the color of the output, but fluid balance is normally the priority. Frequent titration of the suction should not be necessary and feeding is contraindicated if the G tube is in place for drainage.

37. A patient has a gastrostomy tube that has been placed to drain stomach contents by low intermittent suction. What is the nurse's priority during this aspect of the patient's care? A) Measure and record drainage. B) Monitor drainage for change in color. C) Titrate the suction every hour. D) Feed the patient via the G tube as ordered.

B) Frequent abdominal auscultation After bowel surgery, it is important to frequently assess the abdomen, including bowel sounds and abdominal girth, to detect bowel obstruction. The resumption of bowel motility is a priority over each of the other listed assessments, even though each should be performed by the nurse.

37. A patient with a diagnosis of colon cancer is 2 days postoperative following bowel resection and anastomosis. The nurse has planned the patient's care in the knowledge of potential complications. What assessment should the nurse prioritize? A) Close monitoring of temperature B) Frequent abdominal auscultation C) Assessment of hemoglobin, hematocrit, and red blood cell levels D) Palpation of peripheral pulses and leg girth

D) Digestion of proteins Trypsin facilitates the digestion of proteins. It does not influence vitamin D synthesis, the digestion of fats, or peristalsis.

37. A patient with cystic fibrosis takes pancreatic enzyme replacements on a regular basis. The patient's intake of trypsin facilitates what aspect of GI function? A) Vitamin D synthesis B) Digestion of fats C) Maintenance of peristalsis D) Digestion of proteins

A) Preparing the patient to troubleshoot for problems B) Teaching the patient and family strict aseptic technique C) Teaching the patient and family how to set up the infusion An effective home care teaching program prepares the patient to store solutions, set up the infusion, flush the line with heparin, change the dressings, and troubleshoot for problems. The most common complication is sepsis. Strict aseptic technique is taught for hand hygiene, handling equipment, changing the dressing, and preparing the solution. Sterile water is never used for flushes and the access site must never be left open to air.

38. A nurse is preparing to discharge a patient home on parenteral nutrition. What should an effective home care teaching program address? Select all that apply. A) Preparing the patient to troubleshoot for problems B) Teaching the patient and family strict aseptic technique C) Teaching the patient and family how to set up the infusion D) Teaching the patient to flush the line with sterile water E) Teaching the patient when it is safe to leave the access site open to air

A patient with a cholelithiasis has been scheduled for a laparoscopic cholecystectomy. Why is laparoscopic cholecystectomy preferred by surgeons over an open procedure? A) Laparoscopic cholecystectomy poses fewer surgical risks than an open procedure. B) Laparoscopic cholecystectomy can be performed in a clinic setting, while an open procedure requires an OR. C) A laparoscopic approach allows for the removal of the entire gallbladder. D) A laparoscopic approach can be performed under conscious sedation.

A

A) "Does anyone in your family have experience at giving injections?" Patients with malabsorption of vitamin B12 need information about lifelong vitamin B12injections; the nurse may instruct a family member or caregiver how to administer the injections or make arrangements for the patient to receive the injections from a health care provider. Questions addressing sun exposure, blood type and first aid are not directly relevant.

38. A patient with gastritis required hospital treatment for an exacerbation of symptoms and receives a subsequent diagnosis of pernicious anemia due to malabsorption. When planning the patient's continuing care in the home setting, what assessment question is most relevant? A) "Does anyone in your family have experience at giving injections?" B) "Are you going to be anywhere with strong sunlight in the next few months?" C) "Are you aware of your blood type?" D) "Do any of your family members have training in first aid?"

A) Risk for infection Pilonidal cysts frequently develop into an abscess, necessitating surgical repair. These cysts do not contribute to bowel incontinence, constipation, or impaired tissue perfusion.

38. A teenage patient with a pilonidal cyst has been brought for care by her mother. The nurse who is contributing to the patient's care knows that treatment will be chosen based on what risk? A) Risk for infection B) Risk for bowel incontinence C) Risk for constipation D) Risk for impaired tissue perfusion

B) Keeping the head of the bed slightly elevated The patient with GERD is encouraged to elevate the head of the bed on 6- to 8-inch (15- to 20-cm) blocks. Frequent meals are not specifically encouraged and the patient should avoid food and fluid within 2 hours of bedtime. All carbonated beverages should be avoided.

38. Results of a patient barium swallow suggest that the patient has GERD. The nurse is planning health education to address the patient's knowledge of this new diagnosis. Which of the following should the nurse encourage? A) Eating several small meals daily rather than 3 larger meals B) Keeping the head of the bed slightly elevated C) Drinking carbonated mineral water rather than soft drinks D) Avoiding food or fluid intake after 6:00 p.m.

D) Inform the primary care provider of this finding. The nurse should inform the primary care provider of this abnormal finding in the patient's oral cavity, since it necessitates medical treatment. It would be inappropriate to try to remove skin lesions from a patient's mouth and salt water will not resolve this problem, which is likely due to candidiasis. A dietitian referral is unnecessary.

38. The nurse is caring for a patient who has a diagnosis of AIDS. Inspection of the patient's mouth reveals the new presence of white lesions on the patient's oral mucosa. What is the nurse's most appropriate response? A) Encourage the patient to gargle with salt water twice daily. B) Attempt to remove the lesions with a tongue depressor. C) Make a referral to the unit's dietitian. D) Inform the primary care provider of this finding.

B) The correct procedure for taking a sitz bath Sitz baths are usually indicated after perianal surgery. A low-residue, low-fat diet is not necessary and water is used to keep the region clean. Postoperative antibiotics are not normally prescribed.

39. A nurse at an outpatient surgery center is caring for a patient who had a hemorrhoidectomy. What discharge education topics should the nurse address with this patient? A) The appropriate use of antibiotics to prevent postoperative infection B) The correct procedure for taking a sitz bath C) The need to eat a low-residue, low-fat diet for the next 2 weeks D) The correct technique for keeping the perianal region clean without the use of water

A) Perforation into the mediastinum C) Erosion into the great vessels E) Obstruction of the esophagus In the later stages of esophageal cancer, obstruction of the esophagus is noted, with possible perforation into the mediastinum and erosion into the great vessels. Painful swallowing and the emergence of a lesion are early signs of esophageal cancer.

39. A nurse is caring for a patient in the late stages of esophageal cancer. The nurse should plan to prevent or address what characteristics of this stage of the disease? Select all that apply. A) Perforation into the mediastinum B) Development of an esophageal lesion C) Erosion into the great vessels D) Painful swallowing E) Obstruction of the esophagus

A) Specific lifestyle changes associated with each procedure B) Implications of each procedure for eating habits C) Effects of different surgeries on bowel function Different bariatric surgical procedures entail different lifestyle modifications; patients must be well informed about the specific lifestyle changes, eating habits, and bowel habits that may result from a particular procedure. Bariatric surgeries do not influence the future use of immunizations or fertility, though pregnancy should be avoided for 18 months after bariatric surgery.

39. A nurse is presenting a class at a bariatric clinic about the different types of surgical procedures offered by the clinic. When describing the implications of different types of surgeries, the nurse should address which of the following topics? Select all that apply. A) Specific lifestyle changes associated with each procedure B) Implications of each procedure for eating habits C) Effects of different surgeries on bowel function D) Effects of various bariatric surgeries on fertility E) Effects of different surgeries on safety of future immunizations

B) Risk For Impaired Skin Integrity Related to Peptic Ulcers Urea breath tests detect the presence of Helicobacter pylori, the bacteria that can live in the mucosal lining of the stomach and cause peptic ulcer disease. This test does not address fluid volume, nutritional status, or dentition.

39. A patient has been scheduled for a urea breath test in one month's time. What nursing diagnosis most likely prompted this diagnostic test? A) Impaired Dentition Related to Gingivitis B) Risk For Impaired Skin Integrity Related to Peptic Ulcers C) Imbalanced Nutrition: Less Than Body Requirements Related to Enzyme Deficiency D) Diarrhea Related to Clostridium Difficile Infection

C) Gently rotate the tube. The nurse verifies the tube's placement and gently rotates the tube once daily to prevent skin breakdown. Verifying tube placement and taping the tube to the abdomen do not prevent skin breakdown. A gastrostomy wound does not have a wet-to-dry dressing.

39. The nurse is caring for a patient who is postoperative from having a gastrostomy tube placed. What should the nurse do on a daily basis to prevent skin breakdown? A) Verify tube placement. B) Loop adhesive tape around the tube and connect it securely to the abdomen. C) Gently rotate the tube. D) Change the wet-to-dry dressing.

A patient with ongoing back pain, nausea, and abdominal bloating has been diagnosed with cholecystitis secondary to gallstones. The nurse should anticipate that the patient will undergo what intervention? A) Laparoscopic cholecystectomy B) Methyl tertiary butyl ether (MTBE) infusion C) Intracorporeal lithotripsy D) Extracorporeal shock wave therapy (ESWL)

A

D) The early symptoms of gastric cancer are usually not alarming or highly unusual. Symptoms of early gastric cancer, such as pain relieved by antacids, resemble those of benign ulcers and are seldom definitive. Symptoms are rarely a cause for alarm or for detailed diagnostic testing. Symptoms precede metastasis, however, and do not include constipation.

4. A nurse is admitting a patient diagnosed with late-stage gastric cancer. The patient's family is distraught and angry that she was not diagnosed earlier in the course of her disease. What factor contributes to the fact that gastric cancer is often detected at a later stage? A) Gastric cancer does not cause signs or symptoms until metastasis has occurred. B) Adherence to screening recommendations for gastric cancer is exceptionally low. C) Early symptoms of gastric cancer are usually attributed to constipation. D) The early symptoms of gastric cancer are usually not alarming or highly unusual.

A) Premature removal of the G tube A significant postoperative complication of a gastrostomy is premature removal of the G tube. Constipation is a less immediate threat and bowel perforation and PUD are not noted to be likely complications.

4. A nurse is admitting a patient to the postsurgical unit following a gastrostomy. When planning assessments, the nurse should be aware of what potential postoperative complication of a gastrostomy? A) Premature removal of the G tube B) Bowel perforation C) Constipation D) Development of peptic ulcer disease (PUD)

B) Contact the primary care provider promptly and report these signs of perforation. The patient's change in status is suggestive of perforation, which is a surgical emergency. Obstruction does not have this presentation involving fever and abdominal rigidity. An enema would be strongly contraindicated. An order is needed for NG insertion and repositioning is not a priority.

4. A patient admitted with acute diverticulitis has experienced a sudden increase in temperature and complains of a sudden onset of exquisite abdominal tenderness. The nurse's rapid assessment reveals that the patient's abdomen is uncharacteristically rigid on palpation. What is the nurse's best response? A) Administer a Fleet enema as ordered and remain with the patient. B) Contact the primary care provider promptly and report these signs of perforation. C) Position the patient supine and insert an NG tube. D) Page the primary care provider and report that the patient may be obstructed.

C) Hemorrhoids Lower rectal or anal bleeding is suspected if there is streaking of blood on the surface of the stool. Hemorrhoids are often a cause of anal bleeding since they occur in the rectum. Blood from an upper GI bleed would be dark rather than frank. Iron supplements make the stool dark, but not bloody and red meat consumption would not cause frank blood.

4. A patient asks the nursing assistant for a bedpan. When the patient is finished, the nursing assistant notifies the nurse that the patient has bright red streaking of blood in the stool. What is this most likely a result of? A) Diet high in red meat B) Upper GI bleed C) Hemorrhoids D) Use of iron supplements

A) Avoid applying suction on or near the suture line. The nurse should avoid positioning the suction catheter on or near the graft suture lines. Application of suction in these areas could damage the graft. Self-sectioning may be unsafe because the patient may damage the suture line. Following a modified radical neck dissection with graft, the patient is usually positioned with the head of the bed elevated to promote drainage and reduce edema. Assessing viability of the graft is important but is not part of the suctioning procedure and may delay initiating suctioning. Maintenance of a patent airway is a nursing priority. Similarly, the patient's ability to swallow is an important assessment for the nurse to make; however, it is not directly linked to the patient's need for suctioning.

4. The nurse notes that a patient who has undergone skin, tissue, and muscle grafting following a modified radical neck dissection requires suctioning. What is the most important consideration for the nurse when suctioning this patient? A) Avoid applying suction on or near the suture line. B) Position patient on the non operative side with the head of the bed down. C) Assess the patient's ability to perform self-suctioning. D) Evaluate the patient's ability to swallow saliva and clear fluids.

B) "Abdominal ultrasound poses no known safety risks of any kind." An ultrasound produces no ill effects and there are not specific limits on its use, even during pregnancy.

40. A female patient has presented to the emergency department with right upper quadrant pain; the physician has ordered abdominal ultrasound to rule out cholecystitis (gallbladder infection). The patient expresses concern to the nurse about the safety of this diagnostic procedure. How should the nurse best respond? A) "Abdominal ultrasound is very safe, but it can't be performed if you're pregnant." B) "Abdominal ultrasound poses no known safety risks of any kind." C) "Current guidelines state that a person can have up to 3 ultrasounds per year." D) "Current guidelines state that a person can have up to 6 ultrasounds per year."

D) Contact the pharmacy to obtain a new bag of PN. Before PN infusion is administered, the solution must be inspected for separation, oily appearance (also known as a "cracked solution"), or any precipitate (which appears as white crystals). If any of these are present, it is not used. Warming or shaking the bag is inappropriate and unsafe.

40. A nurse is preparing to administer a patient's scheduled parenteral nutrition (PN). Upon inspecting the bag, the nurse notices that the presence of small amounts of white precipitate are present in the bag. What is the nurse's best action? A) Recognize this as an expected finding. B) Place the bag in a warm environment for 30 minutes. C) Shake the bag vigorously for 10 to 20 seconds. D) Contact the pharmacy to obtain a new bag of PN.

A) A GI malignancy Palpable nodules around the umbilicus, called Sister Mary Joseph's nodules, are a sign of a GI malignancy, usually a gastric cancer. This would not be a sign of dumping syndrome, peptic ulcer disease, or esophageal/gastric obstruction.

40. A patient has come to the clinic complaining of pain just above her umbilicus. When assessing the patient, the nurse notes Sister Mary Joseph's nodules. The nurse should refer the patient to the primary care provider to be assessed for what health problem? A) A GI malignancy B) Dumping syndrome C) Peptic ulcer disease D) Esophageal/gastric obstruction

A patients abdominal ultrasound indicates cholelithiasis. When the nurse is reviewing the patients laboratory studies, what finding is most closely associated with this diagnosis? A) Increased bilirubin B) Decreased serum cholesterol C) Increased blood urea nitrogen (BUN) D) Decreased serum alkaline phosphatase leve

A

D) "Instead of eating three meals a day, try eating smaller amounts more often." Management for a hiatal hernia includes frequent, small feedings that can pass easily through the esophagus. Avoiding beverages and particular foods or taking OTC antacids are not noted to be beneficial.

40. A patient seeking care because of recurrent heartburn and regurgitation is subsequently diagnosed with a hiatal hernia. Which of the following should the nurse include in health education? A) "Drinking beverages after your meal, rather than with your meal, may bring some relief." B) "It's best to avoid dry foods, such as rice and chicken, because they're harder to swallow." C) "Many patients obtain relief by taking over-the-counter antacids 30 minutes before eating." D) "Instead of eating three meals a day, try eating smaller amounts more often."

D) Ineffective Sexuality Patterns Related to Changes in Self-Concept The presence of an ostomy frequently has an effect on sexuality; this should be addressed thoughtfully in nursing care. None of the other listed diagnoses reflects the physiologic changes that result from colorectal surgery.

40. Which of the following is the most plausible nursing diagnosis for a patient whose treatment for colon cancer has necessitated a colonostomy? A) Risk for Unstable Blood Glucose Due to Changes in Digestion and Absorption B) Unilateral Neglect Related to Decreased Physical Mobility C) Risk for Excess Fluid Volume Related to Dietary Changes and Changes In Absorption D) Ineffective Sexuality Patterns Related to Changes in Self-Concept

A) Insertion of a nasogastric tube Decompression of the bowel through a nasogastric tube is necessary for all patients with small bowel obstruction. Peripheral IV access is normally sufficient. Enemas, suppositories, and laxatives are not indicated if an obstruction is present.

5. A 35-year-old male patient presents at the emergency department with symptoms of a small bowel obstruction. In collaboration with the primary care provider, what intervention should the nurse prioritize? A) Insertion of a nasogastric tube B) Insertion of a central venous catheter C) Administration of a mineral oil enema D) Administration of a glycerin suppository and an oral laxative

C) The patient maintains or gains weight. Expected outcomes for the patient following gastric surgery include ensuring that the patient is maintaining or gaining weight (patient should be weighed daily), experiencing no excessive diarrhea, and tolerating six small meals a day. Patients may require vitamin B12 supplementation by the intramuscular route and do not require a diet excessively rich in calcium.

5. A nurse is preparing to discharge a patient after recovery from gastric surgery. What is an appropriate discharge outcome for this patient? A) The patient's bowel movements maintain a loose consistency. B) The patient is able to tolerate three large meals a day. C) The patient maintains or gains weight. D) The patient consumes a diet high in calcium.

B) Prevent aspiration Protecting the client from aspirating is essential because aspiration can cause pneumonia, a potentially life-threatening disorder. Gastric ulcers are not a common complication of tube feeding in clients with ET tubes. Abdominal distention and diarrhea can both be associated with tube feeding, but prevention of these problems is not the primary rationale for confirming placement.

5. A nursing educator is reviewing the care of patients with feeding tubes and endotracheal tubes (ET). The educator has emphasized the need to check for tube placement in the stomach as well as residual volume. What is the main purpose of this nursing action? A) Prevent gastric ulcers B) Prevent aspiration C) Prevent abdominal distention D) Prevent diarrhea

A) The patient will require an upper endoscopy every 6 months to detect malignant changes. In the patient with Barrett's esophagus, the cells lining the lower esophagus have undergone change and are no longer squamous cells. The altered cells are considered precancerous and are a precursor to esophageal cancer. In order to facilitate early detection of malignant cells, an upper endoscopy is recommended every 6 months. H2receptor antagonists are commonly prescribed for patients with GERD; however, monitoring of liver enzymes is not routine. Stools that contain evidence of frank bleeding or that are tarry are not expected and should be reported immediately. When antacids are prescribed for patients with GERD, they should be taken as ordered whether or not the patient is symptomatic.

5. A patient with gastroesophageal reflux disease (GERD) has a diagnosis of Barrett's esophagus with minor cell changes. Which of the following principles should be integrated into the patient's subsequent care? A) The patient will require an upper endoscopy every 6 months to detect malignant changes. B) Liver enzymes must be checked regularly, as H2 receptor antagonists may cause hepatic damage. C) Small amounts of blood are likely to be present in the stools and are not cause for concern. D) Antacids may be discontinued when symptoms of heartburn subside.

C) Fluids must be increased to facilitate the evacuation of the stool. Postprocedural patient education includes information about increasing fluid intake; evaluating bowel movements for evacuation of barium; and noting increased number of bowel movements, because barium, due to its high osmolarity, may draw fluid into the bowel, thus increasing the intraluminal contents and resulting in greater output. Yellow stool, diarrhea, and anal bleeding are not expected.

5. An adult patient is scheduled for an upper GI series that will use a barium swallow. What teaching should the nurse include when the patient has completed the test? A) Stool will be yellow for the first 24 hours postprocedure. B) The barium may cause diarrhea for the next 24 hours. C) Fluids must be increased to facilitate the evacuation of the stool. D) Slight anal bleeding may be noted as the barium is passed.

B) Eat several small meals daily spaced at equal intervals. Due to decreased stomach capacity, the patient must consume small meals at intervals to meet nutritional requirements while avoiding a feeling of fullness and complications such as dumping syndrome. The patient should not consume fluids with meals and low-Fowler's positioning is recommended during and after meals. Carbohydrates should be limited.

6. A nurse caring for a patient who has had bariatric surgery is developing a teaching plan in anticipation of the patient's discharge. Which of the following is essential to include? A) Drink a minimum of 12 ounces of fluid with each meal. B) Eat several small meals daily spaced at equal intervals. C) Choose foods that are high in simple carbohydrates. D) Sit upright when eating and for 30 minutes afterward.

B) Tofu Nutritional management of inflammatory bowel disease requires ingestion of a diet that is bland, low-residue, high-protein, and high-vitamin. Tofu meets each of the criteria. Spinach, multigrain bagels, and blueberries are not low-residue.

6. A patient admitted with inflammatory bowel disease asks the nurse for help with menu selections. What menu selection is most likely the best choice for this patient? A) Spinach B) Tofu C) Multigrain bagel D) Blueberries

A) Checking the patient's capillary blood glucose levels regularly The solution, used as a base for most TPN, consists of a high dextrose concentration and may raise blood glucose levels significantly, resulting in hyperglycemia. This is a more salient threat than hunger, though this should be addressed. Dysrhythmias and decreased LOC are not among the most common complications.

6. The nurse is administering total parenteral nutrition (TPN) to a client who underwent surgery for gastric cancer. Which of the nurse's assessments most directly addresses a major complication of TPN? A) Checking the patient's capillary blood glucose levels regularly B) Having the patient frequently rate his or her hunger on a 10-point scale C) Measuring the patient's heart rhythm at least every 6 hours D) Monitoring the patient's level of consciousness each shift

A) Organic fruit juice Dental caries may be prevented by decreasing the amount of sugar and starch in the diet. Patients who snack should be encouraged to choose less cariogenic alternatives, such as fruits, vegetables, nuts, cheeses, or plain yogurt. Fruit juice is high in sugar, regardless of whether it is organic.

6. The school nurse is planning a health fair for a group of fifth graders and dental health is one topic that the nurse plans to address. What would be most likely to increase the risk of tooth decay? A) Organic fruit juice B) Roasted nuts C) Red meat that is high in fat D) Cheddar cheese

C) Inability to take in adequate oral food or fluids within 7 days The indications for PN include an inability to ingest adequate oral food or fluids within 7 days. Weight loss, muscle wasting combined with electrolyte imbalances, and aspiration indicate a need for nutritional support, but this does not necessary have to be parenteral.

7. A critical care nurse is caring for a patient diagnosed with acute pancreatitis. The nurse knows that the indications for starting parenteral nutrition (PN) for this patient are what? A) 5% deficit in body weight compared to preillness weight and increased caloric need B) Calorie deficit and muscle wasting combined with low electrolyte levels C) Inability to take in adequate oral food or fluids within 7 days D) Significant risk of aspiration coupled with decreased level of consciousness

C) Smokes one pack of cigarettes daily. Nicotine reduces secretion of pancreatic bicarbonate, which inhibits neutralization of gastric acid and can underlie gastritis. Protein drinks do not result in gastric inflammation. Antacid use is a response to experiencing symptoms of gastritis, not the etiology of gastritis. Alcohol ingestion can lead to gastritis; however, this generally occurs in patients with a history of consumption of alcohol on a daily basis.

7. A nurse is completing a health history on a patient whose diagnosis is chronic gastritis. Which of the data should the nurse consider most significantly related to the etiology of the patient's health problem? A) Consumes one or more protein drinks daily. B) Takes over-the-counter antacids frequently throughout the day. C) Smokes one pack of cigarettes daily. D) Reports a history of social drinking on a weekly basis.

A) Ineffective Tissue Perfusion Related to Bowel Ischemia When the bowel is completely obstructed, the possibility of strangulation and tissue necrosis (i.e., tissue death) warrants surgical intervention. As such, this immediate physiologic need is a nursing priority. Nutritional support and management of anxiety are necessary, but bowel ischemia is a more immediate threat. Skin integrity is not threatened.

7. A patient is admitted to the medical unit with a diagnosis of intestinal obstruction. When planning this patient's care, which of the following nursing diagnoses should the nurse prioritize? A) Ineffective Tissue Perfusion Related to Bowel Ischemia B) Imbalanced Nutrition: Less Than Body Requirements Related to Impaired Absorption C) Anxiety Related to Bowel Obstruction and Subsequent Hospitalization D) Impaired Skin Integrity Related to Bowel Obstruction

B) Presence of a painless sore with raised edges Malignant lesions of the oral cavity are most often painless lumps or sores with raised borders. Because they do not bother the patient, delay in seeking treatment occurs frequently, and negatively affects prognosis. Dull pain radiating to the ears and teeth is characteristic of malocclusion. Inflammation of the buccal mucosa causes discomfort and often occurs as a side effect of chemotherapy. Tenderness resulting in pain on chewing may be associated with gingivitis, abscess, irritation from dentures, and other causes. Pain related to oral cancer is a late symptom.

7. The nurse's comprehensive assessment of a patient includes inspection for signs of oral cancer. What assessment finding is most characteristic of oral cancer in its early stages? A) Dull pain radiating to the ears and teeth B) Presence of a painless sore with raised edges C) Areas of tenderness that make chewing difficult D) Diffuse inflammation of the buccal mucosa

A) Indicates acceptance of altered appearance and demonstrates positive self-image Since radical neck dissection involves removal of the sternocleidomastoid muscle, spinal accessory muscles, and cervical lymph nodes on one side of the neck, the patient's appearance is visibly altered. The face generally appears asymmetric, with a visible neck depression; shoulder drop also occurs frequently. These changes have the potential to negatively affect self-concept and body image. Facilitating adaptation to these changes is a crucial component of nursing intervention. Patients who have had head and neck surgery generally report less pain as compared with other postoperative patients; however, the nurse must assess each individual patient's level of pain and response to analgesics. Patients may experience transient hoarseness following a radical neck dissection; however, their ability to communicate is not permanently altered. Stress management is beneficial but would not be considered the priority in this clinical situation.

8. A medical nurse who is caring for a patient being discharged home after a radical neck dissection has collaborated with the home health nurse to develop a plan of care for this patient. What is a priority psychosocial outcome for a patient who has had a radical neck dissection? A) Indicates acceptance of altered appearance and demonstrates positive self-image B) Freely expresses needs and concerns related to postoperative pain management C) Compensates effectively for alteration in ability to communicate related to dysarthria D) Demonstrates effective stress management techniques to promote muscle relaxation

A) Fowler's Positioning the patient in a Fowler's position postoperatively promotes comfort and facilitates emptying of the stomach following gastric surgery. Any position that involves lying down delays stomach emptying and is not recommended for this type of patient. Supine positioning and the left lateral (left Sim's) position do not achieve this goal.

8. A nurse in the postanesthesia care unit admits a patient following resection of a gastric tumor. Following immediate recovery, the patient should be placed in which position to facilitate patient comfort and gastric emptying? A) Fowler's B) Supine C) Left lateral D) Left Sim's

A) Intravenous fat emulsions may be infused simultaneously with PN through a Y-connector close to the infusion site and should not be filtered. Intravenous fat emulsions may be infused simultaneously with PN through a Y-connector close to the infusion site and should not be filtered. The patient does not need another intravenous line for the fat emulsion. The IVFE cannot be piggy-backed into any existing IV solution that is infusing.

8. A nurse is preparing to administer a patient's intravenous fat emulsion simultaneously with parenteral nutrition (PN). Which of the following principles should guide the nurse's action? A) Intravenous fat emulsions may be infused simultaneously with PN through a Y-connector close to the infusion site and should not be filtered. B) The nurse should prepare for placement of another intravenous line, as intravenous fat emulsions may not be infused simultaneously through the line used for PN. C) Intravenous fat emulsions may be infused simultaneously with PN through a Y-connector close to the infusion site after running the emulsion through a filter. D) The intravenous fat emulsions can be piggy-backed into any existing IV solution that is infusing.

A) High levels of alcohol consumption Risk factors include high alcohol intake; cigarette smoking; and high-fact, high-protein, low-fiber diet. Diverticulitis, obstruction, and stress are not noted as risk factors for colorectal cancer.

8. A nurse is presenting an educational event to a local community group. When speaking about colorectal cancer, what risk factor should the nurse cite? A) High levels of alcohol consumption B) History of bowel obstruction C) History of diverticulitis D) Longstanding psychosocial stress

C) Lying on the left side with legs drawn toward the chest For best visualization, colonoscopy is performed while the patient is lying on the left side with the legs drawn up toward the chest. A knee-chest position, lying on the stomach with legs drawn to the chest, and a prone position with two pillows elevating the legs do not allow for the best visualization.

8. The nurse is providing health education to a patient scheduled for a colonoscopy. The nurse should explain that she will be placed in what position during this diagnostic test? A) In a knee-chest position (lithotomy position) B) Lying prone with legs drawn toward the chest C) Lying on the left side with legs drawn toward the chest D) In a prone position with two pillows elevating the buttocks

C) TNA is less costly than PN. TNA is mixed in one container and administered to the patient over a 24-hour period. A 1.5-micron filter is used with the TNA solution. Advantages of the TNA over PN include cost savings. Pharmacy staff must prepare both solutions.

9. A nurse is participating in a patient's care conference and the team is deciding between parenteral nutrition (PN) and a total nutritional admixture (TNA). What advantages are associated with providing TNA rather than PN? A) TNA can be mixed by a certified registered nurse. B) TNA can be administered over 8 hours, while PN requires 24-hour administration. C) TNA is less costly than PN. D) TNA does not require the use of a micron filter.

B) Regurgitation of undigested food An esophageal diverticulum is an outpouching of mucosa and submucosa that protrudes through the esophageal musculature. Food becomes trapped in the pouch and is frequently regurgitated when the patient assumes a recumbent position. The patient may experience difficulty swallowing; however, burning pain is not a typical finding. Symptoms mimicking a heart attack are characteristic of GERD. Chronic parotid abscesses are not associated with a diagnosis of esophageal diverticulum.

9. A patient has been diagnosed with an esophageal diverticulum after undergoing diagnostic imaging. When taking the health history, the nurse should expect the patient to describe what sign or symptom? A) Burning pain on swallowing B) Regurgitation of undigested food C) Symptoms mimicking a heart attack D) Chronic parotid abscesses

A) NSAIDs NSAIDs can cause a false-positive fecal occult blood test. Acetaminophen, vitamin D supplements, and fiber supplements do not have this effect.

9. A patient has sought care because of recent dark-colored stools. As a result, a fecal occult blood test has been ordered. The nurse should instruct the patient to avoid which of the following prior to collecting a stool sample? A) NSAIDs B) Acetaminophen C) OTC vitamin D supplements D) Fiber supplements

C) The patient's polyps constitute a risk factor for cancer. Although most polyps do not develop into invasive neoplasms, they must be identified and followed closely. They are very common, but are not classified as a normal, age-related physiologic change. Diet will not help them resolve and they do not typically lead to obstructions.

9. A patient's screening colonoscopy revealed the presence of numerous polyps in the large bowel. What principle should guide the subsequent treatment of this patient's health problem? A) Adherence to a high-fiber diet will help the polyps resolve. B) The patient should be assured that these are a normal, age-related physiologic change. C) The patient's polyps constitute a risk factor for cancer. D) The presence of polyps is associated with an increased risk of bowel obstruction.

A 37-year-old male patient presents at the emergency department (ED) complaining of nausea and vomiting and severe abdominal pain. The patients abdomen is rigid, and there is bruising to the patients flank. The patients wife states that he was on a drinking binge for the past 2 days. The ED nurse should assist in assessing the patient for what health problem? A) Severe pancreatitis with possible peritonitis B) Acute cholecystitis C) Chronic pancreatitis D) Acute appendicitis with possible perforation

A

A nurse is caring for a 73-year-old patient with a urethral obstruction related to prostatic enlargement. When planning this patient's care, the nurse should be aware of the consequent risk of what complication? A) Urinary tract infection B) Enuresis C) Polyuria D) Proteinuria

A

A patient asks the nurse why kidney problems can cause gastrointestinal disturbances. What relationship should the nurse describe? A) The right kidney's proximity to the pancreas, liver, and gallbladder B) The indirect impact of digestive enzymes on renal function C) That the peritoneum encapsulates the GI system and the kidneys D) The left kidney's connection to the common bile duct

A

A patient has been scheduled for an ultrasound of the gallbladder the following morning. What should the nurse do in preparation for this diagnostic study? A) Have the patient refrain from food and fluids after midnight. B) Administer the contrast agent orally 10 to 12 hours before the study. C) Administer the radioactive agent intravenously the evening before the study. D) Encourage the intake of 64 ounces of water 8 hours before the study.

A

The nurse is caring for a patient who has returned to the postsurgical suite after post-anesthetic recovery from a nephrectomy. The nurses most recent hourly assessment reveals a significant drop in level of consciousness and BP as well as scant urine output over the past hour. What is the nurses best response? A) Assess the patient for signs of bleeding and inform the physician. B) Monitor the patients vital signs every 15 minutes for the next hour. C) Reposition the patient and reassess vital signs. D) Palpate the patients flanks for pain and inform the physician.

A) Assess the patient for signs of bleeding and inform the physician. Bleeding may be suspected when the patient experiences fatigue and when urine output is less than 30 mL/h. The physician must be made aware of this finding promptly. Palpating the patients flanks would cause intense pain that is of no benefit to assessment.

The nurse is caring for a patient postoperative day 4 following a kidney transplant. When assessing for potential signs and symptoms of rejection, what assessment should the nurse prioritize? A) Assessment of the quantity of the patients urine output B) Assessment of the patients incision C) Assessment of the patients abdominal girth D) Assessment for flank or abdominal pain

A) Assessment of the quantity of the patients urine output After kidney transplantation, the nurse should perform all of the listed assessments. However, oliguria is considered to be more suggestive of rejection than changes to the patients abdomen or incision.

A patient with ESKD is scheduled to begin hemodialysis. The nurse is working with the patient to adapt the patients diet to maximize the therapeutic effect and minimize the risks of complications. The patients diet should include which of the following modifications? Select all that apply. A) Decreased protein intake B) Decreased sodium intake C) Increased potassium intake D) Fluid restriction E) Vitamin D supplementation

A) Decreased protein intake B) Decreased sodium intake D) Fluid restriction Restricting dietary protein decreases the accumulation of nitrogenous wastes, reduces uremic symptoms, and may even postpone the initiation of dialysis for a few months. Restriction of fluid is also part of the dietary prescription because fluid accumulation may occur. As well, sodium is usually restricted to 2 to 3 g/day. Potassium intake is usually limited, not increased, and there is no particular need for vitamin D supplementation.

Renal failure can have prerenal, renal, or postrenal causes. A patient with acute kidney injury is being assessed to determine where, physiologically, the cause is. If the cause is found to be prerenal, which condition most likely caused it? A) Heart failure B) Glomerulonephritis C) Ureterolithiasis D) Aminoglycoside toxicity

A) Heart failure By causing inadequate renal perfusion, heart failure can lead to prerenal failure. Glomerulonephritis and aminoglycoside toxicity are renal causes, and ureterolithiasis is a postrenal cause.

The nurse is assessing a patient suspected of having developed acute glomerulonephritis. The nurse should expect to address what clinical manifestation that is characteristic of this health problem? A) Hematuria B) Precipitous decrease in serum creatinine levels C) Hypotension unresolved by fluid administration D) Glucosuria

A) Hematuria The primary presenting feature of acute glomerulonephritis is hematuria (blood in the urine), which may be microscopic (identifiable through microscopic examination) or macroscopic or gross (visible to the eye). Proteinuria, primarily albumin, which is present, is due to increased permeability of the glomerular membrane. Blood urea nitrogen (BUN) and serum creatinine levels may rise as urine output drops. Some degree of edema and hypertension is noted in most patients.

A football player is thought to have sustained an injury to his kidneys from being tackled from behind. The ER nurse caring for the patient reviews the initial orders written by the physician and notes that an order to collect all voided urine and send it to the laboratory for analysis. The nurse understands that this nursing intervention is important for what reason? A) Hematuria is the most common manifestation of renal trauma and blood losses may be microscopic, so laboratory analysis is essential. B) Intake and output calculations are essential and the laboratory will calculate the precise urine output produced by this patient. C) A creatinine clearance study may be ordered at a later time and the laboratory will hold all urine until it is determined if the test will be necessary. D) There is great concern about electrolyte imbalances and the laboratory will monitor the urine for changes in potassium and sodium concentrations.

A) Hematuria is the most common manifestation of renal trauma and blood losses may be microscopic, so laboratory analysis is essential. Hematuria is the most common manifestation of renal trauma; its presence after trauma suggests renal injury. Hematuria may not occur, or it may be detectable only on microscopic examination. All urine should be saved and sent to the laboratory for analysis to detect RBCs and to evaluate the course of bleeding. Measuring intake and output is not a function of the laboratory. The laboratory does not save urine to test creatinine clearance at a later time. The laboratory does not monitor the urine for sodium or potassium concentrations.

A 45-year-old man with diabetic nephropathy has ESKD and is starting dialysis. What should the nurse teach the patient about hemodialysis? A) Hemodialysis is a treatment option that is usually required three times a week. B) Hemodialysis is a program that will require you to commit to daily treatment. C) This will require you to have surgery and a catheter will need to be inserted into your abdomen. D) Hemodialysis is a treatment that is used for a few months until your kidney heals and starts to produce urine again.

A) Hemodialysis is a treatment option that is usually required three times a week. Hemodialysis is the most commonly used method of dialysis. Patients receiving hemodialysis must undergo treatment for the rest of their lives or until they undergo successful kidney transplantation. Treatments usually occur three times a week for at least 3 to 4 hours per treatment.

A patient with ESKD receives continuous ambulatory peritoneal dialysis. The nurse observes that the dialysate drainage fluid is cloudy. What is the nurses most appropriate action? A) Inform the physician and assess the patient for signs of infection. B) Flush the peritoneal catheter with normal saline. C) Remove the catheter promptly and have the catheter tip cultured. D) Administer a bolus of IV normal saline as ordered.

A) Inform the physician and assess the patient for signs of infection Peritonitis is the most common and serious complication of peritoneal dialysis. The first sign of peritonitis is cloudy dialysate drainage fluid, so prompt reporting to the primary care provider and rapid assessment for other signs of infection are warranted. Administration of an IV bolus is not necessary or appropriate and the physician would determine whether removal of the catheter is required. Flushing the catheter does not address the risk for infection.

The nurse has identified the nursing diagnosis of risk for infection in a patient who undergoes peritoneal dialysis. What nursing action best addresses this risk? A) Maintain aseptic technique when administering dialysate. B) Wash the skin surrounding the catheter site with soap and water prior to each exchange. C) Add antibiotics to the dialysate as ordered. D) Administer prophylactic antibiotics by mouth or IV as ordered.

A) Maintain aseptic technique when administering dialysate. Aseptic technique is used to prevent peritonitis and other infectious complications of peritoneal dialysis. It is not necessary to cleanse the skin with soap and water prior to each exchange. Antibiotics may be added to dialysate to treat infection, but they are not used to prevent infection.

The nurse is caring for a patient status after a motor vehicle accident. The patient has developed AKI. What is the nurses role in caring for this patient? Select all that apply. A) Providing emotional support for the family B) Monitoring for complications C) Participating in emergency treatment of fluid and electrolyte imbalances D) Providing nursing care for primary disorder (trauma) E) Directing nutritional interventions

A) Providing emotional support for the family B) Monitoring for complications C) Participating in emergency treatment of fluid and electrolyte imbalances D) Providing nursing care for primary disorder (trauma) The nurse has an important role in caring for the patient with AKI. The nurse monitors for complications, participates in emergency treatment of fluid and electrolyte imbalances, assesses the patients progress and response to treatment, and provides physical and emotional support. Additionally, the nurse keeps family members informed about the patients condition, helps them understand the treatments, and provides psychological support. Although the development of AKI may be the most serious problem, the nurse continues to provide nursing care indicated for the primary disorder (e.g., burns, shock, trauma, obstruction of the urinary tract). The nurse does not direct the patients nutritional status; the dietician and the physician normally collaborate on directing the patients nutritional status.

The nurse is caring for a patient who has just returned to the post-surgical unit following renal surgery. When assessing the patients output from surgical drains, the nurse should assess what parameters? Select all that apply. A) Quantity of output B) Color of the output C) Visible characteristics of the output D) Odor of the output E) pH of the output

A) Quantity of output B) Color of the output C) Visible characteristics of the output Urine output and drainage from tubes inserted during surgery are monitored for amount, color, and type or characteristics. Odor and pH are not normally assessed.

A nurse on the renal unit is caring for a patient who will soon begin peritoneal dialysis. The family of the patient asks for education about the peritoneal dialysis catheter that has been placed in the patients peritoneum. The nurse explains the three sections of the catheter and talks about the two cuffs on the dialysis catheter. What would the nurse explain about the cuffs? Select all that apply. A) The cuffs are made of Dacron polyester. B) The cuffs stabilize the catheter. C) The cuffs prevent the dialysate from leaking. D) The cuffs provide a barrier against microorganisms. E) The cuffs absorb dialysate

A) The cuffs are made of Dacron polyester. B) The cuffs stabilize the catheter. C) The cuffs prevent the dialysate from leaking. D) The cuffs provide a barrier against microorganisms. Most of these catheters have two cuffs, which are made of Dacron polyester. The cuffs stabilize the catheter, limit movement, prevent leaks, and provide a barrier against microorganisms. They do not absorb dialysate.

The nurse is working on the renal transplant unit. To reduce the risk of infection in a patient with a transplanted kidney, it is imperative for the nurse to do what? A) Wash hands carefully and frequently. B) Ensure immediate function of the donated kidney. C) Instruct the patient to wear a face mask. D) Bar visitors from the patients room.

A) Wash hands carefully and frequently. The nurse ensures that the patient is protected from exposure to infection by hospital staff, visitors, and other patients with active infections. Careful handwashing is imperative; face masks may be worn by hospital staff and visitors to reduce the risk for transmitting infectious agents while the patient is receiving high doses of immunosuppressants. Visitors may be limited, but are not normally barred outright. Ensuring kidney function is vital, but does not prevent infection.

A nurse knows that specific areas in the ureters have a propensity for obstruction. Prompt management of renal calculi is most important when the stone is located where?

A) In the ureteropelvic junction

A patient with pancreatic cancer has been scheduled for a pancreaticoduodenectomy (Whipple procedure). During health education, the patient should be informed that this procedure will involve the removal of which of the following? Select all that apply. A) Gallbladder B) Part of the stomach C) Duodenum D) Part of the common bile duct E) Part of the rectum

A, B, C, D

A patients assessment and diagnostic testing are suggestive of acute pancreatitis. When the nurse is performing the health interview, what assessment questions address likely etiologic factors? Select all that apply. A) How many alcoholic drinks do you typically consume in a week? B) Have you ever been tested for diabetes? C) Have you ever been diagnosed with gallstones? D) Would you say that you eat a particularly high-fat diet? E) Does anyone in your family have cystic fibrosis?

A, C

The nurse is preparing to collect an ordered urine sample for urinalysis. The nurse should be aware that this test will include what assessment parameters? Select all that apply. A) Specific gravity of the patient's urine B) Testing for the presence of glucose in the patient's urine C) Microscopic examination of urine sediment for RBCs D) Microscopic examination of urine sediment for casts E) Testing for BUN and creatinine in the patient's urine

A,B,C,D

A patient's physician has ordered a liver panel in response to the patient's development of jaundice. When reviewing the results of this laboratory testing, the nurse should expect to review what blood tests? Select all that apply. A. Alanine aminotransferase (ALT) B. C-reactive protein (CRP) C. Gamma-glutamyl transferase (GGT) D. Aspartate aminotransferase (AST) E. B-type natriuretic peptide (BNP)

A. Alanine aminotransferase (ALT) B. C-reactive protein (CRP) D. Aspartate aminotransferase (AST) Rationale: Liver function testing includes GGT, ALT, and AST. CRP addresses the presence of generalized inflammation and BNP is relevant to heart failure; neither is included in a liver panel.

A nurse is caring for a patient with hepatic encephalopathy. While making the initial shift assessment, the nurse notes that the patient has a flapping tremor of the hands. The nurse should document the presence of what sign of liver disease? A. Asterixis B. Constructional apraxia C. Fector hepaticus D. Palmar erythema

A. Asterixis Rationale: The nurse will document that a patient exhibiting a flapping tremor of the hands is demonstrating asterixis. While constructional apraxia is a motor disturbance, it is the inability to reproduce a simple figure. Fector hepaticus is a sweet, slightly fecal odor to the breath and not associated with a motor disturbance. Skin changes associated with liver dysfunction may include palmar erythema, which is a reddening of the palms, but is not a flapping tremor.

A group of nurses have attended an inservice on the prevention of occupationally acquired diseases that affect healthcare providers. What action has the greatest potential to reduce a nurse's risk of acquiring hepatitis C in the workplace? A. Disposing of sharps appropriately and not recapping needles B. Performing meticulous hand hygiene at the appropriate moments in care C. Adhering to the recommended schedule of immunizations D. Wearing an N95 mask when providing care for patients on airborne precautions

A. Disposing of sharps appropriately and not recapping needles Rationale: HCV is blood-borne. Consequently, prevention of needle stick injuries are paramount. Hand hygiene, immunizations and appropriate use of masks are important aspects of overall infection control, but these actions do not directly mitigate the risk of HCV.

A nurse is caring for a patient with cancer of the liver whose condition has required the insertion of a percutaneous biliary drainage system. The nurse's most recent assessment reveals the presence of dark green fluid in the collection container. What is the nurse's best response to this assessment finding? A. Document the presence of normal bile output. B. Irrigate the drainage system with normal saline as ordered. C. Aspirate a sample of the drainage for culture. D. Promptly report this assessment finding to the primary care provider.

A. Document the presence of normal bile output. Rationale: Bile is usually a dark green or brownish-yellow color, so this would constitute an expected assessment finding, with no other action necessary.

During a health education session, a participant has asked about the hepatitis E virus. What prevention measure should the nurse recommend for preventing infection with this virus? A. Following proper hand-washing techniques B. Avoiding chemicals that are toxic to the liver C. Wearing a condom during sexual contact D. Limiting alcohol intake

A. Following proper hand-washing techniques Rationale: Avoiding contact with the hepatitis E virus through good hygiene, including hand-washing, is the major method of prevention. Hepatitis E is transmitted by the fecaloral route, principally through contaminated water in areas with poor sanitation. Consequently, none of the other listed preventative measure is indicated.

A triage nurse in the emergency department is assessing a patient who presented with complaints of general malaise. Assessment reveals the presence of jaundice and increased abdominal girth. What assessment question best addresses the possible etiology of this patient's presentation? A. How many alcoholic drinks do you typically consume in a week? B. To the best of your knowledge, are your immunizations up to date? C. Have you ever worked in an occupation where you might have been exposed to toxins? D. Has anyone in your family ever experienced symptoms similar to yours?

A. How many alcoholic drinks do you typically consume in a week? Rationale: Signs or symptoms of hepatic dysfunction indicate a need to assess for alcohol use. Immunization status, occupational risks, and family history are also relevant considerations, but alcohol use is a more common etiologic factor in liver disease.

A nurse educate is teaching a group of recent nursing graduates about their occupational risks for contracting hepatitis B. What preventative measures should the educator promote? Select all that apply. A. Immunizations B. Use of standard precautions C. Consumption of vitamin-rich diet D. Annual vitamin K injections E. Annual vitamin B12 injections

A. Immunizations B. Use of standard precautions Rationale: People who are at high risk, including nurses and other health care personnel exposed to blood or blood products, should receive active immunization. The consistent use of standard precautions is also highly beneficial. Vitamin supplementation is unrelated to an individual's risk of HBV.

A patient with a C5 spinal cord injury is tetraplegic. After being moved out of the ICU, the patient complains of a severe throbbing headache. What should the nurse do first? A) Check the patient's indwelling urinary catheter for kinks to ensure patency. B) Lower the HOB to improve perfusion. C) Administer analgesia. D) Reassure the patient that headaches are expected after spinal cord injuries.

Ans: A Feedback: A severe throbbing headache is a common symptom of autonomic dysreflexia, which occurs after injuries to the spinal cord above T6. The syndrome is usually brought on by sympathetic stimulation, such as bowel and bladder distention. Lowering the HOB can increase ICP. Before administering analgesia, the nurse should check the patient's catheter, record vital signs, and perform an abdominal assessment. A severe throbbing headache is a dangerous symptom in this patient and is not expected.

A patient who just suffered a suspected ischemic stroke is brought to the ED by ambulance. On what should the nurse's primary assessment focus? A) Cardiac and respiratory status B) Seizure activity C) Pain D) Fluid and electrolyte balance

Ans: A Feedback: Acute care begins with managing ABCs. Patients may have difficulty keeping an open and clear airway secondary to decreased LOC. Neurologic assessment with close monitoring for signs of increased neurologic deficit and seizure activity occurs next. Fluid and electrolyte balance must be controlled carefully with the goal of adequate hydration to promote perfusion and decrease further brain activity.

When preparing to discharge a patient home, the nurse has met with the family and warned them that the patient may exhibit unexpected emotional responses. The nurse should teach the family that these responses are typically a result of what cause? A) Frustration around changes in function and communication B) Unmet physiologic needs C) Changes in brain activity during sleep and wakefulness D) Temporary changes in metabolism

Ans: A Feedback: Emotional problems associated with stroke are often related to the new challenges around ADLs and communication. These challenges are more likely than metabolic changes, unmet physiologic needs, or changes in brain activity, each of which should be ruled out.

A patient with lower back pain is scheduled for myelography using metrizamide (a water-soluble contrast dye). After the test, the nurse should prioritize what action? A) Positioning the patient with the head of the bed elevated 45 degrees B) Administering IV morphine sulfate to prevent headache C) Limiting fluids for the next 12 hours D) Helping the patient perform deep breathing and coughing exercises

Ans: A Feedback: After myelography, the patient lies in bed with the head of the bed elevated 30 to 45 degrees. The patient is advised to remain in bed in the recommended position for 3 hours or as prescribed. Drinking liberal amounts of fluid for rehydration and replacement of CSF may decrease the incidence of post-lumbar puncture headache. Deep breathing and coughing exercises are not normally necessary since there is no consequent risk of atelectasis.

A patient who has experienced an ischemic stroke has been admitted to the medical unit. The patient's family in adamant that she remain on bed rest to hasten her recovery and to conserve energy. What principle of care should inform the nurse's response to the family? A) The patient should mobilize as soon as she is physically able. B) To prevent contractures and muscle atrophy, bed rest should not exceed 4 weeks. C) The patient should remain on bed rest until she expresses a desire to mobilize. D) Lack of mobility will greatly increase the patient's risk of stroke recurrence.

Ans: A Feedback: As soon as possible, the patient is assisted out of bed and an active rehabilitation program is started. Delaying mobility causes complications, but not necessarily stroke recurrence. Mobility should not be withheld until the patient initiates.

The nurse is caring for a patient whose recent health history includes an altered LOC. What should be the nurse's first action when assessing this patient? A) Assessing the patient's verbal response B) Assessing the patient's ability to follow complex commands C) Assessing the patient's judgment D) Assessing the patient's response to pain

Ans: A Feedback: Assessment of the patient with an altered LOC often starts with assessing the verbal response through determining the patient's orientation to time, person, and place. In most cases, this assessment will precede each of the other listed assessments, even though each may be indicated.

The school nurse has been called to the football field where player is immobile on the field after landing awkwardly on his head during a play. While awaiting an ambulance, what action should the nurse perform? A) Ensure that the player is not moved. B) Obtain the player's vital signs, if possible. C) Perform a rapid assessment of the player's range of motion. D) Assess the player's reflexes.

Ans: A Feedback: At the scene of the injury, the patient must be immobilized on a spinal (back) board, with the head and neck maintained in a neutral position, to prevent an incomplete injury from becoming complete. This is a priority over determining the patient's vital signs. It would be inappropriate to test ROM or reflexes.

The nurse is caring for a patient whose spinal cord injury has caused recent muscle spasticity. What medication should the nurse expect to be ordered to control this? A) Baclofen (Lioresal) B) Dexamethasone (Decadron) C) Mannitol (Osmitrol) D) Phenobarbital (Luminal)

Ans: A Feedback: Baclofen is classified as an antispasmodic agent in the treatment of muscles spasms related to spinal cord injury. Decadron is an anti-inflammatory medication used to decrease inflammation in both SCI and head injury. Mannitol is used to decrease cerebral edema in patients with head injury. Phenobarbital is an anticonvulsant that is used in the treatment of seizure activity.

A patient who is being treated in the hospital for a spinal cord injury is advocating for the removal of his urinary catheter, stating that he wants to try to resume normal elimination. What principle should guide the care team's decision regarding this intervention? A) Urinary retention can have serious consequences in patients with SCIs. B) Urinary function is permanently lost following an SCI. C) Urinary catheters should not remain in place for more than 7 days. D) Overuse of urinary catheters can exacerbate nerve damage.

Ans: A Feedback: Bladder distention, a major cause of autonomic dysreflexia, can also cause trauma. For this reason, removal of a urinary catheter must be considered with caution. Extended use of urinary catheterization is often necessary following SCI. The effect of a spinal cord lesion on urinary function depends on the level of the injury. Catheter use does not cause nerve damage, although it is a major risk factor for UTIs.

A patient is admitted to the neurologic ICU with a suspected diffuse axonal injury. What would be the primary neuroimaging diagnostic tool used on this patient to evaluate the brain structure? A) MRI B) PET scan C) X-ray D) Ultrasound

Ans: A Feedback: CT and MRI scans, the primary neuroimaging diagnostic tools, are useful in evaluating the brain structure. Ultrasound would not show the brain nor would an x-ray. A PET scan shows brain function, not brain structure.

A patient has developed diabetes insipidus after having increased ICP following head trauma. What nursing assessment best addresses this complication? A) Vigilant monitoring of fluid balance B) Continuous BP monitoring C) Serial arterial blood gases (ABGs) D) Monitoring of the patient's airway for patency

Ans: A Feedback: Diabetes insipidus requires fluid and electrolyte replacement, along with the administration of vasopressin, to replace and slow the urine output. Because of these alterations in fluid balance, careful monitoring is necessary. None of the other listed assessments directly addresses the major manifestations of diabetes insipidus.

A patient with a T2 injury is in spinal shock. The nurse will expect to observe what assessment finding? A) Absence of reflexes along with flaccid extremities B) Positive Babinski's reflex along with spastic extremities C) Hyperreflexia along with spastic extremities D) Spasticity of all four extremities

Ans: A Feedback: During the period immediately following a spinal cord injury, spinal shock occurs. In spinal shock, all reflexes are absent and the extremities are flaccid. When spinal shock subsides, the patient demonstrates a positive Babinski's reflex, hyperreflexia, and spasticity of all four extremities

The nurse has created a plan of care for a patient who is at risk for increased ICP. The patient's care plan should specify monitoring for what early sign of increased ICP? A) Disorientation and restlessness B) Decreased pulse and respirations C) Projectile vomiting D) Loss of corneal reflex

Ans: A Feedback: Early indicators of ICP include disorientation and restlessness. Later signs include decreased pulse and respirations, projectile vomiting, and loss of brain stem reflexes, such as the corneal reflex.

The nurse is planning the care of a patient with a T1 spinal cord injury. The nurse has identified the diagnosis of "risk for impaired skin integrity." How can the nurse best address this risk? A) Change the patient's position frequently. B) Provide a high-protein diet. C) Provide light massage at least daily. D) Teach the patient deep breathing and coughing exercises.

Ans: A Feedback: Frequent position changes are among the best preventative measures against pressure ulcers. A high-protein diet can benefit wound healing, but does not necessarily prevent skin breakdown. Light massage and deep breathing do not protect or restore skin integrity.

The nurse is caring for a patient who sustained a moderate head injury following a bicycle accident. The nurse's most recent assessment reveals that the patient's respiratory effort has increased. What is the nurse's most appropriate response? A) Inform the care team and assess for further signs of possible increased ICP. B) Administer bronchodilators as ordered and monitor the patient's LOC. C) Increase the patient's bed height and reassess in 30 minutes. D) Administer a bolus of normal saline as ordered.

Ans: A Feedback: Increased respiratory effort can be suggestive of increasing ICP, and the care team should be promptly informed. A bolus of IV fluid will not address the problem. Repositioning the patient and administering bronchodilators are insufficient responses, even though these actions may later be ordered.

After a major ischemic stroke, a possible complication is cerebral edema. Nursing care during the immediate recovery period from an ischemic stroke should include which of the following? A) Positioning to avoid hypoxia B) Maximizing PaCO2 C) Administering hypertonic IV solution D) Initiating early mobilization

Ans: A Feedback: Interventions during this period include measures to reduce ICP, such as administering an osmotic diuretic (e.g., mannitol), maintaining the partial pressure of carbon dioxide (PaCO2) within the range of 30 to 35 mm Hg, and positioning to avoid hypoxia. Hypertonic IV solutions are not used unless sodium depletion is evident. Mobilization would take place after the immediate threat of increased ICP has past.

A patient is being given a medication that stimulates her parasympathetic system. Following administration of this medication, the nurse should anticipate what effect? A) Constricted pupils B) Dilated bronchioles C) Decreased peristaltic movement D) Relaxed muscular walls of the urinary bladder

Ans: A Feedback: Parasympathetic stimulation results in constricted pupils, constricted bronchioles, increased peristaltic movement, and contracted muscular walls of the urinary bladder.

A patient who suffered an ischemic stroke now has disturbed sensory perception. What principle should guide the nurse's care of this patient? A) The patient should be approached on the side where visual perception is intact. B) Attention to the affected side should be minimized in order to decrease anxiety. C) The patient should avoid turning in the direction of the defective visual field to minimize shoulder subluxation. D) The patient should be approached on the opposite side of where the visual perception is intact to promote recovery.

Ans: A Feedback: Patients with decreased field of vision should first be approached on the side where visual perception is intact. All visual stimuli should be placed on this side. The patient can and should be taught to turn the head in the direction of the defective visual field to compensate for this loss. The nurse should constantly remind the patient of the other side of the body and should later stand at a position that encourages the patient to move or turn to visualize who and what is in the room.

A gerontologic nurse educator is providing practice guidelines to unlicensed care providers. Because reaction to painful stimuli is sometimes blunted in older adults, what must be used with caution? A) Hot or cold packs B) Analgesics C) Anti-inflammatory medications D) Whirlpool baths

Ans: A Feedback: Reaction to painful stimuli may be decreased with age. Because pain is an important warning signal, caution must be used when hot or cold packs are used. The older patient may be burned or suffer frostbite before being aware of any discomfort. Any medication is used with caution in the elderly, but not because of the decreased sense of heat or cold. Whirlpool baths are generally not a routine treatment ordered for the elderly.

The nurse is caring for a patient with increased intracranial pressure (ICP) caused by a traumatic brain injury. Which of the following clinical manifestations would suggest that the patient may be experiencing increased brain compression causing brain stem damage? A) Hyperthermia B) Tachycardia C) Hypertension D) Bradypnea

Ans: A Feedback: Signs of increasing ICP include slowing of the heart rate (bradycardia), increasing systolic BP, and widening pulse pressure. As brain compression increases, respirations become rapid, BP may decrease, and the pulse slows further. A rapid rise in body temperature is regarded as unfavorable. Hyperthermia increases the metabolic demands of the brain and may indicate brain stem damage.

A patient exhibiting an uncoordinated gait has presented at the clinic. Which of the following is the most plausible cause of this patient's health problem? A) Cerebellar dysfunction B) A lesion in the pons C) Dysfunction of the medulla D) A hemorrhage in the midbrain

Ans: A Feedback: The cerebellum controls fine movement, balance, position sense, and integration of sensory input. Portions of the pons control the heart, respiration, and blood pressure. Cranial nerves IX through XII connect to the brain in the medulla. Cranial nerves III and IV originate in the midbrain.

The patient has been diagnosed with aphasia after suffering a stroke. What can the nurse do to best make the patient's atmosphere more conducive to communication? A) Provide a board of commonly used needs and phrases. B) Have the patient speak to loved ones on the phone daily. C) Help the patient complete his or her sentences. D) Speak in a loud and deliberate voice to the patient.

Ans: A Feedback: The inability to talk on the telephone or answer a question or exclusion from conversation causes anger, frustration, fear of the future, and hopelessness. A common pitfall is for the nurse or other health care team member to complete the thoughts or sentences of the patient. This should be avoided because it may cause the patient to feel more frustrated at not being allowed to speak and may deter efforts to practice putting thoughts together and completing a sentence. The patient may also benefit from a communication board, which has pictures of commonly requested needs and phrases. The board may be translated into several languages.

When caring for a patient with increased ICP the nurse knows the importance of monitoring for possible secondary complications, including syndrome of inappropriate antidiuretic hormone (SIADH). What nursing interventions would the nurse most likely initiate if the patient developed SIADH? A) Fluid restriction B) Transfusion of platelets C) Transfusion of fresh frozen plasma (FFP) D) Electrolyte restriction

Ans: A Feedback: The nurse also assesses for complications of increased ICP, including diabetes insipidus, and SIADH. SIADH requires fluid restriction and monitoring of serum electrolyte levels. Transfusions are unnecessary.

The nurse is caring for a patient with permanent neurologic impairments resulting from a traumatic head injury. When working with this patient and family, what mutual goal should be prioritized? A) Achieve as high a level of function as possible. B) Enhance the quantity of the patient's life. C) Teach the family proper care of the patient. D) Provide community assistance.

Ans: A Feedback: The overarching goals of care are to achieve as high a level of function as possible and to enhance the quality of life for the patient with neurologic impairment and his or her family. This goal encompasses family and community participation.

Following diagnostic testing, a patient has been admitted to the ICU and placed on cerebral aneurysm precautions. What nursing action should be included in patient's plan of care? A) Supervise the patient's activities of daily living closely. B) Initiate early ambulation to prevent complications of immobility. C) Provide a high-calorie, low-protein diet. D) Perform all of the patient's hygiene and feeding.

Ans: A Feedback: The patient is placed on immediate and absolute bed rest in a quiet, nonstressful environment, because activity, pain, and anxiety elevate BP, which increases the risk for bleeding. As such, independent ADLs and ambulation are contraindicated. There is no need for a high-calorie or low-protein diet.

A patient is recovering from intracranial surgery performed approximately 24 hours ago and is complaining of a headache that the patient rates at 8 on a 10-point pain scale. What nursing action is most appropriate? A) Administer morphine sulfate as ordered. B) Reposition the patient in a prone position. C) Apply a hot pack to the patient's scalp. D) Implement distraction techniques.

Ans: A Feedback: The patient usually has a headache after a craniotomy as a result of stretching and irritation of nerves in the scalp during surgery. Morphine sulfate may also be used in the management of postoperative pain in patients who have undergone a craniotomy. Prone positioning is contraindicated due to the consequent increase in ICP. Distraction would likely be inadequate to reduce pain and a hot pack may cause vasodilation and increased pain.

The nurse is providing health education to a patient who has a C6 spinal cord injury. The patient asks why autonomic dysreflexia is considered an emergency. What would be the nurse's best answer? A) "The sudden increase in BP can raise the ICP or rupture a cerebral blood vessel." B) "The suddenness of the onset of the syndrome tells us the body is struggling to maintain its normal state." C) "Autonomic dysreflexia causes permanent damage to delicate nerve fibers that are healing." D) "The sudden, severe headache increases muscle tone and can cause further nerve damage."

Ans: A Feedback: The sudden increase in BP may cause a rupture of one or more cerebral blood vessels or lead to increased ICP. Autonomic dysreflexia does not directly cause nerve damage.

A patient is brought by ambulance to the ED after suffering what the family thinks is a stroke. The nurse caring for this patient is aware that an absolute contraindication for thrombolytic therapy is what? A) Evidence of hemorrhagic stroke B) Blood pressure of ³ 180/110 mm Hg C) Evidence of stroke evolution D) Previous thrombolytic therapy within the past 12 months

Ans: A Feedback: Thrombolytic therapy would exacerbate a hemorrhagic stroke with potentially fatal consequences. Stroke evolution, high BP, or previous thrombolytic therapy does not contraindicate its safe and effective use.

A hospital patient has experienced a seizure. In the immediate recovery period, what action best protects the patient's safety? A) Place the patient in a side-lying position. B) Pad the patient's bed rails. C) Administer antianxiety medications as ordered. D) Reassure the patient and family members.

Ans: A Feedback: To prevent complications, the patient is placed in the side-lying position to facilitate drainage of oral secretions. Suctioning is performed, if needed, to maintain a patent airway and prevent aspiration. None of the other listed actions promotes safety during the immediate recovery period.

An 82-year-old man is admitted for observation after a fall. Due to his age, the nurse knows that the patient is at increased risk for what complication of his injury? A) Hematoma B) Skull fracture C) Embolus D) Stroke

Ans: A Feedback: Two major factors place older adults at increased risk for hematomas. First, the dura becomes more adherent to the skull with increasing age. Second, many older adults take aspirin and anticoagulants as part of routine management of chronic conditions. Because of these factors, the patient's risk of a hematoma is likely greater than that of stroke, embolism, or skull fracture.

The patient in the ED has just had a diagnostic lumbar puncture. To reduce the incidence of a post-lumbar puncture headache, what is the nurse's most appropriate action? A) Position the patient prone. B) Position the patient supine with the head of bed flat. C) Position the patient left side-lying. D) Administer acetaminophen as ordered.

Ans: A The lumbar puncture headache may be avoided if a small-gauge needle is used and if the patient remains prone after the procedure. Acetaminophen is not administered as a preventative measure for post-lumbar puncture headaches.

A patient with spinal cord injury is ready to be discharged home. A family member asks the nurse to review potential complications one more time. What are the potential complications that should be monitored for in this patient? Select all that apply. A) Orthostatic hypotension B) Autonomic dysreflexia C) DVT D) Salt-wasting syndrome E) Increased ICP

Ans: A, B, C Feedback: For a spinal cord-injured patient, based on the assessment data, potential complications that may develop include DVT, orthostatic hypotension, and autonomic dysreflexia. Salt-wasting syndrome or increased ICP are not typical complications following the immediate recovery period.

The nurse is planning the care of a patient with Parkinson's disease. The nurse should be aware that treatment will focus on what pathophysiological phenomenon? A) Premature degradation of acetylcholine B) Decreased availability of dopamine C) Insufficient synthesis of epinephrine D) Delayed reuptake of serotonin

Ans: B Feedback: Parkinson's disease develops from decreased availability of dopamine, not acetylcholine, epinephrine, or serotonin.

The nurse is admitting a patient to the unit who is scheduled for removal of an intracranial mass. What diagnostic procedures might be included in this patient's admission orders? Select all that apply. A) Transcranial Doppler flow study B) Cerebral angiography C) MRI D) Cranial radiography E) Electromyelography (EMG)

Ans: A, B, C Feedback: Preoperative diagnostic procedures may include a CT scan to demonstrate the lesion and show the degree of surrounding brain edema, the ventricular size, and the displacement. An MRI scan provides information similar to that of a CT scan with improved tissue contrast, resolution, and anatomic definition. Cerebral angiography may be used to study a tumor's blood supply or to obtain information about vascular lesions. Transcranial Doppler flow studies are used to evaluate the blood flow within intracranial blood vessels. Regular x-rays of the skull would not be diagnostic for an intracranial mass. An EMG would not be ordered prior to intracranial surgery to remove a mass.

An adult patient has sought care for the treatment of headaches that have become increasingly severe and frequent over the past several months. Which of the following questions addresses potential etiological factors? Select all that apply? A) "Are you exposed to any toxins or chemicals at work?" B) "How would you describe your ability to cope with stress?" C) "What medications are you currently taking?" D) "When was the last time you were hospitalized?" E) "Does anyone else in your family struggle with headaches?"

Ans: A, B, C, E Feedback: Headaches are multifactorial, and may involve medications, exposure to toxins, family history, and stress. Hospitalization is an unlikely contributor to headaches.

The nurse caring for a patient in a persistent vegetative state is regularly assessing for potential complications. Complications of neurologic dysfunction for which the nurse should assess include which of the following? Select all that apply. A) Contractures B) Hemorrhage C) Pressure ulcers D) Venous thromboembolism E) Pneumonia

Ans: A, C, D, E Feedback: Based on the assessment data, potential complications may include respiratory distress or failure, pneumonia, aspiration, pressure ulcer, deep vein thrombosis (DVT), and contractures. The pathophysiology of decreased LOC does not normally create a heightened risk for hemorrhage.

During a patient's recovery from stroke, the nurse should be aware of predictors of stroke outcome in order to help patients and families set realistic goals. What are the predictors of stroke outcome? Select all that apply. A) National Institutes of Health Stroke Scale (NIHSS) score B) Race C) LOC at time of admission D) Gender E) Age

Ans: A, C, E Feedback: It is helpful for clinicians to be knowledgeable about the relative importance of predictors of stroke outcome (age, NIHSS score, and LOC at time of admission) to provide stroke survivors and their families with realistic goals. Race and gender are not predictors of stroke outcome.

The school nurse is giving a presentation on preventing spinal cord injuries (SCI). What should the nurse identify as prominent risk factors for SCI? Select all that apply. A) Young age B) Frequent travel C) African American race D) Male gender E) Alcohol or drug use

Ans: A, D, E Feedback: The predominant risk factors for SCI include young age, male gender, and alcohol and drug use. Ethnicity and travel are not risk factors.

The nurse is participating in the care of a patient with increased ICP. What diagnostic test is contraindicated in this patient's treatment? A) Computed tomography (CT) scan B) Lumbar puncture C) Magnetic resonance imaging (MRI) D) Venous Doppler studies

Ans: B Feedback: A lumbar puncture in a patient with increased ICP may cause the brain to herniate from the withdrawal of fluid and change in pressure during the lumbar puncture. Herniation of the brain is a dire and frequently fatal event. CT, MRI, and venous Doppler are considered noninvasive procedures and they would not affect the ICP itself.

A nurse is caring for a patient who experiences debilitating cluster headaches. The patient should be taught to take appropriate medications at what point in the course of the onset of a new headache? A) As soon as the patient's pain becomes unbearable B) As soon as the patient senses the onset of symptoms C) Twenty to 30 minutes after the onset of symptoms D) When the patient senses his or her symptoms peaking

Ans: B Feedback: A migraine or a cluster headache in the early phase requires abortive medication therapy instituted as soon as possible. Delaying medication administration would lead to unnecessary pain.

The nurse is caring for a patient who exhibits abnormal results of the Weber test and Rinne test. The nurse should suspect dysfunction involving what cranial nerve? A) Trigeminal B) Acoustic C) Hypoglossal D) Trochlear

Ans: B Feedback: Abnormal hearing can correlate with damage to cranial nerve VIII (acoustic). The acoustic nerve functions in hearing and equilibrium. The trigeminal nerve functions in facial sensation, corneal reflex, and chewing. The hypoglossal nerve moves the tongue. The trochlear nerve controls muscles that move the eye.

A clinic nurse is caring for a patient diagnosed with migraine headaches. During the patient teaching session, the patient questions the nurse regarding alcohol consumption. What would the nurse be correct in telling the patient about the effects of alcohol? A) Alcohol causes hormone fluctuations. B) Alcohol causes vasodilation of the blood vessels. C) Alcohol has an excitatory effect on the CNS. D) Alcohol diminishes endorphins in the brain.

Ans: B Feedback: Alcohol causes vasodilation of the blood vessels and may exacerbate migraine headaches. Alcohol has a depressant effect on the CNS. Alcohol does not cause hormone fluctuations, nor does it decrease endorphins (morphine-like substances produced by the body) in the brain.

When caring for a patient who had a hemorrhagic stroke, close monitoring of vital signs and neurologic changes is imperative. What is the earliest sign of deterioration in a patient with a hemorrhagic stroke of which the nurse should be aware? A) Generalized pain B) Alteration in level of consciousness (LOC) C) Tonic-clonic seizures D) Shortness of breath

Ans: B Feedback: Alteration in LOC is the earliest sign of deterioration in a patient after a hemorrhagic stroke, such as mild drowsiness, slight slurring of speech, and sluggish papillary reaction. Sudden headache may occur, but generalized pain is less common. Seizures and shortness of breath are not identified as early signs of hemorrhagic stroke.

An elderly woman found with a head injury on the floor of her home is subsequently admitted to the neurologic ICU. What is the best rationale for the following physician orders: elevate the HOB; keep the head in neutral alignment with no neck flexion or head rotation; avoid sharp hip flexion? A) To decrease cerebral arterial pressure B) To avoid impeding venous outflow C) To prevent flexion contractures D) To prevent aspiration of stomach contents

Ans: B Feedback: Any activity or position that impedes venous outflow from the head may contribute to increased volume inside the skull and possibly increase ICP. Cerebral arterial pressure will be affected by the balance between oxygen and carbon dioxide. Flexion contractures are not a priority at this time. Stomach contents could still be aspirated in this position.

A nurse is caring for a critically ill patient with autonomic dysreflexia. What clinical manifestations would the nurse expect in this patient? A) Respiratory distress and projectile vomiting B) Bradycardia and hypertension C) Tachycardia and agitation D) Third-spacing and hyperthermia

Ans: B Feedback: Autonomic dysreflexia is characterized by a pounding headache, profuse sweating, nasal congestion, piloerection ("goose bumps"), bradycardia, and hypertension. It occurs in cord lesions above T6 after spinal shock has resolved; it does not result in vomiting, tachycardia

The nurse is caring for a patient recovering from an ischemic stroke. What intervention best addresses a potential complication after an ischemic stroke? A) Providing frequent small meals rather than three larger meals B) Teaching the patient to perform deep breathing and coughing exercises C) Keeping a urinary catheter in situ for the full duration of recovery D) Limiting intake of insoluble fiber

Ans: B Feedback: Because pneumonia is a potential complication of stroke, deep breathing and coughing exercises should be encouraged unless contraindicated. No particular need exists to provide frequent meals and normally fiber intake should not be restricted. Urinary catheters should be discontinued as soon as possible

A preceptor is discussing stroke with a new nurse on the unit. The preceptor would tell the new nurse which cardiac dysrhythmia is associated with cardiogenic embolic strokes? A) Ventricular tachycardia B) Atrial fibrillation C) Supraventricular tachycardia D) Bundle branch block

Ans: B Feedback: Cardiogenic embolic strokes are associated with cardiac dysrhythmias, usually atrial fibrillation. The other listed dysrhythmias are less commonly associated with this type of stroke.

A patient has been admitted to the ICU after being recently diagnosed with an aneurysm and the patient's admission orders include specific aneurysm precautions. What nursing action will the nurse incorporate into the patient's plan of care? A) Elevate the head of the bed to 45 degrees. B) Maintain the patient on complete bed rest. C) Administer enemas when the patient is constipated. D) Avoid use of thigh-high elastic compression stockings.

Ans: B Feedback: Cerebral aneurysm precautions are implemented for the patient with a diagnosis of aneurysm to provide a nonstimulating environment, prevent increases in ICP, and prevent further bleeding. The patient is placed on immediate and absolute bed rest in a quiet, nonstressful environment because activity, pain, and anxiety elevate BP, which increases the risk for bleeding. Visitors, except for family, are restricted. The head of the bed is elevated 15 to 30 degrees to promote venous drainage and decrease ICP. Some neurologists, however, prefer that the patient remains flat to increase cerebral perfusion. No enemas are permitted, but stool softeners and mild laxatives are prescribed. Thigh-high elastic compression stockings or sequential compression boots may be ordered to decrease the patient's risk for deep vein thrombosis (DVT).

A patient has recently begun mobilizing during the recovery from an ischemic stroke. To protect the patient's safety during mobilization, the nurse should perform what action? A) Support the patient's full body weight with a waist belt during ambulation. B) Have a colleague follow the patient closely with a wheelchair. C) Avoid mobilizing the patient in the early morning or late evening. D) Ensure that the patient's family members do not participate in mobilization.

Ans: B Feedback: During mobilization, a chair or wheelchair should be readily available in case the patient suddenly becomes fatigued or feels dizzy. The family should be encouraged to participate, as appropriate, and the nurse should not have to support the patient's full body weight. Morning and evening activity are not necessarily problematic.

A nurse is caring for a patient diagnosed with a hemorrhagic stroke. When creating this patient's plan of care, what goal should be prioritized? A) Prevent complications of immobility. B) Maintain and improve cerebral tissue perfusion. C) Relieve anxiety and pain. D) Relieve sensory deprivation.

Ans: B Feedback: Each of the listed goals is appropriate in the care of a patient recovering from a stroke. However, promoting cerebral perfusion is a priority physiologic need, on which the patient's survival depends.

When caring for a patient who has had a stroke, a priority is reduction of ICP. What patient position is most consistent with this goal? A) Head turned slightly to the right side B) Elevation of the head of the bed C) Position changes every 15 minutes while awake D) Extension of the neck

Ans: B Feedback: Elevation of the head of the bed promotes venous drainage and lowers ICP; the nurse should avoid flexing or extending the neck or turning the head side to side. The head should be in a neutral midline position. Excessively frequent position changes are unnecessary.

A patient is brought to the trauma center by ambulance after sustaining a high cervical spinal cord injury 1½ hours ago. Endotracheal intubation has been deemed necessary and the nurse is preparing to assist. What nursing diagnosis should the nurse associate with this procedure? A) Risk for impaired skin integrity B) Risk for injury C) Risk for autonomic dysreflexia D) Risk for suffocation

Ans: B Feedback: If endotracheal intubation is necessary, extreme care is taken to avoid flexing or extending the patient's neck, which can result in extension of a cervical injury. Intubation does not directly cause autonomic dysreflexia and the threat to skin integrity is a not a primary concern. Intubation does not carry the potential to cause suffocation.

A neurologic flow chart is often used to document the care of a patient with a traumatic brain injury. At what point in the patient's care should the nurse begin to use a neurologic flow chart? A) When the patient's condition begins to deteriorate B) As soon as the initial assessment is made C) At the beginning of each shift D) When there is a clinically significant change in the patient's condition

Ans: B Feedback: Neurologic parameters are assessed initially and as frequently as the patient's condition requires. As soon as the initial assessment is made, the use of a neurologic flowchart is started and maintained. A new chart is not begun at the start of every shift.

Following a spinal cord injury a patient is placed in halo traction. While performing pin site care, the nurse notes that one of the traction pins has become detached. The nurse would be correct in implementing what priority nursing action? A) Complete the pin site care to decrease risk of infection. B) Notify the neurosurgeon of the occurrence. C) Stabilize the head in a lateral position. D) Reattach the pin to prevent further head trauma.

Ans: B Feedback: If one of the pins became detached, the head is stabilized in neutral position by one person while another notifies the neurosurgeon. Reattaching the pin as a nursing intervention would not be done due to risk of increased injury. Pin site care would not be a priority in this instance. Prevention of neurologic injury is the priority.

The nurse is performing a neurologic assessment of a patient whose injuries have rendered her unable to follow verbal commands. How should the nurse proceed with assessing the patient's level of consciousness (LOC)? A) Assess the patient's vital signs and correlate these with the patient's baselines. B) Assess the patient's eye opening and response to stimuli. C) Document that the patient currently lacks a level of consciousness. D) Facilitate diagnostic testing in an effort to obtain objective data.

Ans: B Feedback: If the patient is not alert or able to follow commands, the examiner observes for eye opening; verbal response and motor response to stimuli, if any; and the type of stimuli needed to obtain a response. Vital signs and diagnostic testing are appropriate, but neither will allow the nurse to gauge the patient's LOC. Inability to follow commands does not necessarily denote an absolute lack of consciousness.

A rehabilitation nurse caring for a patient who has had a stroke is approached by the patient's family and asked why the patient has to do so much for herself when she is obviously struggling. What would be the nurse's best answer? A) "We are trying to help her be as useful as she possibly can." B) "The focus on care in a rehabilitation facility is to help the patient to resume as much self-care as possible." C) "We aren't here to care for her the way the hospital staff did; we are here to help her get better so she can go home." D) "Rehabilitation means helping patients do exactly what they did before their stroke."

Ans: B Feedback: In both acute care and rehabilitation facilities, the focus is on teaching the patient to resume as much self-care as possible. The goal of rehabilitation is not to be "useful," nor is it to return patients to their prestroke level of functioning, which may be unrealistic.

While completing a health history on a patient who has recently experienced a seizure, the nurse would assess for what characteristic associated with the postictal state? A) Epileptic cry B) Confusion C) Urinary incontinence D) Body rigidity

Ans: B Feedback: In the postictal state (after the seizure), the patient is often confused and hard to arouse and may sleep for hours. The epileptic cry occurs from the simultaneous contractions of the diaphragm and chest muscles that occur during the seizure. Urinary incontinence and intense rigidity of the entire body are followed by alternating muscle relaxation and contraction (generalized tonic-clonic contraction) during the seizure.

A patient with spinal cord injury has a nursing diagnosis of altered mobility and the nurse recognizes the increased the risk of deep vein thrombosis (DVT). Which of the following would be included as an appropriate nursing intervention to prevent a DVT from occurring? A) Placing the patient on a fluid restriction as ordered B) Applying thigh-high elastic stockings C) Administering an antifibrinolyic agent D) Assisting the patient with passive range of motion (PROM) exercises

Ans: B Feedback: It is important to promote venous return to the heart and prevent venous stasis in a patient with altered mobility. Applying elastic stockings will aid in the prevention of a DVT. The patient should not be placed on fluid restriction because a dehydrated state will increase the risk of clotting throughout the body. Antifibrinolytic agents cause the blood to clot, which is absolutely contraindicated in this situation. PROM exercises are not an effective protection against the development of DVT.

A patient exhibiting an altered level of consciousness (LOC) due to blunt-force trauma to the head is admitted to the ED. The physician determines the patient's injury is causing increased intracranial pressure (ICP). The nurse should gauge the patient's LOC on the results of what diagnostic tool? A) Monro-Kellie hypothesis B) Glasgow Coma Scale C) Cranial nerve function D) Mental status examination

Ans: B Feedback: LOC, a sensitive indicator of neurologic function, is assessed based on the criteria in the Glasgow Coma Scale: eye opening, verbal response, and motor response. The Monro-Kellie hypothesis states that because of the limited space for expansion within the skull, an increase in any one of the components (blood, brain tissue, cerebrospinal fluid) causes a change in the volume of the others. Cranial nerve function and the mental status examination would be part of the neurologic examination for this patient, but would not be the priority in evaluating LOC.

The nurse is providing care for a patient who is unconscious. What nursing intervention takes highest priority? A) Maintaining accurate records of intake and output B) Maintaining a patent airway C) Inserting a nasogastric (NG) tube as ordered D) Providing appropriate pain control

Ans: B Feedback: Maintaining a patent airway always takes top priority, even though each of the other listed actions is necessary and appropriate.

The staff educator is precepting a nurse new to the critical care unit when a patient with a T2 spinal cord injury is admitted. The patient is soon exhibiting manifestations of neurogenic shock. In addition to monitoring the patient closely, what would be the nurse's most appropriate action? A) Prepare to transfuse packed red blood cells. B) Prepare for interventions to increase the patient's BP. C) Place the patient in the Trendelenberg position. D) Prepare an ice bath to lower core body temperature.

Ans: B Feedback: Manifestations of neurogenic shock include decreased BP and heart rate. Cardiac markers would be expected to rise in cardiogenic shock. Transfusion, repositioning, and ice baths are not indicated interventions.

The nurse is caring for a patient who is in status epilepticus. What medication does the nurse know may be given to halt the seizure immediately? A) Intravenous phenobarbital (Luminal) B) Intravenous diazepam (Valium) C) Oral lorazepam (Ativan) D) Oral phenytoin (Dilantin)

Ans: B Feedback: Medical management of status epilepticus includes IV diazepam (Valium) and IV lorazepam (Ativan) given slowly in an attempt to halt seizures immediately. Other medications (phenytoin, phenobarbital) are given later to maintain a seizure-free state. Oral medications are not given during status epilepticus.

The physician has ordered a somatosensory evoked responses (SERs) test for a patient for whom the nurse is caring. The nurse is justified in suspecting that this patient may have a history of what type of neurologic disorder? A) Hypothalamic disorder B) Demyelinating disease C) Brainstem deficit D) Diabetic neuropathy

Ans: B Feedback: SERs are used to detect deficits in the spinal cord or peripheral nerve conduction and to monitor spinal cord function during surgical procedures. The test is also useful in the diagnosis of demyelinating diseases, such as multiple sclerosis and polyneuropathies, where nerve conduction is slowed. The test is not done to diagnose hypothalamic disorders, brainstem deficits, or diabetic neuropathies.

A family member brings the patient to the clinic for a follow-up visit after a stroke. The family member asks the nurse what he can do to decrease his chance of having another stroke. What would be the nurse's best answer? A) "Have your heart checked regularly." B) "Stop smoking as soon as possible." C) "Get medication to bring down your sodium levels." D) "Eat a nutritious diet."

Ans: B Feedback: Smoking is a modifiable and highly significant risk factor for stroke. The significance of smoking, and the potential benefits of quitting, exceed the roles of sodium, diet, and regular medical assessments.

A school nurse is called to the playground where a 6-year-old girl has been found unresponsive and "staring into space," according to the playground supervisor. How would the nurse document the girl's activity in her chart at school? A) Generalized seizure B) Absence seizure C) Focal seizure D) Unclassified seizure

Ans: B Feedback: Staring episodes characterize an absence seizure, whereas focal seizures, generalized seizures, and unclassified seizures involve uncontrolled motor activity.

A trauma patient in the ICU has been declared brain dead. What diagnostic test is used in making the determination of brain death? A) Magnetic resonance imaging (MRI) B) Electroencephalography (EEG) C) Electromyelography (EMG) D) Computed tomography (CT

Ans: B Feedback: The EEG can be used in determining brain death. MRI, CT, and EMG are not normally used in determining brain death.

A nurse is reviewing the trend of a patient's scores on the Glasgow Coma Scale (GCS). This allows the nurse to gauge what aspect of the patient's status? A) Reflex activity B) Level of consciousness C) Cognitive ability D) Sensory involvement

Ans: B Feedback: The Glasgow Coma Scale (GCS) examines three responses related to LOC: eye opening, best verbal response, and best motor response.

The neurologic ICU nurse is admitting a patient following a craniotomy using the supratentorial approach. How should the nurse best position the patient? A) Position the patient supine. B) Maintain head of bed (HOB) elevated at 30 to 45 degrees. C) Position patient in prone position. D) Maintain bed in Trendelenberg position.

Ans: B Feedback: The patient undergoing a craniotomy with a supratentorial (above the tentorium) approach should be placed with the HOB elevated 30 to 45 degrees, with the neck in neutral alignment. Each of the other listed positions would cause a dangerous elevation in ICP.

The nurse is caring for a patient who is postoperative following a craniotomy. When writing the plan of care, the nurse identifies a diagnosis of "deficient fluid volume related to fluid restriction and osmotic diuretic use." What would be an appropriate intervention for this diagnosis? A) Change the patient's position as indicated. B) Monitor serum electrolytes. C) Maintain NPO status. D) Monitor arterial blood gas (ABG) values.

Ans: B Feedback: The postoperative fluid regimen depends on the type of neurosurgical procedure and is determined on an individual basis. The volume and composition of fluids are adjusted based on daily serum electrolyte values, along with fluid intake and output. Fluids may have to be restricted in patients with cerebral edema. Changing the patient's position, maintaining an NPO status, and monitoring ABG values do not relate to the nursing diagnosis of deficient fluid volume.

The nurse is performing stroke risk screenings at a hospital open house. The nurse has identified four patients who might be at risk for a stroke. Which patient is likely at the highest risk for a hemorrhagic stroke? A) White female, age 60, with history of excessive alcohol intake B) White male, age 60, with history of uncontrolled hypertension C) Black male, age 60, with history of diabetes D) Black male, age 50, with history of smoking

Ans: B Feedback: Uncontrolled hypertension is the primary cause of a hemorrhagic stroke. Control of hypertension, especially in individuals over 55 years of age, clearly reduces the risk for hemorrhagic stroke. Additional risk factors are increased age, male gender, and excessive alcohol intake. Another high-risk group includes African Americans, where the incidence of first stroke is almost twice that as in Caucasians.

Assessment is crucial to the care of patients with neurologic dysfunction. What does accurate and appropriate assessment require? Select all that apply. A) The ability to select mediations for the neurologic dysfunction B) Understanding of the tests used to diagnose neurologic disorders C) Knowledge of nursing interventions related to assessment and diagnostic testing D) Knowledge of the anatomy of the nervous system E) The ability to interpret the results of diagnostic tests

Ans: B, C, D Feedback: Assessment requires knowledge of the anatomy and physiology of the nervous system and an understanding of the array of tests and procedures used to diagnose neurologic disorders. Knowledge about the nursing implications and interventions related to assessment and diagnostic testing is also essential. Selecting medications and interpreting diagnostic tests are beyond the normal scope of the nurse.

The nurse educator is reviewing the assessment of cranial nerves. What should the educator identify as the specific instances when cranial nerves should be assessed? Select all that apply. A) When a neurogenic bladder develops B) When level of consciousness is decreased C) With brain stem pathology D) In the presence of peripheral nervous system disease E) When a spinal reflex is interrupted

Ans: B, C, D Feedback: Cranial nerves are assessed when level of consciousness is decreased, with brain stem pathology, or in the presence of peripheral nervous system disease. Abnormalities in muscle tone and involuntary movements are less likely to prompt the assessment of cranial nerves, since these nerves do not directly mediate most aspects of muscle tone and movement.

The nurse is caring for a patient who is rapidly progressing toward brain death. The nurse should be aware of what cardinal signs of brain death? Select all that apply. A) Absence of pain response B) Apnea C) Coma D) Absence of brain stem reflexes E) Absence of deep tendon reflexes

Ans: B, C, D Feedback: The three cardinal signs of brain death upon clinical examination are coma, the absence of brain stem reflexes, and apnea. Absences of pain response and deep tendon reflexes are not necessarily indicative of brain death.

A patient is admitted to the neurologic ICU with a C4 spinal cord injury. When writing the plan of care for this patient, which of the following nursing diagnoses would the nurse prioritize in the immediate care of this patient? A) Risk for impaired skin integrity related to immobility and sensory loss B) Impaired physical mobility related to loss of motor function C) Ineffective breathing patterns related to weakness of the intercostal muscles D) Urinary retention related to inability to void spontaneously

Ans: C Feedback: A nursing diagnosis related to breathing pattern would be the priority for this patient. A C4 spinal cord injury will require ventilatory support, due to the diaphragm and intercostals being affected. The other nursing diagnoses would be used in the care plan, but not designated as a higher priority than ineffective breathing patterns.

A patient has had an ischemic stroke and has been admitted to the medical unit. What action should the nurse perform to best prevent joint deformities? A) Place the patient in the prone position for 30 minutes/day. B) Assist the patient in acutely flexing the thigh to promote movement. C) Place a pillow in the axilla when there is limited external rotation. D) Place patient's hand in pronation.

Ans: C Feedback: A pillow in the axilla prevents adduction of the affected shoulder and keeps the arm away from the chest. The prone position with a pillow under the pelvis, not flat, promotes hyperextension of the hip joints, essential for normal gait. To promote venous return and prevent edema, the upper thigh should not be flexed acutely. The hand is placed in slight supination, not pronation, which is its most functional position.

A community health nurse is giving an educational presentation about stroke and heart disease at the local senior citizens center. What nonmodifiable risk factor for stroke should the nurse cite? A) Female gender B) Asian American race C) Advanced age D) Smoking

Ans: C Feedback: Advanced age, male gender, and race are well-known nonmodifiable risk factors for stroke. High-risk groups include people older than 55 years of age; the incidence of stroke more than doubles in each successive decade. Men have a higher rate of stroke than that of women. Another high-risk group is African Americans; the incidence of first stroke in African Americans is almost twice that as in Caucasian Americans; Asian American race is not a risk factor. Smoking is a modifiable risk.

The ED nurse is caring for a patient who has been brought in by ambulance after sustaining a fall at home. What physical assessment finding is suggestive of a basilar skull fracture? A) Epistaxis B) Periorbital edema C) Bruising over the mastoid D) Unilateral facial numbness

Ans: C Feedback: An area of ecchymosis (bruising) may be seen over the mastoid (Battle's sign) in a basilar skull fracture. Numbness, edema, and epistaxis are not directly associated with a basilar skull fracture.

A patient is brought to the ED by her family after falling off the roof. A family member tells the nurse that when the patient fell she was "knocked out," but came to and "seemed okay." Now she is complaining of a severe headache and not feeling well. The care team suspects an epidural hematoma, prompting the nurse to prepare for which priority intervention? A) Insertion of an intracranial monitoring device B) Treatment with antihypertensives C) Emergency craniotomy D) Administration of anticoagulant therapy

Ans: C Feedback: An epidural hematoma is considered an extreme emergency. Marked neurologic deficit or respiratory arrest can occur within minutes. Treatment consists of making an opening through the skull to decrease ICP emergently, remove the clot, and control the bleeding. Antihypertensive medications would not be a priority. Anticoagulant therapy should not be ordered for a patient who has a cranial bleed. This could further increase bleeding activity. Insertion of an intracranial monitoring device may be done during the surgery, but is not priority for this patient.

A patient with a new diagnosis of ischemic stroke is deemed to be a candidate for treatment with tissue plasminogen activator (t-PA) and has been admitted to the ICU. In addition to closely monitoring the patient's cardiac and neurologic status, the nurse monitors the patient for signs of what complication? A) Acute pain B) Septicemia C) Bleeding D) Seizures

Ans: C Feedback: Bleeding is the most common side effect of t-PA administration, and the patient is closely monitored for any bleeding. Septicemia, pain, and seizures are much less likely to result from thrombolytic therapy.

Following a traumatic brain injury, a patient has been in a coma for several days. Which of the following statements is true of this patient's current LOC? A) The patient occasionally makes incomprehensible sounds. B) The patient's current LOC will likely become a permanent state. C) The patient may occasionally make nonpurposeful movements. D) The patient is incapable of spontaneous respirations

Ans: C Feedback: Coma is a clinical state of unarousable unresponsiveness in which no purposeful responses to internal or external stimuli occur, although nonpurposeful responses to painful stimuli and brain stem reflexes may be present. Verbal sounds, however, are atypical. Ventilator support may or may not be necessary. Comas are not permanent states.

A patient with increased ICP has a ventriculostomy for monitoring ICP. The nurse's most recent assessment reveals that the patient is now exhibiting nuchal rigidity and photophobia. The nurse would be correct in suspecting the presence of what complication? A) Encephalitis B) CSF leak C) Meningitis D) Catheter occlusion

Ans: C Feedback: Complications of a ventriculostomy include ventricular infectious meningitis and problems with the monitoring system. Nuchal rigidity and photophobia are clinical manifestations of meningitis, but are not suggestive of encephalitis, a CSF leak, or an occluded catheter.

A patient is admitted to the neurologic ICU with a spinal cord injury. When assessing the patient the nurse notes there is a sudden depression of reflex activity in the spinal cord below the level of injury. What should the nurse suspect? A) Epidural hemorrhage B) Hypertensive emergency C) Spinal shock D) Hypovolemia

Ans: C Feedback: In spinal shock, the reflexes are absent, BP and heart rate fall, and respiratory failure can occur. Hypovolemia, hemorrhage, and hypertension do not cause this sudden change in neurologic function.

A patient had a lumbar puncture performed at the outpatient clinic and the nurse has phoned the patient and family that evening. What does this phone call enable the nurse to determine? A) What are the patient's and family's expectations of the test B) Whether the patient's family had any questions about why the test was necessary C) Whether the patient has had any complications of the test D) Whether the patient understood accurately why the test was done

Ans: C Feedback: Contacting the patient and family after diagnostic testing enables the nurse to determine whether they have any questions about the procedure or whether the patient had any untoward results. The other listed information should have been elicited from the patient and family prior to the test.

When caring for a patient with an altered level of consciousness, the nurse is preparing to test cranial nerve VII. What assessment technique would the nurse use to elicit a response from cranial nerve VII? A) Palpate trapezius muscle while patient shrugs should against resistance. B) Administer the whisper or watch-tick test. C) Observe for facial movement symmetry, such as a smile. D) Note any hoarseness in the patient's voice.

Ans: C Feedback: Cranial nerve VII is the facial nerve. An appropriate assessment technique for this cranial nerve would include observing for symmetry while the patient performs facial movements: smiles, whistles, elevates eyebrows, and frowns. Palpating and noting strength of the trapezius muscle while the patient shrugs shoulders against resistance would be completed to assess cranial nerve XI (spinal accessory). Assessing cranial nerve VIII (acoustic) would involve using the whisper or watch-tick test to evaluate hearing. Noting any hoarseness in the patient's voice would involve assessment of cranial nerve X (vagus)

The nurse is discharging home a patient who suffered a stroke. He has a flaccid right arm and leg and is experiencing problems with urinary incontinence. The nurse makes a referral to a home health nurse because of an awareness of what common patient response to a change in body image? A) Denial B) Fear C) Depression D) Disassociation

Ans: C Feedback: Depression is a common and serious problem in the patient who has had a stroke. It can result from a profound disruption in his or her life and changes in total function, leaving the patient with a loss of independence. The nurse needs to encourage the patient to verbalize feelings to assess the effect of the stroke on self-esteem. Denial, fear, and disassociation are not the most common patient response to a change in body image, although each can occur in some patients.

A patient diagnosed with a hemorrhagic stroke has been admitted to the neurologic ICU. The nurse knows that teaching for the patient and family needs to begin as soon as the patient is settled on the unit and will continue until the patient is discharged. What will family education need to include? A) How to differentiate between hemorrhagic and ischemic stroke B) Risk factors for ischemic stroke C) How to correctly modify the home environment D) Techniques for adjusting the patient's medication dosages at home

Ans: C Feedback: For a patient with a hemorrhagic stroke, teaching addresses the use of assistive devices or modification of the home environment to help the patient live with the disability. This is more important to the patient's needs than knowing about risk factors for ischemic stroke. It is not necessary for the family to differentiate between different types of strokes. Medication regimens should never be altered without consultation.

A patient has experienced a seizure in which she became rigid and then experienced alternating muscle relaxation and contraction. What type of seizure does the nurse recognize? A) Unclassified seizure B) Absence seizure C) Generalized seizure D) Focal seizure

Ans: C Feedback: Generalized seizures often involve both hemispheres of the brain, causing both sides of the body to react. Intense rigidity of the entire body may occur, followed by alternating muscle relaxation and contraction (generalized tonic-clonic contraction). This pattern of rigidity does not occur in patients who experience unclassified, absence, or focal seizures.

A patient who has been on long-term phenytoin (Dilantin) therapy is admitted to the unit. In light of the adverse of effects of this medication, the nurse should prioritize which of the following in the patient's plan of care? A) Monitoring of pulse oximetry B) Administration of a low-protein diet C) Administration of thorough oral hygiene D) Fluid restriction as ordered

Ans: C Feedback: Gingival hyperplasia (swollen and tender gums) can be associated with long-term phenytoin (Dilantin) use. Thorough oral hygiene should be provided consistently and encouraged after discharge. Fluid and protein restriction are contraindicated and there is no particular need for constant oxygen saturation monitoring

The nurse is caring for a patient with a brain tumor. What drug would the nurse expect to be ordered to reduce the edema surrounding the tumor? A) Solumedrol B) Dextromethorphan C) Dexamethasone D) Furosemide

Ans: C Feedback: If a brain tumor is the cause of the increased ICP, corticosteroids (e.g., dexamethasone) help reduce the edema surrounding the tumor. Solumedrol, a steroid, and furosemide, a loop diuretic, are not the drugs of choice in this instance. Dextromethorphan is used in cough medicines.

The nurse is caring for a patient diagnosed with an ischemic stroke and knows that effective positioning of the patient is important. Which of the following should be integrated into the patient's plan of care? A) The patient's hip joint should be maintained in a flexed position. B) The patient should be in a supine position unless ambulating. C) The patient should be placed in a prone position for 15 to 30 minutes several times a day. D) The patient should be placed in a Trendelenberg position two to three times daily to promote cerebral perfusion.

Ans: C Feedback: If possible, the patient is placed in a prone position for 15 to 30 minutes several times a day. A small pillow or a support is placed under the pelvis, extending from the level of the umbilicus to the upper third of the thigh. This helps to promote hyperextension of the hip joints, which is essential for normal gait, and helps prevent knee and hip flexion contractures. The hip joints should not be maintained in flexion and the Trendelenberg position is not indicated.

Paramedics have brought an intubated patient to the RD following a head injury due to acceleration-deceleration motor vehicle accident. Increased ICP is suspected. Appropriate nursing interventions would include which of the following? A) Keep the head of the bed (HOB) flat at all times. B) Teach the patient to perform the Valsalva maneuver. C) Administer benzodiazepines on a PRN basis. D) Perform endotracheal suctioning every hour.

Ans: C Feedback: If the patient with a brain injury is very agitated, benzodiazepines are the most commonly used sedatives and do not affect cerebral blood flow or ICP. The HOB should be elevated 30 degrees. Suctioning should be done a limited basis, due to increasing the pressure in the cranium. The Valsalva maneuver is to be avoided. This also causes increased ICP.

The nurse recognizes that a patient with a SCI is at risk for muscle spasticity. How can the nurse best prevent this complication of an SCI? A) Position the patient in a high Fowler's position when in bed. B) Support the knees with a pillow when the patient is in bed. C) Perform passive ROM exercises as ordered. D) Administer NSAIDs as ordered.

Ans: C Feedback: Passive ROM exercises can prevent muscle spasticity following SCI. NSAIDs are not used for this purpose. Pillows and sitting upright do not directly address the patient's risk of muscle spasticity.

A patient is scheduled for CT scanning of the head because of a recent onset of neurologic deficits. What should the nurse tell the patient in preparation for this test? A) "No metal objects can enter the procedure room." B) "You need to fast for 8 hours prior to the test." C) "You will need to lie still throughout the procedure." D) "There will be a lot of noise during the test."

Ans: C Feedback: Preparation for CT scanning includes teaching the patient about the need to lie quietly throughout the procedure. If the patient were having an MRI, metal and noise would be appropriate teaching topics. There is no need to fast prior to a CT scan of the brain.

The nurse is reviewing the medication administration record of a female patient who possesses numerous risk factors for stroke. Which of the woman's medications carries the greatest potential for reducing her risk of stroke? A) Naproxen 250 PO b.i.d. B) Calcium carbonate 1,000 mg PO b.i.d. C) Aspirin 81 mg PO o.d. D) Lorazepam 1 mg SL b.i.d. PRN

Ans: C Feedback: Research findings suggest that low-dose aspirin may lower the risk of stroke in women who are at risk. Naproxen, lorazepam, and calcium supplements do not have this effect.

During the performance of the Romberg test, the nurse observes that the patient sways slightly. What is the nurse's most appropriate action? A) Facilitate a referral to a neurologist. B) Reposition the patient supine to ensure safety. C) Document successful completion of the assessment. D) Follow up by having the patient perform the Rinne test.

Ans: C Feedback: Slight swaying during the Romberg test is normal, but a loss of balance is abnormal and is considered a positive Romberg test. Slight swaying is not a significant threat to the patient's safety. The Rinne test assesses hearing, not balance.

A patient is having a "fight or flight response" after receiving bad news about his prognosis. What affect will this have on the patient's sympathetic nervous system? A) Constriction of blood vessels in the heart muscle B) Constriction of bronchioles C) Increase in the secretion of sweat D) Constriction of pupils

Ans: C Feedback: Sympathetic nervous system stimulation results in dilated blood vessels in the heart and skeletal muscle, dilated bronchioles, increased secretion of sweat, and dilated pupils.

A trauma patient was admitted to the ICU with a brain injury. The patient had a change in level of consciousness, increased vital signs, and became diaphoretic and agitated. The nurse should recognize which of the following syndromes as the most plausible cause of these symptoms? A) Adrenal crisis B) Hypothalamic collapse C) Sympathetic storm D) Cranial nerve deficit

Ans: C Feedback: Sympathetic storm is a syndrome associated with changes in level of consciousness, altered vital signs, diaphoresis, and agitation that may result from hypothalamic stimulation of the sympathetic nervous system following traumatic brain injury. Alterations in cranial nerve or adrenal function would not have this result.

The nurse caring for an 80 year-old patient knows that she has a pre-existing history of dulled tactile sensation. The nurse should first consider what possible cause for this patient's diminished tactile sensation? A) Damage to cranial nerve VIII B) Adverse medication effects C) Age-related neurologic changes D) An undiagnosed cerebrovascular accident in early adulthood

Ans: C Feedback: Tactile sensation is dulled in the elderly person due to a decrease in the number of sensory receptors. While thorough assessment is necessary, it is possible that this change is unrelated to pathophysiological processes.

The nurse is caring for a patient in the ICU who has a brain stem herniation and who is exhibiting an altered level of consciousness. Monitoring reveals that the patient's mean arterial pressure (MAP) is 60 mm Hg with an intracranial pressure (ICP) reading of 5 mm Hg. What is the nurse's most appropriate action? A) Position the patient in the high Fowler's position as tolerated. B) Administer osmotic diuretics as ordered. C) Participate in interventions to increase cerebral perfusion pressure. D) Prepare the patient for craniotomy.

Ans: C Feedback: The cerebral perfusion pressure (CPP) is 55 mm Hg, which is considered low. The normal CPP is 70 to 100 mm Hg. Patients with a CPP of less than 50 mm Hg experience irreversible neurologic damage. As a result, interventions are necessary. A craniotomy is not directly indicated. Diuretics and increased height of bed would exacerbate the patient's condition.

Splints have been ordered for a patient who is at risk of developing footdrop following a spinal cord injury. The nurse caring for this patient knows that the splints are removed and reapplied when? A) At the patient's request B) Each morning and evening C) Every 2 hours D) One hour prior to mobility exercises

Ans: C Feedback: The feet are prone to footdrop; therefore, various types of splints are used to prevent footdrop. When used, the splints are removed and reapplied every 2 hours.

A patient in the OR goes into malignant hyperthermia due to an abnormal reaction to the anesthetic. The nurse knows that the area of the brain that regulates body temperature is which of the following? A) Cerebellum B) Thalamus C) Hypothalamus D) Midbrain

Ans: C Feedback: The hypothalamus plays an important role in the endocrine system because it regulates the pituitary secretion of hormones that influence metabolism, reproduction, stress response, and urine production. It works with the pituitary to maintain fluid balance through hormonal release and maintains temperature regulation by promoting vasoconstriction or vasodilatation. The cerebellum, thalamus, and midbrain and not directly involved in temperature regulation.

A patient diagnosed with transient ischemic attacks (TIAs) is scheduled for a carotid endarterectomy. The nurse explains that this procedure will be done for what purpose? A) To decrease cerebral edema B) To prevent seizure activity that is common following a TIA C) To remove atherosclerotic plaques blocking cerebral flow D) To determine the cause of the TIA

Ans: C Feedback: The main surgical procedure for select patients with TIAs is carotid endarterectomy, the removal of an atherosclerotic plaque or thrombus from the carotid artery to prevent stroke in patients with occlusive disease of the extracranial arteries. An endarterectomy does not decrease cerebral edema, prevent seizure activity, or determine the cause of a TIA.

A patient is being admitted to the neurologic ICU following an acute head injury that has resulted in cerebral edema. When planning this patient's care, the nurse would expect to administer what priority medication? A) Hydrochlorothiazide (HydroDIURIL) B) Furosemide (Lasix) C) Mannitol (Osmitrol) D) Spirolactone (Aldactone)

Ans: C Feedback: The osmotic diuretic mannitol is given to dehydrate the brain tissue and reduce cerebral edema. This drug acts by reducing the volume of brain and extracellular fluid. Spirolactone, furosemide, and hydrochlorothiazide are diuretics that are not typically used in the treatment of increased ICP resulting from cerebral edema.

The nurse is preparing health education for a patient who is being discharged after hospitalization for a hemorrhagic stroke. What content should the nurse include in this education? A) Mild, intermittent seizures can be expected. B) Take ibuprofen for complaints of a serious headache. C) Take antihypertensive medication as ordered. D) Drowsiness is normal for the first week after discharge.

Ans: C Feedback: The patient and family are provided with information that will enable them to cooperate with the care and restrictions required during the acute phase of hemorrhagic stroke and to prepare the patient to return home. Patient and family teaching includes information about the causes of hemorrhagic stroke and its possible consequences. Symptoms of hydrocephalus include gradual onset of drowsiness and behavioral changes. Hypertension is the most serious risk factor, suggesting that appropriate antihypertensive treatment is essential for a patient being discharged. Seizure activity is not normal; complaints of a serious headache should be reported to the physician before any medication is taken. Drowsiness is not normal or expected.

A patient is recovering from intracranial surgery that was performed using the transsphenoidal approach. The nurse should be aware that the patient may have required surgery on what neurologic structure? A) Cerebellum B) Hypothalamus C) Pituitary gland D) Pineal gland

Ans: C Feedback: The transsphenoidal approach (through the mouth and nasal sinuses) is often used to gain access to the pituitary gland. This surgical approach would not allow for access to the pineal gland, cerebellum, or hypothalamus.

A neurologic nurse is reviewing seizures with a group of staff nurses. How should this nurse best describe the cause of a seizure? A) Sudden electrolyte changes throughout the brain B) A dysrhythmia in the peripheral nervous system C) A dysrhythmia in the nerve cells in one section of the brain D) Sudden disruptions in the blood flow throughout the brain

Ans: C Feedback: The underlying cause of a seizure is an electrical disturbance (dysrhythmia) in the nerve cells in one section of the brain; these cells emit abnormal, recurring, uncontrolled electrical discharges. Seizures are not caused by changes in blood flow or electrolytes.

The nurse planning the care of a patient with head injuries is addressing the patient's nursing diagnosis of "sleep deprivation." What action should the nurse implement? A) Administer a benzodiazepine at bedtime each night. B) Do not disturb the patient between 2200 and 0600. C) Cluster overnight nursing activities to minimize disturbances. D) Ensure that the patient does not sleep during the day.

Ans: C Feedback: To allow the patient longer times of uninterrupted sleep and rest, the nurse can group nursing care activities so that the patient is disturbed less frequently. However, it is impractical and unsafe to provide no care for an 8-hour period. The use of benzodiazepines should be avoided.

The nurse caring for a patient with a spinal cord injury notes that the patient is exhibiting early signs and symptoms of disuse syndrome. Which of the following is the most appropriate nursing action? A) Limit the amount of assistance provided with ADLs. B) Collaborate with the physical therapist and immobilize the patient's extremities temporarily. C) Increase the frequency of ROM exercises. D) Educate the patient about the importance of frequent position changes.

Ans: C Feedback: To prevent disuse syndrome, ROM exercises must be provided at least four times a day, and care is taken to stretch the Achilles tendon with exercises. The patient is repositioned frequently and is maintained in proper body alignment whether in bed or in a wheelchair. The patient must be repositioned by caregivers, not just taught about repositioning. It is inappropriate to limit assistance for the sole purpose of preventing disuse syndrome.

A patient with a spinal cord injury has experienced several hypotensive episodes. How can the nurse best address the patient's risk for orthostatic hypotension? A) Administer an IV bolus of normal saline prior to repositioning. B) Maintain bed rest until normal BP regulation returns. C) Monitor the patient's BP before and during position changes. D) Allow the patient to initiate repositioning.

Ans: C Feedback: To prevent hypotensive episodes, close monitoring of vital signs before and during position changes is essential. Prolonged bed rest carries numerous risks and it is not possible to provide a bolus before each position change. Following the patient's lead may or may not help regulate BP.

A nurse is admitting a patient with a severe migraine headache and a history of acute coronary syndrome. What migraine medication would the nurse question for this patient? A) Rizatriptan (Maxalt) B) Naratriptan (Amerge) C) Sumatriptan succinate (Imitrex) D) Zolmitriptan (Zomig)

Ans: C Feedback: Triptans can cause chest pain and are contraindicated in patients with ischemic heart disease. Maxalt, Amerge, and Zomig are triptans used in routine clinical use for the treatment of migraine headaches.

A nurse is collaborating with the interdisciplinary team to help manage a patient's recurrent headaches. What aspect of the patient's health history should the nurse identify as a potential contributor to the patient's headaches? A) The patient leads a sedentary lifestyle. B) The patient takes vitamin D and calcium supplements. C) The patient takes vasodilators for the treatment of angina. D) The patient has a pattern of weight loss followed by weight gain.

Ans: C Feedback: Vasodilators are known to contribute to headaches. Weight fluctuations, sedentary lifestyle, and vitamin supplements are not known to have this effect.

The nurse is caring for a patient with an upper motor neuron lesion. What clinical manifestations should the nurse anticipate when planning the patient's neurologic assessment? A) Decreased muscle tone B) Flaccid paralysis C) Loss of voluntary control of movement D) Slow reflexes

Ans: C Upper motor neuron lesions do not cause muscle atrophy, flaccid paralysis, or slow reflexes. However, upper motor neuron lesions normally cause loss of voluntary control.

A patient who suffered a spinal cord injury is experiencing an exaggerated autonomic response. What aspect of the patient's current health status is most likely to have precipitated this event? A) The patient received a blood transfusion. B) The patient's analgesia regimen was recent changed. C) The patient was not repositioned during the night shift. D) The patient's urinary catheter became occluded.

Ans: D Feedback: A distended bladder is the most common cause of autonomic dysreflexia. Infrequent positioning is a less likely cause, although pressure ulcers or tactile stimulation can cause it. Changes in mediations or blood transfusions are unlikely causes.

A patient diagnosed with a cerebral aneurysm reports a severe headache to the nurse. What action is a priority for the nurse? A) Sit with the patient for a few minutes. B) Administer an analgesic. C) Inform the nurse-manager. D) Call the physician immediately.

Ans: D Feedback: A headache may be an indication that the aneurysm is leaking. The nurse should notify the physician immediately. The physician will decide whether administration of an analgesic is indicated. Informing the nurse-manager is not necessary. Sitting with the patient is appropriate, once the physician has been notified of the change in the patient's condition.

The ED is notified that a 6-year-old is in transit with a suspected brain injury after being struck by a car. The child is unresponsive at this time, but vital signs are within acceptable limits. What will be the primary goal of initial therapy? A) Promoting adequate circulation B) Treating the child's increased ICP C) Assessing secondary brain injury D) Preserving brain homeostasis

Ans: D Feedback: All therapy is directed toward preserving brain homeostasis and preventing secondary brain injury, which is injury to the brain that occurs after the original traumatic event. The scenario does not indicate the child has increased ICP or a secondary brain injury at this point. Promoting circulation is likely secondary to the broader goal of preserving brain homeostasis.

A nurse in the ICU is providing care for a patient who has been admitted with a hemorrhagic stroke. The nurse is performing frequent neurologic assessments and observes that the patient is becoming progressively more drowsy over the course of the day. What is the nurse's best response to this assessment finding? A) Report this finding to the physician as an indication of decreased metabolism. B) Provide more stimulation to the patient and monitor the patient closely. C) Recognize this as the expected clinical course of a hemorrhagic stroke. D) Report this to the physician as a possible sign of clinical deterioration.

Ans: D Feedback: Alteration in LOC often is the earliest sign of deterioration in a patient with a hemorrhagic stroke. Drowsiness and slight slurring of speech may be early signs that the LOC is deteriorating. This finding is unlikely to be the result of metabolic changes and it is not expected. Stimulating a patient with an acute stroke is usually contraindicated.

A 72-year-old man has been brought to his primary care provider by his daughter, who claims that he has been experiencing uncharacteristic lapses in memory. What principle should underlie the nurse's assessment and management of this patient? A) Loss of short-term memory is normal in older adults, but loss of long-term memory is pathologic. B) Lapses in memory in older adults are considered benign unless they have negative consequences. C) Gradual increases in confusion accompany the aging process. D) Thorough assessment is necessary because changes in cognition are always considered to be pathologic

Ans: D Feedback: Although mental processing time decreases with age, memory, language, and judgment capacities remain intact. Change in mental status should never be assumed to be a normal part of aging.

A patient with a documented history of seizure disorder experiences a generalized seizure. What nursing action is most appropriate? A) Restrain the patient to prevent injury. B) Open the patient's jaws to insert an oral airway. C) Place patient in high Fowler's position. D) Loosen the patient's restrictive clothing.

Ans: D Feedback: An appropriate nursing intervention would include loosening any restrictive clothing on the patient. No attempt should be made to restrain the patient during the seizure because muscular contractions are strong and restraint can produce injury. Do not attempt to pry open jaws that are clenched in a spasm to insert anything. Broken teeth and injury to the lips and tongue may result from such an action. If possible, place the patient on one side with head flexed forward, which allows the tongue to fall forward and facilitates drainage of saliva and mucus.

The nurse is caring for a patient with increased intracranial pressure (ICP). The patient has a nursing diagnosis of "ineffective cerebral tissue perfusion." What would be an expected outcome that the nurse would document for this diagnosis? A) Copes with sensory deprivation. B) Registers normal body temperature. C) Pays attention to grooming. D) Obeys commands with appropriate motor responses.

Ans: D Feedback: An expected outcome of the diagnosis of ineffective cerebral tissue perfusion in a patient with increased intracranial pressure (ICP) would include obeying commands with appropriate motor responses. Vitals signs and neurologic status are assessed every 15 minutes to every hour. Coping with sensory deprivation would relate to the nursing diagnosis of "disturbed sensory perception." The outcome of "registers normal body temperature" relates to the diagnosis of "potential for ineffective thermoregulation." Body image disturbance would have a potential outcome of "pays attention to grooming."

The nurse is caring for a patient who has undergone supratentorial removal of a pituitary mass. What medication would the nurse expect to administer prophylactically to prevent seizures in this patient? A) Prednisone B) Dexamethasone C) Cafergot D) Phenytoin

Ans: D Feedback: Antiseizure medication (phenytoin, diazepam) is often prescribed prophylactically for patients who have undergone supratentorial craniotomy because of the high risk of seizures after this procedure. Prednisone and dexamethasone are steroids and do not prevent seizures. Cafergot is used in the treatment of migraines.

A patient has a poor prognosis after being involved in a motor vehicle accident resulting in a head injury. As the patient's ICP increases and condition worsens, the nurse knows to assess for indications of approaching death. These indications include which of the following? A) Hemiplegia B) Dry mucous membranes C) Signs of internal bleeding D) Loss of brain stem reflexes

Ans: D Feedback: Loss of brain stem reflexes, including pupillary, corneal, gag, and swallowing reflexes, is an ominous sign of approaching death. Dry mucous membranes, hemiplegia, and bleeding must be promptly addressed, but none of these is a common sign of impending death.

The nurse is providing care for a patient who is withdrawing from heavy alcohol use. The nurse and other members of the care team are present at the bedside when the patient has a seizure. In preparation for documenting this clinical event, the nurse should note which of the following? A) The ability of the patient to follow instructions during the seizure. B) The success or failure of the care team to physically restrain the patient. C) The patient's ability to explain his seizure during the postictal period. D) The patient's activities immediately prior to the seizure.

Ans: D Feedback: Before and during a seizure, the nurse observes the circumstances before the seizure, including visual, auditory, or olfactory stimuli; tactile stimuli; emotional or psychological disturbances; sleep; and hyperventilation. Communication with the patient is not possible during a seizure and physical restraint is not attempted. The patient's ability to explain the seizure is not clinically relevant.

The neurologic nurse is testing the function of a patient's cerebellum and basal ganglia. What action will most accurately test these structures? A) Have the patient identify the location of a cotton swab on his or her skin with the eyes closed. B) Elicit the patient's response to a hypothetical problem. C) Ask the patient to close his or her eyes and discern between hot and cold stimuli. D) Guide the patient through the performance of rapid, alternating movements.

Ans: D Feedback: Cerebellar and basal ganglia influence on the motor system is reflected in balance control and coordination. Coordination in the hands and upper extremities is tested by having the patient perform rapid, alternating movements and point-to-point testing. The cerebellum and basal ganglia do not mediate cutaneous sensation or judgment.

A nurse is caring for a patient diagnosed with Ménière's disease. While completing a neurologic examination on the patient, the nurse assesses cranial nerve VIII. The nurse would be correct in identifying the function of this nerve as what? A) Movement of the tongue B) Visual acuity C) Sense of smell D) Hearing and equilibrium

Ans: D Feedback: Cranial nerve VIII (acoustic) is responsible for hearing and equilibrium. Cranial nerve XII (hypoglossal) is responsible for movement of the tongue. Cranial nerve II (optic) is responsible for visual acuity and visual fields. Cranial nerve I (olfactory) functions in sense of smell.

The nursing students are learning how to assess function of cranial nerve VIII. To assess the function of cranial nerve VIII the students would be correct in completing which of the following assessment techniques? A) Have the patient identify familiar odors with the eyes closed. B) Assess papillary reflex. C) Utilize the Snellen chart. D) Test for air and bone conduction (Rinne test).

Ans: D Feedback: Cranial nerve VIII is the acoustic nerve. It functions in hearing and equilibrium. When assessing this nerve, the nurse would test for air and bone conduction (Rinne) with a tuning fork. Assessment of papillary reflex would be completed for cranial nerves III (oculomotor), IV (trochlear), and VI (abducens). The Snellen chart would be used to assess cranial nerve II (optic).

What should the nurse suspect when hourly assessment of urine output on a patient postcraniotomy exhibits a urine output from a catheter of 1,500 mL for two consecutive hours? A) Cushing syndrome B) Syndrome of inappropriate antidiuretic hormone (SIADH) C) Adrenal crisis D) Diabetes insipidus

Ans: D Feedback: Diabetes insipidus is an abrupt onset of extreme polyuria that commonly occurs in patients after brain surgery. Cushing syndrome is excessive glucocorticoid secretion resulting in sodium and water retention. SIADH is the result of increased secretion of ADH; the patient becomes volume-overloaded, urine output diminishes, and serum sodium concentration becomes dilute. Adrenal crisis is undersecretion of glucocorticoids resulting in profound hypoglycemia, hypovolemia, and hypotension.

The nurse has implemented interventions aimed at facilitating family coping in the care of a patient with a traumatic brain injury. How can the nurse best facilitate family coping? A) Help the family understand that the patient could have died. B) Emphasize the importance of accepting the patient's new limitations. C) Have the members of the family plan the patient's inpatient care. D) Assist the family in setting appropriate short-term goals.

Ans: D Feedback: Helpful interventions to facilitate coping include providing family members with accurate and honest information and encouraging them to continue to set well-defined, short-term goals. Stating that a patient's condition could be worse downplays their concerns. Emphasizing the importance of acceptance may not necessarily help the family accept the patient's condition. Family members cannot normally plan a patient's hospital care, although they may contribute to the care in some ways.

A 13-year-old was brought to the ED, unconscious, after being hit in the head by a baseball. When the child regains consciousness, 5 hours after being admitted, he cannot remember the traumatic event. MRI shows no structural sign of injury. What injury would the nurse suspect the patient has? A) Diffuse axonal injury B) Grade 1 concussion with frontal lobe involvement C) Contusion D) Grade 3 concussion with temporal lobe involvement

Ans: D Feedback: In a grade 3 concussion there is a loss of consciousness lasting from seconds to minutes. Temporal lobe involvement results in amnesia. Frontal lobe involvement can cause uncharacteristic behavior and a grade 1 concussion does not involve loss of consciousness. Diagnostic studies may show no apparent structural sign of injury, but the duration of unconsciousness is an indicator of the severity of the concussion. Diffuse axonal injury (DAI) results from widespread shearing and rotational forces that produce damage throughout the brain—to axons in the cerebral hemispheres, corpus callosum, and brain stem. In cerebral contusion, a moderate to severe head injury, the brain is bruised and damaged in a specific area because of severe acceleration-deceleration force or blunt trauma.

A patient for whom the nurse is caring has positron emission tomography (PET) scheduled. In preparation, what should the nurse explain to the patient? A) The test will temporarily limit blood flow through the brain. B) An allergy to iodine precludes getting the radio-opaque dye. C) The patient will need to endure loud noises during the test. D) The test may result in dizziness or lightheadedness.

Ans: D Feedback: Key nursing interventions for PET scan include explaining the test and teaching the patient about inhalation techniques and the sensations (e.g., dizziness, light-headedness, and headache) that may occur. A PET scan does not impede blood flow through the brain. An allergy to iodine precludes the dye for an MRI, and loud noise is heard in an MRI.

A nurse on the neurologic unit is providing care for a patient who has spinal cord injury at the level of C4. When planning the patient's care, what aspect of the patient's neurologic and functional status should the nurse consider? A) The patient will be unable to use a wheelchair. B) The patient will be unable to swallow food. C) The patient will be continent of urine, but incontinent of bowel. D) The patient will require full assistance for all aspects of elimination

Ans: D Feedback: Patients with a lesion at C4 are fully dependent for elimination. The patient is dependent for feeding, but is able to swallow. The patient will be capable of using an electric wheelchair.

During the examination of an unconscious patient, the nurse observes that the patient's pupils are fixed and dilated. What is the most plausible clinical significance of the nurse's finding? A) It suggests onset of metabolic problems. B) It indicates paralysis on the right side of the body. C) It indicates paralysis of cranial nerve X. D) It indicates an injury at the midbrain level.

Ans: D Feedback: Pupils that are fixed and dilated indicate injury at the midbrain level. This finding is not suggestive of unilateral paralysis, metabolic deficits, or damage to CN X.

The public health nurse is planning a health promotion campaign that reflects current epidemiologic trends. The nurse should know that hemorrhagic stroke currently accounts for what percentage of total strokes in the United States? A) 43% B) 33% C) 23% D) 13%

Ans: D Feedback: Strokes can be divided into two major categories: ischemic (87%), in which vascular occlusion and significant hypoperfusion occur, and hemorrhagic (13%), in which there is extravasation of blood into the brain or subarachnoid space.

What should be included in the patient's care plan when establishing an exercise program for a patient affected by a stroke? A) Schedule passive range of motion every other day. B) Keep activity limited, as the patient may be over stimulated. C) Have the patient perform active range-of-motion (ROM) exercises once a day. D) Exercise the affected extremities passively four or five times a day.

Ans: D Feedback: The affected extremities are exercised passively and put through a full ROM four or five times a day to maintain joint mobility, regain motor control, prevent development of a contracture in the paralyzed extremity, prevent further deterioration of the neuromuscular system, and enhance circulation. Active ROM exercises should ideally be performed more than once per day.

The nurse is conducting a focused neurologic assessment. When assessing the patient's cranial nerve function, the nurse would include which of the following assessments? A) Assessment of hand grip B) Assessment of orientation to person, time, and place C) Assessment of arm drift D) Assessment of gag reflex

Ans: D Feedback: The gag reflex is governed by the glossopharyngeal nerve, one of the cranial nerves. Hand grip and arm drifting are part of motor function assessment. Orientation is an assessment parameter related to a mental status examination.

An ED nurse has just received a call from EMS that they are transporting a 17-year-old man who has just sustained a spinal cord injury (SCI). The nurse recognizes that the most common cause of this type of injury is what? A) Sports-related injuries B) Acts of violence C) Injuries due to a fall D) Motor vehicle accidents

Ans: D Feedback: The most common causes of SCIs are motor vehicle crashes (46%), falls (22%), violence (16%), and sports (12%).

A patient is currently being stimulated by the parasympathetic nervous system. What effect will this nervous stimulation have on the patient's bladder? A) The parasympathetic nervous system causes urinary retention. B) The parasympathetic nervous system causes bladder spasms. C) The parasympathetic nervous system causes urge incontinence. D) The parasympathetic nervous system makes the bladder contract.

Ans: D Feedback: The parasympathetic division of the nervous system causes contraction (stimulation) of the urinary bladder muscles and a decrease (inhibition) in heart rate, whereas the sympathetic division produces relaxation (inhibition) of the urinary bladder and an increase (stimulation) in the rate and force of the heartbeat.

A patient with a cerebral aneurysm exhibits signs and symptoms of an increase in intracranial pressure (ICP). What nursing intervention would be most appropriate for this patient? A) Range-of-motion exercises to prevent contractures B) Encouraging independence with ADLs to promote recovery C) Early initiation of physical therapy D) Absolute bed rest in a quiet, nonstimulating environment

Ans: D Feedback: The patient is placed on immediate and absolute bed rest in a quiet, nonstressful environment because activity, pain, and anxiety elevate BP, which increases the risk for bleeding. Visitors are restricted. The nurse administers all personal care. The patient is fed and bathed to prevent any exertion that might raise BP.

After a subarachnoid hemorrhage, the patient's laboratory results indicate a serum sodium level of less than 126 mEq/L. What is the nurse's most appropriate action? A) Administer a bolus of normal saline as ordered. B) Prepare the patient for thrombolytic therapy as ordered. C) Facilitate testing for hypothalamic dysfunction. D) Prepare to administer 3% NaCl by IV as ordered.

Ans: D Feedback: The patient may be experiencing syndrome of inappropriate antidiuretic hormone (SIADH) or cerebral salt-wasting syndrome. The treatment most often is the use of IV hypertonic 3% saline. A normal saline bolus would exacerbate the problem and there is no indication for tests of hypothalamic function or thrombolytic therapy.

A nursing student is writing a care plan for a newly admitted patient who has been diagnosed with a stroke. What major nursing diagnosis should most likely be included in the patient's plan of care? A) Adult failure to thrive B) Post-trauma syndrome C) Hyperthermia D) Disturbed sensory perception

Ans: D Feedback: The patient who has experienced a stroke is at a high risk for disturbed sensory perception. Stroke is associated with multiple other nursing diagnoses, but hyperthermia, adult failure to thrive, and post-trauma syndrome are not among these.

18. The nurse is working with a patient who has been experiencing episodes of urinary retention. What assessment finding would suggest that the patient is experiencing retention? A) The patient's suprapubic region is dull on percussion. B) The patient is uncharacteristically drowsy. C) The patient claims to void large amounts of urine 2 to 3 times daily. D) The patient takes a beta adrenergic blocker for the treatment of hypertension.

Ans: A Feedback: Dullness on percussion of the suprapubic region is suggestive of urinary retention. Patients retaining urine are typically restless, not drowsy. A patient experiencing retention usually voids frequent, small amounts of urine and the use of beta-blockers is unrelated to urinary retention.

A female patient is diagnosed with a right-sided stroke. The patient is now experiencing hemianopsia. How might the nurse help the patient manage her potential sensory and perceptional difficulties? A) Keep the lighting in the patient's room low. B) Place the patient's clock on the affected side. C) Approach the patient on the side where vision is impaired. D) Place the patient's extremities where she can see them.

Ans: D Feedback: The patient with homonymous hemianopsia (loss of half of the visual field) turns away from the affected side of the body and tends to neglect that side and the space on that side; this is called amorphosynthesis. In such instances, the patient cannot see food on half of the tray, and only half of the room is visible. It is important for the nurse to remind the patient constantly of the other side of the body, to maintain alignment of the extremities, and if possible, to place the extremities where the patient can see them. Patients with a decreased field of vision should be approached on the side where visual perception is intact. All visual stimuli (clock, calendar, and television) should be placed on this side. The patient can be taught to turn the head in the direction of the defective visual field to compensate for this loss. Increasing the natural or artificial lighting in the room and providing eyeglasses are important in increasing vision. There is no reason to keep the lights dim.

A patient recovering from a stroke has severe shoulder pain from subluxation of the shoulder and is being cared for on the unit. To prevent further injury and pain, the nurse caring for this patient is aware of what principle of care? A) The patient should be fitted with a cast because use of a sling should be avoided due to adduction of the affected shoulder. B) Elevation of the arm and hand can lead to further complications associated with edema. C) Passively exercising the affected extremity is avoided in order to minimize pain. D) The patient should be taught to interlace fingers, place palms together, and slowly bring scapulae forward to avoid excessive force to shoulder.

Ans: D Feedback: To prevent shoulder pain, the nurse should never lift a patient by the flaccid shoulder or pull on the affected arm or shoulder. The patient is taught how to move and exercise the affected arm/shoulder through proper movement and positioning. The patient is instructed to interlace the fingers, place the palms together, and push the clasped hands slowly forward to bring the scapulae forward; he or she then raises both hands above the head. This is repeated throughout the day. The use of a properly worn sling when the patient is out of bed prevents the paralyzed upper extremity from dangling without support. Range-of-motion exercises are still vitally important in preventing a frozen shoulder and ultimately atrophy of subcutaneous tissues, which can cause more pain. Elevation of the arm and hand is also important in preventing dependent edema of the hand.

A patient with a head injury has been increasingly agitated and the nurse has consequently identified a risk for injury. What is the nurse's best intervention for preventing injury? A) Restrain the patient as ordered. B) Administer opioids PRN as ordered. C) Arrange for friends and family members to sit with the patient. D) Pad the side rails of the patient's bed.

Ans: D Feedback: To protect the patient from self-injury, the nurse uses padded side rails. The nurse should avoid restraints, because straining against them can increase ICP or cause other injury. Narcotics used to control restless patients should be avoided because these medications can depress respiration, constrict the pupils, and alter the patient's responsiveness. Visitors should be limited if the patient is agitated.

The nurse is admitting a patient to the unit who is diagnosed with a lower motor neuron lesion. What entry in the patient's electronic record is most consistent with this diagnosis? A) "Patient exhibits increased muscle tone." B) "Patient demonstrates normal muscle structure with no evidence of atrophy." C) "Patient demonstrates hyperactive deep tendon reflexes." D) "Patient demonstrates an absence of deep tendon reflexes."

Ans: D Lower motor neuron lesions cause flaccid muscle paralysis, muscle atrophy, decreased muscle tone, and loss of voluntary control.

An elderly patient is being discharged home. The patient lives alone and has atrophy of his olfactory organs. The nurse tells the patient's family that it is essential that the patient have what installed in the home? A) Grab bars B) Nonslip mats C) Baseboard heaters D) A smoke detector

Ans: D The sense of smell deteriorates with age. The olfactory organs are responsible for smell. This may present a safety hazard for the patient because he or she may not smell smoke or gas leaks. Smoke detectors are universally necessary, but especially for this patient.

What term is used to describe the fibrous connective tissue that hugs the brain closely and extends into every fold of the brain's surface? A) Dura mater B) Arachnoid C) Fascia D) Pia mater

Ans: D The term "meninges" describes the fibrous connective tissue that covers the brain and spinal cord. The meninges have three layers, the dura mater, arachnoid, and pia mater. The pia mater is the innermost membrane that hugs the brain closely and extends into every fold of the brain's surface. The dura mater, the outermost layer, covers the brain and spinal cord. The arachnoid, the middle membrane, is responsible for the production of cerebrospinal fluid.

12. A patient with cancer of the bladder has just returned to the unit from the PACU after surgery to create an ileal conduit. The nurse is monitoring the patient's urine output hourly and notifies the physician when the hourly output is less than what? A) 30 mL B) 50 mL C) 100 mL D) 125 mL

Ans: A Feedback: A urine output below 30 mL/hr may indicate dehydration or an obstruction in the ileal conduit, with possible backflow or leakage from the ureteroileal anastomosis.

39. The nurse has tested the pH of urine from a patient's newly created ileal conduit and obtained a result of 6.8. What is the nurse's best response to this assessment finding? A) Obtain an order to increase the patient's dose of ascorbic acid. B) Administer IV sodium bicarbonate as ordered. C) Encourage the patient to drink at least 500 mL of water and retest in 3 hours. D) Irrigate the ileal conduit with a dilute citric acid solution as ordered.

Ans: A Feedback: Because severe alkaline encrustation can accumulate rapidly around the stoma, the urine pH is kept below 6.5 by administration of ascorbic acid by mouth. An increased pH may suggest a need to increase ascorbic acid dosing. This is not treated by administering bicarbonate or citric acid, nor by increasing fluid intake.

19. A patient with kidney stones is scheduled for extracorporeal shock wave lithotripsy (ESWL). What should the nurse include in the patient's post-procedure care? A) Strain the patient's urine following the procedure. B) Administer a bolus of 500 mL normal saline following the procedure. C) Monitor the patient for fluid overload following the procedure. D) Insert a urinary catheter for 24 to 48 hours after the procedure.

Ans: A Feedback: Following ESWL, the nurse should strain the patient's urine for gravel or sand. There is no need to administer an IV bolus after the procedure and there is not a heightened risk of fluid overload. Catheter insertion is not normally indicated following ESWL.

21. A patient is undergoing diagnostic testing for a suspected urinary obstruction. The nurse should know that incomplete emptying of the bladder due to bladder outlet obstruction can cause what? A) Hydronephrosis B) Nephritic syndrome C) Pylonephritis D) Nephrotoxicity

Ans: A Feedback: If voiding dysfunction goes undetected and untreated, the upper urinary system may become compromised. Chronic incomplete bladder emptying from poor detrusor pressure results in recurrent bladder infection. Incomplete bladder emptying due to bladder outlet obstruction, causing high-pressure detrusor contractions, can result in hydronephrosis from the high detrusor pressure that radiates up the ureters to the renal pelvis. This problem does not normally cause nephritic syndrome or pyelonephritis. Nephrotoxicity results from chemical causes.

37. The nurse who is leading a wellness workshop has been asked about actions to reduce the risk of bladder cancer. What health promotion action most directly addresses a major risk factor for bladder cancer? A) Smoking cessation B) Reduction of alcohol intake C) Maintenance of a diet high in vitamins and nutrients D) Vitamin D supplementation

Ans: A Feedback: People who smoke develop bladder cancer twice as often as those who do not smoke. High alcohol intake and low vitamin intake are not noted to contribute to bladder cancer.

5. The nurse on a urology unit is working with a patient who has been diagnosed with oxalate renal calculi. When planning this patient's health education, what nutritional guidelines should the nurse provide? A) Restrict protein intake as ordered. B) Increase intake of potassium-rich foods. C) Follow a low-calcium diet. D) Encourage intake of food containing oxalates.

Ans: A Feedback: Protein is restricted to 60 g/d, while sodium is restricted to 3 to 4 g/d. Low-calcium diets are generally not recommended except for true absorptive hypercalciuria. The patient should avoid intake of oxalate-containing foods and there is no need to increase potassium intake.

2. A 42-year-old woman comes to the clinic complaining of occasional urinary incontinence when she sneezes. The clinic nurse should recognize what type of incontinence? A) Stress incontinence B) Reflex incontinence C) Overflow incontinence D) Functional incontinence

Ans: A Feedback: Stress incontinence is the involuntary loss of urine through an intact urethra as a result of sudden increase in intra-abdominal pressure. Reflex incontinence is loss of urine due to hyperreflexia or involuntary urethral relaxation in the absence of normal sensations usually associated with voiding. Overflow incontinence is an involuntary urine loss associated with overdistension of the bladder. Functional incontinence refers to those instances in which the function of the lower urinary tract is intact, but other factors (outside the urinary system) make it difficult or impossible for the patient to reach the toilet in time for voiding.

9. The nurse and urologist have both been unsuccessful in catheterizing a patient with a prostatic obstruction and a full bladder. What approach does the nurse anticipate the physician using to drain the patient's bladder? A) Insertion of a suprapubic catheter B) Scheduling the patient immediately for a prostatectomy C) Application of warm compresses to the perineum to assist with relaxation D) Medication administration to relax the bladder muscles and reattempting catheterization in 6 hours

Ans: A Feedback: When the patient cannot void, catheterization is used to prevent overdistention of the bladder. In the case of prostatic obstruction, attempts at catheterization by the urologist may not be successful, requiring insertion of a suprapubic catheter. A prostatectomy may be necessary, but would not be undertaken for the sole purpose of relieving a urethral obstruction. Delaying by applying compresses or administering medications could result in harm.

25. A female patient has been prescribed a course of antibiotics for the treatment of a UTI. When providing health education for the patient, the nurse should address what topic? A) The risk of developing a vaginal yeast infection as a consequent of antibiotic therapy B) The need to expect a heavy menstrual period following the course of antibiotics C) The risk of developing antibiotic resistance after the course of antibiotics D) The need to undergo a series of three urine cultures after the antibiotics have been completed

Ans: A Feedback: Yeast vaginitis occurs in as many as 25% of patients treated with antimicrobial agents that affect vaginal flora. Yeast vaginitis can cause more symptoms and be more difficult and costly to treat than the original UTI. Antibiotics do not affect menstrual periods and serial urine cultures are not normally necessary. Resistance is normally a result of failing to complete a prescribed course of antibiotics.

22. The nurse is assessing a patient admitted with renal stones. During the admission assessment, what parameters would be priorities for the nurse to address? Select all that apply. A) Dietary history B) Family history of renal stones C) Medication history D) Surgical history E) Vaccination history

Ans: A, B, C Feedback: Dietary and medication histories and family history of renal stones are obtained to identify factors predisposing the patient to stone formation. When caring for a patient with renal stones it would not normally be a priority to assess the vaccination history or surgical history, since these factors are not usually related to the etiology of kidney stones.

The nurse is doing an initial assessment on a patient newly admitted to the unit with a diagnosis of cerebrovascular accident (CVA). The patient has difficulty copying a figure that the nurse has drawn and is diagnosed with visual-receptive aphasia. What brain region is primarily involved in this deficit? A) Temporal lobe B) Parietal-occipital area C) Inferior posterior frontal areas D) Posterior frontal area

Ans: B Difficulty copying a figure that the nurse has drawn would be considered visual-receptive aphasia, which involves the parietal-occipital area. Expressive aphasia, the inability to express oneself, is often associated with damage to the frontal area. Receptive aphasia, the inability to understand what someone else is saying, is often associated with damage to the temporal lobe area.

A patient scheduled for magnetic resonance imaging (MRI) has arrived at the radiology department. The nurse who prepares the patient for the MRI should prioritize which of the following actions? A) Withholding stimulants 24 to 48 hours prior to exam B) Removing all metal-containing objects C) Instructing the patient to void prior to the MRI D) Initiating an IV line for administration of contrast

Ans: B Patient preparation for an MRI consists of removing all metal-containing objects prior to the examination. Withholding stimulants would not affect an MRI; this relates to an electroencephalography (EEG). Instructing the patient to void is patient preparation for a lumbar puncture. Initiating an IV line for administration of contrast would be done if the patient was having a CT scan with contrast.

A gerontologic nurse planning the neurologic assessment of an older adult is considering normal, age-related changes. Of what phenomenon should the nurse be aware? A) Hyperactive deep tendon reflexes B) Reduction in cerebral blood flow C) Increased cerebral metabolism D) Hypersensitivity to painful stimuli

Ans: B Reduction in cerebral blood flow (CBF) is a change that occurs in the normal aging process. Deep tendon reflexes can be decreased or, in some cases, absent. Cerebral metabolism decreases as the patient advances in age. Reaction to painful stimuli may be decreased with age. Because pain is an important warning signal, caution must be used when hot or cold packs are used.

A patient is brought to the ER following a motor vehicle accident in which he sustained head trauma. Preliminary assessment reveals a vision deficit in the patient's left eye. The nurse should associate this abnormal finding with trauma to which of the following cerebral lobes? A)Temporal B)Occipital C)Parietal D)Frontal

Ans: B The posterior lobe of the cerebral hemisphere is responsible for visual interpretation. The temporal lobe contains the auditory receptive areas. The parietal lobe contains the primary sensory cortex, and is essential to an individual's awareness of the body in space, as well as orientation in space and spatial relations. The frontal lobe functions in concentration, abstract thought, information storage or memory, and motor function.

The nurse has admitted a new patient to the unit. One of the patient's admitting orders is for an adrenergic medication. The nurse knows that this medication will have what effect on the circulatory system? A) Thin, watery saliva B) Increased heart rate C) Decreased BP D) Constricted bronchioles

Ans: B The term "adrenergic" refers to the sympathetic nervous system. Sympathetic effects include an increased rate and force of the heartbeat. Cholinergic effects, which correspond to the parasympathetic division of the autonomic nervous system, include thin, watery saliva, decreased rate and force of heartbeat, and decreased BP.

42. A patient has undergone the creation of an Indiana pouch for the treatment of bladder cancer. The nurse identified the nursing diagnosis of disturbed body image. How can the nurse best address the effects of this urinary diversion on the patient's body image? A) Emphasize that the diversion is an integral part of successful cancer treatment. B) Encourage the patient to speak openly and frankly about the diversion. C) Allow the patient to initiate the process of providing care for the diversion. D) Provide the patient with detailed written materials about the diversion at the time of discharge.

Ans: B Feedback: Allowing the patient to express concerns and anxious feelings can help with body image, especially in adjusting to the changes in toileting habits. The nurse may have to initiate dialogue about the management of the diversion, especially if the patient is hesitant. Provision of educational materials is rarely sufficient to address a sudden change and profound change in body image. Emphasizing the role of the diversion in cancer treatment does not directly address the patient's body image.

30. A patient with a sacral pressure ulcer has had a urinary catheter inserted. As a result of this new intervention, the nurse should prioritize what nursing diagnosis in the patient's plan of care? A) Impaired physical mobility related to presence of an indwelling urinary catheter B) Risk for infection related to presence of an indwelling urinary catheter C) Toileting self-care deficit related to urinary catheterization D) Disturbed body image related to urinary catheterization

Ans: B Feedback: Catheters create a high risk for UTIs. Because of this acute physiologic threat, the patient's risk for infection is usually prioritized over functional and psychosocial diagnoses.

10. The nurse has implemented a bladder retraining program for an older adult patient. The nurse places the patient on a timed voiding schedule and performs an ultrasonic bladder scan after each void. The nurse notes that the patient typically has approximately 50 mL of urine remaining in her bladder after voiding. What would be the nurse's best response to this finding? A) Perform a straight catheterization on this patient. B) Avoid further interventions at this time, as this is an acceptable finding. C) Place an indwelling urinary catheter. D) Press on the patient's bladder in an attempt to encourage complete emptying.

Ans: B Feedback: In adults older than 60 years of age, 50 to 100 mL of residual urine may remain after each voiding because of the decreased contractility of the detrusor muscle. Consequently, further interventions are not likely warranted.

35. A patient has been successfully treated for kidney stones and is preparing for discharge. The nurse recognizes the risk of recurrence and has planned the patient's discharge education accordingly. What preventative measure should the nurse encourage the patient to adopt? A) Increasing intake of protein from plant sources B) Increasing fluid intake C) Adopting a high-calcium diet D) Eating several small meals each day

Ans: B Feedback: Increased fluid intake is encouraged to prevent the recurrence of kidney stones. Protein intake from all sources should be limited. Most patients do not require a low-calcium diet, but increased calcium intake would be contraindicated for all patients. Eating small, frequent meals does not influence the risk for recurrence.

32. A nurse on a busy medical unit provides care for many patients who require indwelling urinary catheters at some point during their hospital care. The nurse should recognize a heightened risk of injury associated with indwelling catheter use in which patient? A) A patient whose diagnosis of chronic kidney disease requires a fluid restriction B) A patient who has Alzheimer's disease and who is acutely agitated C) A patient who is on bed rest following a recent episode of venous thromboembolism D) A patient who has decreased mobility following a transmetatarsal amputation

Ans: B Feedback: Patients who are confused and agitated risk trauma through the removal of an indwelling catheter which has the balloon still inflated. Recent VTE, amputation, and fluid restriction do not directly create a risk for injury or trauma associated with indwelling catheter use.

8. The clinic nurse is preparing a plan of care for a patient with a history of stress incontinence. What role will the nurse have in implementing a behavioral therapy approach? A) Provide medication teaching related to pseudoephedrine sulfate. B) Teach the patient to perform pelvic floor muscle exercises. C) Prepare the patient for an anterior vaginal repair procedure. D) Provide information on periurethral bulking.

Ans: B Feedback: Pelvic floor muscle exercises (sometimes called Kegel exercises) represent the cornerstone of behavioral intervention for addressing symptoms of stress, urge, and mixed incontinence. None of the other listed interventions has a behavioral approach.

14. The nurse is teaching a health class about UTIs to a group of older adults. What characteristic of UTIs should the nurse cite? A) Men over age 65 are equally prone to UTIs as women, but are more often asymptomatic. B) The prevalence of UTIs in men older than 50 years of age approaches that of women in the same age group. C) Men of all ages are less prone to UTIs, but typically experience more severe symptoms. D) The prevalence of UTIs in men cannot be reliably measured, as men generally do not report UTIs.

Ans: B Feedback: The antibacterial activity of the prostatic secretions that protect men from bacterial colonization of the urethra and bladder decreases with aging. The prevalence of infection in men older than 50 years of age approaches that of women in the same age group. Men are not more likely to be asymptomatic and are not known to be reluctant to report UTIs.

38. Resection of a patient's bladder tumor has been incomplete and the patient is preparing for the administration of the first ordered instillation of topical chemotherapy. When preparing the patient, the nurse should emphasize the need to do which of the following? A) Remain NPO for 12 hours prior to the treatment. B) Hold the solution in the bladder for 2 hours before voiding. C) Drink the intravesical solution quickly and on an empty stomach. D) Avoid acidic foods and beverages until the full cycle of treatment is complete.

Ans: B Feedback: The patient is allowed to eat and drink before the instillation procedure. Once the bladder is full, the patient must retain the intravesical solution for 2 hours before voiding. The solution is instilled through the meatus; it is not consumed orally. There is no need to avoid acidic foods and beverages during treatment.

3. A nurse is caring for a female patient whose urinary retention has not responded to conservative treatment. When educating this patient about self-catheterization, the nurse should encourage what practice? A) Assuming a supine position for self-catheterization B) Using clean technique at home to catheterize C) Inserting the catheter 1 to 2 inches into the urethra D) Self-catheterizing every 2 hours at home

Ans: B Feedback: The patient may use a "clean" (nonsterile) technique at home, where the risk of cross-contamination is reduced. The average daytime clean intermittent catheterization schedule is every 4 to 6 hours and just before bedtime. The female patient assumes a Fowler's position and uses a mirror to help locate the urinary meatus. The nurse teaches her to catheterize herself by inserting a catheter 7.5 cm (3 inches) into the urethra, in a downward and backward direction.

4. A 52-year-old patient is scheduled to undergo ileal conduit surgery. When planning this patient's discharge education, what is the most plausible nursing diagnosis that the nurse should address? A) Impaired mobility related to limitations posed by the ileal conduit B) Deficient knowledge related to care of the ileal conduit C) Risk for deficient fluid volume related to urinary diversion D) Risk for autonomic dysreflexia related to disruption of the sacral plexus

Ans: B Feedback: The patient will most likely require extensive teaching about the care and maintenance of a new urinary diversion. A diversion does not create a serious risk of fluid volume deficit. Mobility is unlikely to be impaired after the immediate postsurgical recovery. The sacral plexus is not threatened by the creation of a urinary diversion.

11. The nurse is caring for a patient recently diagnosed with renal calculi. The nurse should instruct the patient to increase fluid intake to a level where the patient produces at least how much urine each day? A) 1,250 mL B) 2,000 mL C) 2,750 mL D) 3,500 mL

Ans: B Feedback: Unless contraindicated by renal failure or hydronephrosis, patients with renal stones should drink at least eight 8-ounce glasses of water daily or have IV fluids prescribed to keep the urine dilute. A urine output exceeding 2 L a day is advisable.

23. A nurse who provides care in a long-term care facility is aware of the high incidence and prevalence of urinary tract infections among older adults. What action has the greatest potential to prevent UTIs in this population? A) Administer prophylactic antibiotics as ordered. B) Limit the use of indwelling urinary catheters. C) Encourage frequent mobility and repositioning. D) Toilet residents who are immobile on a scheduled basis.

Ans: B Feedback: When indwelling catheters are used, the risk of UTI increases dramatically. Limiting their use significantly reduces an older adult's risk of developing a UTI. Regular toileting promotes continence, but has only an indirect effect on the risk of UTIs. Prophylactic antibiotics are not normally administered. Mobility does not have a direct effect on UTI risk.

A patient with a new diagnosis of ischemic stroke is deemed to be a candidate treatment with tissue plasminogen activator (t-PA) and has been admitted to t addition to closely monitoring the patient's cardiac and neurologic status, the monitors the patient for signs of what complication? A) Acute pain B) Septicemia C) Bleeding D) Seizures

Ans: C Feedback: Bleeding is the most common side effect of t-PA administration, and the patie closely monitored for any bleeding. Septicemia, pain, and seizures are much l to result from thrombolytic therapy.

A nurse is assessing reflexes in a patient with hyperactive reflexes. When the patient's foot is abruptly dorsiflexed, it continues to "beat" two to three times before settling into a resting position. How would the nurse document this finding? A) Rigidity B) Flaccidity C) Clonus D) Ataxia

Ans: C When reflexes are very hyperactive, a phenomenon called clonus may be elicited. If the foot is abruptly dorsiflexed, it may continue to "beat" two to three times before it settles into a position of rest. Rigidity is an increase in muscle tone at rest characterized by increased resistance to passive stretch. Flaccidity is lack of muscle tone. Ataxia is the inability to coordinate muscle movements, resulting in difficulty walking, talking, and performing self-care activities.

20. The nurse is caring for a patient who has undergone creation of a urinary diversion. Forty-eight hours postoperatively, the nurse's assessment reveals that the stoma is a dark purplish color. What is the nurse's most appropriate response? A) Document the presence of a healthy stoma. B) Assess the patient for further signs and symptoms of infection. C) Inform the primary care provider that the vascular supply may be compromised. D) Liaise with the wound-ostomy-continence (WOC) nurse because the ostomy appliance around the stoma may be too loose.

Ans: C Feedback: A healthy stoma is pink or red. A change from this normal color to a dark purplish color suggests that the vascular supply may be compromised. A loose ostomy appliance and infections do not cause a dark purplish stoma.

36. A patient who has recently undergone ESWL for the treatment of renal calculi has phoned the urology unit where he was treated, telling the nurse that he has a temperature of 101.1ºF (38.4ºC). How should the nurse best respond to the patient? A) Remind the patient that renal calculi have a noninfectious etiology and that a fever is unrelated to their recurrence. B) Remind the patient that occasional febrile episodes are expected following ESWL. C) Tell the patient to report to the ED for further assessment. D) Tell the patient to monitor his temperature for the next 24 hours and then contact his urologist's office.

Ans: C Feedback: Following ESWL, the development of a fever is abnormal and is suggestive of a UTI; prompt medical assessment and treatment are warranted. It would be inappropriate to delay further treatment.

31. A patient has had her indwelling urinary catheter removed after having it in place for 10 days during recovery from an acute illness. Two hours after removal of the catheter, the patient informs the nurse that she is experiencing urinary urgency resulting in several small-volume voids. What is the nurse's best response? A) Inform the patient that urgency and occasional incontinence are expected for the first few weeks post-removal. B) Obtain an order for a loop diuretic in order to enhance urine output and bladder function. C) Inform the patient that this is not unexpected in the short term and scan the patient's bladder following each void. D) Obtain an order to reinsert the patient's urinary catheter and attempt removal in 24 to 48 hours.

Ans: C Feedback: Immediately after the indwelling catheter is removed, the patient is placed on a timed voiding schedule, usually every 2 to 3 hours. At the given time interval, the patient is instructed to void. The bladder is then scanned using a portable ultrasonic bladder scanner; if the bladder has not emptied completely, straight catheterization may be performed. An indwelling catheter would not be reinserted to resolve the problem and diuretics would not be beneficial. Ongoing incontinence is not an expected finding after catheter removal.

28. A nurse is working with a female patient who has developed stress urinary incontinence. Pelvic floor muscle exercises have been prescribed by the primary care provider. How can the nurse best promote successful treatment? A) Clearly explain the potential benefits of pelvic floor muscle exercises. B) Ensure the patient knows that surgery will be required if the exercises are unsuccessful. C) Arrange for biofeedback when the patient is learning to perform the exercises. D) Contact the patient weekly to ensure that she is performing the exercises consistently.

Ans: C Feedback: Research shows that written or verbal instruction alone is usually inadequate to teach an individual how to identify and strengthen the pelvic floor for sufficient bladder and bowel control. Biofeedback-assisted pelvic muscle exercise (PME) uses either electromyography or manometry to help the individual identify the pelvic muscles as he or she attempts to learn which muscle group is involved when performing PME. This objective assessment is likely superior to weekly contact with the patient. Surgery is not necessarily indicated if behavioral techniques are unsuccessful.

15. A patient has been admitted to the postsurgical unit following the creation of an ileal conduit. What should the nurse measure to determine the size of the appliance needed? A) The circumference of the stoma B) The narrowest part of the stoma C) The widest part of the stoma D) Half the width of the stoma

Ans: C Feedback: The correct appliance size is determined by measuring the widest part of the stoma with a ruler. The permanent appliance should be no more than 1.6 mm (1/8 inch) larger than the diameter of the stoma and the same shape as the stoma to prevent contact of the skin with drainage.

1. A female patient has been experiencing recurrent urinary tract infections. What health education should the nurse provide to this patient? A) Bathe daily and keep the perineal region clean. B) Avoid voiding immediately after sexual intercourse. C) Drink liberal amounts of fluids. D) Void at least every 6 to 8 hours.

Ans: C Feedback: The patient is encouraged to drink liberal amounts of fluids (water is the best choice) to increase urine production and flow, which flushes the bacteria from the urinary tract. Frequent voiding (every 2 to 3 hours) is encouraged to empty the bladder completely because this can significantly lower urine bacterial counts, reduce urinary stasis, and prevent reinfection. The patient should be encouraged to shower rather than bathe.

6. The nurse is caring for a patient who underwent percutaneous lithotripsy earlier in the day. What instruction should the nurse give the patient? A) Limit oral fluid intake for 1 to 2 days. B) Report the presence of fine, sand like particles through the nephrostomy tube. C) Notify the physician about cloudy or foul-smelling urine. D) Report any pink-tinged urine within 24 hours after the procedure.

Ans: C Feedback: The patient should report the presence of foul-smelling or cloudy urine since this is suggestive of a UTI. Unless contraindicated, the patient should be instructed to drink large quantities of fluid each day to flush the kidneys. Sand like debris is normal due to residual stone products. Hematuria is common after lithotripsy.

34. A patient with a recent history of nephrolithiasis has presented to the ED. After determining that the patient's cardiopulmonary status is stable, what aspect of care should the nurse prioritize? A) IV fluid administration B) Insertion of an indwelling urinary catheter C) Pain management D) Assisting with aspiration of the stone

Ans: C Feedback: The patient with kidney stones is often in excruciating pain, and this is a high priority for nursing interventions. In the short term, this would supersede the patient's need for IV fluids or for catheterization. Kidney stones cannot be aspirated.

13. The nurse is caring for a patient with an indwelling urinary catheter. The nurse is aware that what nursing action helps prevent infection in a patient with an indwelling catheter? A) Vigorously clean the meatus area daily. B) Apply powder to the perineal area twice daily. C) Empty the drainage bag at least every 8 hours. D) Irrigate the catheter every 8 hours with normal saline.

Ans: C Feedback: To reduce the risk of bacterial proliferation, the nurse should empty the collection bag at least every 8 hours through the drainage spout, and more frequently if there is a large volume of urine. Vigorous cleaning of the meatus while the catheter is in place is discouraged, because the cleaning action can move the catheter, increasing the risk of infection. The spout (or drainage port) of any urinary drainage bag can become contaminated when opened to drain the bag. Irrigation of the catheter opens the closed system, increasing the likelihood of infection.

33. A patient has been admitted to the medical unit with a diagnosis of ureteral colic secondary to urolithiasis. When planning the patient's admission assessment, the nurse should be aware of the signs and symptoms that are characteristic of this diagnosis? Select all that apply. A) Diarrhea B) High fever C) Hematuria D) Urinary frequency E) Acute pain

Ans: C, D, E Feedback: Stones lodged in the ureter (ureteral obstruction) cause acute, excruciating, colicky, wavelike pain, radiating down the thigh and to the genitalia. Often, the patient has a desire to void, but little urine is passed, and it usually contains blood because of the abrasive action of the stone. This group of symptoms is called ureteral colic. Diarrhea is not associated with this presentation and a fever is usually absent due to the noninfectious nature of the health problem.

40. A patient is postoperative day 3 following the creation of an ileal conduit for the treatment of invasive bladder cancer. The patient is quickly learning to self-manage the urinary diversion, but expresses concern about the presence of mucus in the urine. What is the nurse's most appropriate response? A) Report this finding promptly to the primary care provider. B) Obtain a sterile urine sample and send it for culture. C) Obtain a urine sample and check it for pH. D) Reassure the patient that this is an expected phenomenon.

Ans: D Feedback: Because mucous membrane is used in forming the conduit, the patient may excrete a large amount of mucus mixed with urine. This causes anxiety in many patients. To help relieve this anxiety, the nurse reassures the patient that this is a normal occurrence after an ileal conduit procedure. Urine testing for culture or pH is not required.

7. A female patient's most recent urinalysis results are suggestive of bacteriuria. When assessing this patient, the nurse's data analysis should be informed by what principle? A) Most UTIs in female patients are caused by viruses and do not cause obvious symptoms. B) A diagnosis of bacteriuria requires three consecutive positive results. C) Urine contains varying levels of healthy bacterial flora. D) Urine samples are frequently contaminated by bacteria normally present in the urethral area.

Ans: D Feedback: Because urine samples (especially in women) are commonly contaminated by the bacteria normally present in the urethral area, a bacterial count exceeding 105colonies/mL of clean-catch, midstream urine is the measure that distinguishes true bacteriuria from contamination. A diagnosis does not require three consecutive positive results and urine does not contain a normal flora in the absence of a UTI. Most UTIs have a bacterial etiology.

27. An older adult has experienced a new onset of urinary incontinence and family members identify this problem as being unprecedented. When assessing the patient for factors that may have contributed to incontinence, the nurse should prioritize what assessment? A) Reviewing the patient's 24-hour food recall for changes in diet B) Assessing for recent contact with individuals who have UTIs C) Assessing for changes in the patient's level of psychosocial stress D) Reviewing the patient's medication administration record for recent changes

Ans: D Feedback: Many medications affect urinary continence in addition to causing other unwanted or unexpected effects. Stress and dietary changes could potentially affect the patient's continence, but medications are more frequently causative of incontinence. UTIs can cause incontinence, but these infections do not result from contact with infected individuals.

17. A nurse's colleague has applied an incontinence pad to an older adult patient who has experienced occasional episodes of functional incontinence. What principle should guide the nurse's management of urinary incontinence in older adults? A) Diuretics should be promptly discontinued when an older adult experiences incontinence. B) Restricting fluid intake is recommended for older adults experiencing incontinence. C) Urinary catheterization is a first-line treatment for incontinence in older adults with incontinence. D) Urinary incontinence is not considered a normal consequence of aging.

Ans: D Feedback: Nursing management is based on the premise that incontinence is not inevitable with illness or aging and that it is often reversible and treatable. Diuretics cannot always be safely discontinued. Fluid restriction and catheterization are not considered to be safe, first-line interventions for the treatment of incontinence.

16. A patient being treated in the hospital has been experiencing occasional urinary retention. What nursing action should the nurse take to encourage a patient who is having difficulty voiding? A) Use a slipper bedpan. B) Apply a cold compress to the perineum. C) Have the patient lie in a supine position. D) Provide privacy for the patient.

Ans: D Feedback: Nursing measures to encourage normal voiding patterns include providing privacy, ensuring an environment and body position conducive to voiding, and assisting the patient with the use of the bathroom or bedside commode, rather than a bedpan, to provide a more natural setting for voiding. Most people find supine positioning not conducive to voiding.

A nurse is caring for a patient who has been admitted for the treatment of advanced cirrhosis. What assessment should the nurse prioritize in this patient's plan of care? A. Measurement of abdominal girth and body weight B. Assessment for variceal bleeding C. Assessment for signs and symptoms of jaundice D. Monitoring of results of liver function testing

B. Assessment of variceal bleeding Rationale: Esophageal varices are a major cause of mortality in patients with uncompensated cirrhosis. Consequently, this should be a focus of the nurse's assessments and should be prioritized over the other listed assessments, even though each should be performed.

A patient has had a laparoscopic cholecystectomy. The patient is now complaining of right shoulder pain. What should the nurse suggest to relieve the pain? A) Aspirin every 4 to 6 hours as ordered B) Application of heat 15 to 20 minutes each hour C) Application of an ice pack for no more than 15 minutes D) Application of liniment rub to affected area

B

A patient is admitted to the unit with acute cholecystitis. The physician has noted that surgery will be scheduled in 4 days. The patient asks why the surgery is being put off for a week when he has a sick gallbladder. What rationale would underlie the nurses response? A) Surgery is delayed until the patient can eat a regular diet without vomiting. B) Surgery is delayed until the acute symptoms subside. C) The patient requires aggressive nutritional support prior to surgery. D) Time is needed to determine whether a laparoscopic procedure can be used.

B

A patient is scheduled for a diagnostic MRI of the lower urinary system. What pre-procedure education should the nurse include? A) The need to be NPO for 12 hours prior to the test B) Relaxation techniques to apply during the test C) The need for conscious sedation prior to the test D) The need to limit fluid intake to 1 liter in the 24 hours before the test

B

A patient with a history of progressively worsening fatigue is undergoing a comprehensive assessment which includes test of renal function relating to erythropoiesis. When assessing the oxygen transport ability of the blood, the nurse should prioritize the review of what blood value? A) Hematocrit B) Hemoglobin C) Erythrocyte sedimentation rate (ESR) D) Serum creatinine

B

A patient with recurrent urinary tract infections has just undergone a cystoscopy and complains of slight hematuria during the first void after the procedure. What is the nurse's most appropriate action? A) Administer a STAT dose of vitamin K, as ordered. B) Reassure the patient that this is not unexpected and then monitor the patient for further bleeding. C) Promptly inform the physician of this assessment finding. D) Position the patient supine and insert a Foley catheter, as ordered.

B

The nurse is caring for a patient scheduled for renal angiography following a motor vehicle accident. What patient preparation should the nurse most likely provide before this test? A) Administration of IV potassium chloride B) Administration of a laxative C) Administration of Gastrografin D) Administration of a 24-hour urine test

B

The nurse is caring for a patient who describes changes in his voiding patterns. The patient states, "I feel the urge to empty my bladder several times an hour and when the urge hits me I have to get to the restroom quickly. But when I empty my bladder, there doesn't seem to be a great deal of urine flow." What would the nurse expect this patient's physical assessment to reveal? A) Hematuria B) Urine retention C) Dehydration D) Renal failure

B

The nurse is caring for a patient who has just returned from the ERCP removal of gallstones. The nurse should monitor the patient for signs of what complications? A) Pain and peritonitis B) Bleeding and perforation C) Acidosis and hypoglycemia D) Gangrene of the gallbladder and hyperglycemia

B

The critical care nurse is monitoring the patients urine output and drains following renal surgery. What should the nurse promptly report to the physician? A) Increased pain on movement B) Absence of drain output C) Increased urine output D) Blood-tinged serosanguineous drain output

B) Absence of drain output Urine output and drainage from tubes inserted during surgery are monitored for amount, color, and type or characteristics. Decreased or absent drainage is promptly reported to the physician because it may indicate obstruction that could cause pain, infection, and disruption of the suture lines. Reporting increased pain on movement has nothing to do with the scenario described. Increased urine output and serosanguineous drainage are expected.

The nurse is caring for a patient with acute glomerular inflammation. When assessing for the characteristic signs and symptoms of this health problem, the nurse should include which assessments? Select all that apply. A) Percuss for pain in the right lower abdominal quadrant. B) Assess for the presence of peripheral edema. C) Auscultate the patients apical heart rate for dysrhythmias. D) Assess the patients BP. E) Assess the patients orientation and judgment.

B) Assess for the presence of peripheral edema. D) Assess the patients BP. Most patients with acute glomerular inflammation have some degree of edema and hypertension. Dysrhythmias, RLQ pain, and changes in mental status are not among the most common manifestations of acute glomerular inflammation.

A patient is being treated for AKI and the patient daily weights have been ordered. The nurse notes a weight gain of 3 pounds over the past 48 hours. What nursing diagnosis is suggested by this assessment finding? A) Imbalanced nutrition: More than body requirements B) Excess fluid volume C) Sedentary lifestyle D) Adult failure to thrive

B) Excess fluid volume If the patient with AKI gains or does not lose weight, fluid retention should be suspected. Short-term weight gain is not associated with excessive caloric intake or a sedentary lifestyle. Failure to thrive is not associated with weight gain.

A nurse is performing an admission assessment for an 81-year-old patient who generally enjoys good health. When considering normal, age-related changes to hepatic function, the nurse should anticipate what finding? A. Similar liver size and texture as in younger adults B. A nonpalpable liver C. A slightly enlarged liver with palpably hard edges D. A slightly decreased size of the liver

D. A slightly decreased size of the liver Rationale: The most common age-related change in the liver is a decrease in size and weight. The liver is usually still palpable, however, it is not expected to have hardened edges.

A 71-year-old patient with ESKD has been told by the physician that it is time to consider hemodialysis until a transplant can be found. The patient tells the nurse she is not sure she wants to undergo a kidney transplant. What would be an appropriate response for the nurse to make? A) The decision is certainly yours to make, but be sure not to make a mistake. B) Kidney transplants in patients your age are as successful as they are in younger patients. C) I understand your hesitancy to commit to a transplant surgery. Success is comparatively rare. D) Have you talked this over with your family?

B) Kidney transplants in patients your age are as successful as they are in younger patients. Although there is no specific age limitation for renal transplantation, concomitant disorders (e.g., coronary artery disease, peripheral vascular disease) have made it a less common treatment for the elderly. However, the outcome is comparable to that of younger patients. The other listed options either belittle the patient or give the patient misinformation.

A patient on the critical care unit is postoperative day 1 following kidney transplantation from a living donor. The nurses most recent assessments indicate that the patient is producing copious quantities of dilute urine. What is the nurses most appropriate response? A) Assess the patient for further signs or symptoms of rejection. B) Recognize this as an expected finding. C) Inform the primary care provider of this finding. D) Administer exogenous antidiuretic hormone as ordered.

B) Recognize this as an expected finding. A kidney from a living donor related to the patient usually begins to function immediately after surgery and may produce large quantities of dilute urine. This is not suggestive of rejection and treatment is not warranted. There is no obvious need to report this finding.

A patient with chronic kidney disease is completing an exchange during peritoneal dialysis. The nurse observes that the peritoneal fluid is draining slowly and that the patients abdomen is increasing in girth. What is the nurses most appropriate action? A) Advance the catheter 2 to 4 cm further into the peritoneal cavity. B) Reposition the patient to facilitate drainage. C) Aspirate from the catheter using a 60-mL syringe. D) Infuse 50 mL of additional dialysate.

B) Reposition the patient to facilitate drainage. If the peritoneal fluid does not drain properly, the nurse can facilitate drainage by turning the patient from side to side or raising the head of the bed. The catheter should never be pushed further into the peritoneal cavity. It would be unsafe to aspirate or to infuse more dialysate.

A patient on the medical unit has a documented history of polycystic kidney disease (PKD). What principle should guide the nurses care of this patient? A) The disease is self-limiting and cysts usually resolve spontaneously in the fifth or sixth decade of life. B) The patients disease is incurable and the nurses interventions will be supportive. C) The patient will eventually require surgical removal of his or her renal cysts. D) The patient is likely to respond favorably to lithotripsy treatment of the cysts.

B) The patients disease is incurable and the nurses interventions will be supportive. PKD is incurable and care focuses on support and symptom control. It is not self-limiting and is not treated surgically or with lithotripsy.

A nurse is caring for a patient with impaired renal function. A creatinine clearance measurement has been ordered. The nurse should facilitate collection of what samples?

B) A 24-hour urine specimen and a serum creatinine level midway through the urine collection process

A patient is scheduled for diagnostic testing to address prolonged signs and symptoms of genitourinary dysfunction. What signs and symptoms are particularly suggestive of urinary tract disease? Select all that apply. A) Petechiae B) Pain C) Gastrointestinal symptoms D) Changes in voiding E) Jaundice

B,C,D

Dipstick testing of an older adult patient's urine indicates the presence of protein. Which of the following statements is true of this assessment finding? A) This finding needs to be considered in light of other forms of testing. B) This finding is a risk factor for urinary incontinence. C) This finding is likely the result of an age-related physiologic change. D) This result confirms that the patient has diabetes. Select all that apply.

B,C,D

A patient with a history of injection drug use has been diagnosed with Hepatitis C. When collaborating with the care team to plan this patient's treatment, the nurse should anticipate what intervention? A. Administration of immune globulins B. A regimen of antiviral medications C. Rest and watchful waiting D. Administration of fresh-frozen plasma (FFP)

B. A regimen of antiviral medications Rationale: There is no benefit from rest, diet, or vitamin supplements in HCV treatment. Studies have demonstrated that a combination of two antiviral agents, Peg-interferon and ribavirin (Rebetol), is effective in producing improvement in patients with hepatitis C and in treating relapses. Immune globulins and FFP are not indicated.

A patient has been diagnosed with advanced stage breast cancer and will soon begin aggressive treatment. What assessment findings would most strongly suggest that the patient may have developed liver metastases? A. Persistent fever and cognitive changes B. Abdominal pain and hepatomegaly C. Peripheral edema unresponsive to diuresis D. Spontaneous bleeding and jaundice

B. Abdominal pain and hepatomegaly Rationale: The early manifestations of malignancy of the liver include pain, a continuous dull ache in the right upper quadrant, epigastrium, or back. Weight loss, loss of strength, anorexia, and anemia may also occur. The liver may be enlarged and irregular on palpation. Jaundice is present only if the larger bile ducts are occluded by the pressure of malignant nodules in the hilum of the liver. Fever, cognitive changes, peripheral edema, and bleeding are atypical signs.

A participant in a health fair has asked the nurse about the role of drugs in liver disease. What health promotion teaching has the most potential to prevent drug-induced hepatitis? A. Finish all prescribed courses of antibiotics, regardless of symptoms resolution. B. Adhere to dosing recommendations of OTC analgesics. C. Ensure that expired medications are disposed of safely. D. Ensure that pharmacists regularly review drug regimens for potential interactions.

B. Adhere to dosing recommendations of OTC analgesics. Rationale: Although any medication can affect liver function, use of acetaminophen (found in many OTC medications used to treat fever and pain) has been identified as the leading cause of acute liver failure. Finishing prescribed antibiotics and avoiding expired medications are unrelated to this disease. Drug interactions are rarely the cause of drug-induced hepatitis.

A patient with cirrhosis has experienced a progressive decline in his health; and liver transplantation is being considered by the interdisciplinary team. How will the patient's prioritization for receiving a donor liver be determined? A. By considering that patient's age and prognosis B. By objectively determining the patient's medical need C. By objectively assessing the patient's willingness to adhere to post-transplantation care D. By systematically ruling out alternative treatment options

B. By objectively determining the patient's medical need Rationale: The patient would undergo a classification of the degree of medical need through an objective determination known as the Model of End-Stage Liver Disease (MELD) classification, which stratifies the level of illness of those awaiting a liver transplant. This algorithm considers multiple variables, not solely age, prognosis, potential for adherence and the rejection of alternative options.

A patient has undergone a laparoscopic cholecystectomy and is being prepared for discharge home. When providing health education, the nurse should prioritize which of the following topics? A) Management of fluid balance in the home setting B) The need for blood glucose monitoring for the next week C) Signs and symptoms of intra-abdominal complications D) Appropriate use of prescribed pancreatic enzymes

C

A patient is admitted to the ICU with acute pancreatitis. The patients family asks what causes acute pancreatitis. The critical care nurse knows that a majority of patients with acute pancreatitis have what? A) Type 1 diabetes B) An impaired immune system C) Undiagnosed chronic pancreatitis D) An amylase deficiency

C

A patient is complaining of genitourinary pain shortly after returning to the unit from a scheduled cystoscopy. What intervention should the nurse perform? A) Encourage mobilization. B) Apply topical lidocaine to the patient's meatus, as ordered. C) Apply moist heat to the patient's lower abdomen. D) Apply an ice pack to the patient's perineum.

C

A patient is receiving care in the intensive care unit for acute pancreatitis. The nurse is aware that pancreatic necrosis is a major cause of morbidity and mortality in patients with acute pancreatitis. Consequently, the nurse should assess for what signs or symptoms of this complication? A) Sudden increase in random blood glucose readings B) Increased abdominal girth accompanied by decreased level of consciousness C) Fever, increased heart rate and decreased blood pressure D) Abdominal pain unresponsive to analgesics

C

A patient who had surgery for gallbladder disease has just returned to the postsurgical unit from postanesthetic recovery. The nurse caring for this patient knows to immediately report what assessment finding to the physician? A) Decreased breath sounds B) Drainage of bile-colored fluid onto the abdominal dressing C) Rigidity of the abdomen D) Acute pain with movement

C

A student nurse is caring for a patient who has a diagnosis of acute pancreatitis and who is receiving parenteral nutrition. The student should prioritize which of the following assessments? A) Fluid output B) Oral intake C) Blood glucose levels D) BUN and creatinine levels

C

Diagnostic testing of an adult patient reveals renal glycosuria. The nurse should recognize the need for the patient to be assessed for what health problem? A) Diabetes insipidus B) Syndrome of inappropriate antidiuretic hormone secretion (SIADH) C) Diabetes mellitus D) Renal carcinoma

C

Results of a patient's 24-hour urine sample indicate osmolality of 510 mOsm/kg, which is within reference range. What conclusion can the nurse draw from this assessment finding? A) The patient's kidneys are capable of maintaining acid-base balance. B) The patient's kidneys reabsorb most of the potassium that the patient ingests. C) The patient's kidneys can produce sufficiently concentrated urine. D) The patient's kidneys are producing sufficient erythropoietin.

C

The family of a patient in the ICU diagnosed with acute pancreatitis asks the nurse why the patient has been moved to an air bed. What would be the nurses best response? A) Air beds allow the care team to reposition her more easily while shes on bed rest. B) Air beds are far more comfortable than regular beds and shell likely have to be on bed rest a long time. C) The bed automatically moves, so shes less likely to develop pressure sores while shes in bed. D) The bed automatically moves, so she is likely to have less pain.

C

The nurse is caring for a patient suspected of having renal dysfunction. When reviewing laboratory results for this patient, the nurse interprets the presence of which substances in the urine as most suggestive of pathology? A) Potassium and sodium B) Bicarbonate and urea C) Glucose and protein D) Creatinine and chloride

C

The nurse is caring for a patient with a nursing diagnosis of deficient fluid volume. The nurse's assessment reveals a BP of 98/52 mm Hg. The nurse should recognize that the patient's kidneys will compensate by secreting what substance? A) Antidiuretic hormone (ADH) B) Aldosterone C) Renin D) Angiotensin

C

The nurse is performing a focused genitourinary and renal assessment of a patient. Where should the nurse assess for pain at the costovertebral angle? A) At the umbilicus and the right lower quadrant of the abdomen B) At the suprapubic region and the umbilicus C) At the lower border of the 12th rib and the spine D) At the 7th rib and the xyphoid process

C

An 84-year-old woman diagnosed with cancer is admitted to the oncology unit for surgical treatment. The patient has been on chemotherapeutic agents to decrease the tumor size prior to the planned surgery. The nurse caring for the patient is aware that what precipitating factors in this patient may contribute to AKI? Select all that apply. A) Anxiety B) Low BMI C) Age-related physiologic changes D) Chronic systemic disease E) NPO status

C) Age-related physiologic changes D) Chronic systemic disease Changes in kidney function with normal aging increase the susceptibility of elderly patients to kidney dysfunction and renal failure. In addition, the presence of chronic, systemic diseases increases the risk of AKI. Low BMI and anxiety are not risk factors for acute renal disease. NPO status is not a risk, provided adequate parenteral hydration is administered.

A patient is admitted to the ICU after a motor vehicle accident. On the second day of the hospital admission, the patient develops acute kidney injury. The patient is hemodynamically unstable, but renal replacement therapy is needed to manage the patients hypervolemia and hyperkalemia. Which of the following therapies will the patients hemodynamic status best tolerate? A) Hemodialysis B) Peritoneal dialysis C) Continuous venovenous hemodialysis (CVVHD) D) Plasmapheresis

C) Continuous venovenous hemodialysis (CVVHD) CVVHD facilitates the removal of uremic toxins and fluid. The hemodynamic effects of CVVHD are usually mild in comparison to hemodialysis, so CVVHD is best tolerated by an unstable patient. Peritoneal dialysis is not the best choice, as the patient may have sustained abdominal injuries during the accident and catheter placement would be risky. Plasmapheresis does not achieve fluid removal and electrolyte balance.

A nurse is caring for a patient who is in the diuresis phase of AKI. The nurse should closely monitor the patient for what complication during this phase? A) Hypokalemia B) Hypocalcemia C) Dehydration D) Acute flank pain

C) Dehydration The diuresis period is marked by a gradual increase in urine output, which signals that glomerular filtration has started to recover. The patient must be observed closely for dehydration during this phase; if dehydration occurs, the uremic symptoms are likely to increase. Excessive losses of potassium and calcium are not typical during this phase, and diuresis does not normally result in pain.

The nurse is creating an education plan for a patient who underwent a nephrectomy for the treatment of a renal tumor. What should the nurse include in the teaching plan? A) The importance of increased fluid intake B) Signs and symptoms of rejection C) Inspection and care of the incision D) Techniques for preventing metastasis

C) Inspection and care of the incision The nurse teaches the patient to inspect and care for the incision and perform other general postoperative care, including activity and lifting restrictions, driving, and pain management. There would be no need to teach the signs or symptoms of rejection as there has been no transplant. Increased fluid intake is not normally recommended and the patient has minimal control on the future risk for metastasis.

The nurse is caring for a patient after kidney surgery. The nurse is aware that bleeding is a major complication of kidney surgery and that if it goes undetected and untreated can result in hypovolemia and hemorrhagic shock in the patient. When assessing for bleeding, what assessment parameter should the nurse evaluate? A) Oral intake B) Pain intensity C) Level of consciousness D) Radiation of pain

C) Level of consciousness Bleeding is a major complication of kidney surgery. If undetected and untreated, this can result in hypovolemia and hemorrhagic shock. The nurses role is to observe for these complications, to report their signs and symptoms, and to administer prescribed parenteral fluids and blood and blood components. Monitoring of vital signs, skin condition, the urinary drainage system, the surgical incision, and the level of consciousness is necessary to detect evidence of bleeding, decreased circulating blood, and fluid volume and cardiac output. Bleeding is not normally evidenced by changes in pain or oral intake.

The nurse is providing a health education workshop to a group of adults focusing on cancer prevention. The nurse should emphasize what action in order to reduce participants risks of renal carcinoma? A) Avoiding heavy alcohol use B) Control of sodium intake C) Smoking cessation D) Adherence to recommended immunization schedules

C) Smoking cessation Tobacco use is a significant risk factor for renal cancer, surpassing the significance of high alcohol and sodium intake. Immunizations do not address an individuals risk of renal cancer.

The nurse is caring for a patient receiving hemodialysis three times weekly. The patient has had surgery to form an arteriovenous fistula. What is most important for the nurse to be aware of when providing care for this patient? A) Using a stethoscope for auscultating the fistula is contraindicated. B) The patient feels best immediately after the dialysis treatment. C) Taking a BP reading on the affected arm can damage the fistula. D) The patient should not feel pain during initiation of dialysis.

C) Taking a BP reading on the affected arm can damage the fistula. When blood flow is reduced through the access for any reason (hypotension, application of BP cuff/tourniquet), the access site can clot. Auscultation of a bruit in the fistula is one way to determine patency. Typically, patients feel fatigued immediately after hemodialysis because of the rapid change in fluid and electrolyte status. Although the area over the fistula may have some decreased sensation, a needle stick is still painful.

A previously healthy adult's sudden and precipitous decline in health has been attributed to fulminant hepatic failure, and the patient has been admitted to the intensive care unit. The nurse should be aware that the treatment of choice for this patient is what? A. IV administration of immune globulins B. Transfusion of packed red blood cells and fresh-frozen plasma (FFP) C. Liver transplantation D. Lobectomy

C. Liver transplantation Rationale: Liver transplantation carries the highest potential for the resolution of fulminant hepatic failure. This is preferred over other interventions, such as pharmacologic treatments, transfusions, and surgery

The nurse is caring for a patient with a history of systemic lupus erythematosus who has been recently diagnosed with end-stage kidney disease (ESKD). The patient has an elevated phosphorus level and has been prescribed calcium acetate to bind the phosphorus. The nurse should teach the patient to take the prescribed phosphorus-binding medication at what time? A) Only when needed B) Daily at bedtime C) First thing in the morning D) With each meal

D) With each meal Both calcium carbonate and calcium acetate are medications that bind with the phosphate and assist in excreting the phosphate from the body, in turn lowering the phosphate levels. Phosphate-binding medications must be administered with food to be effective.

Diagnostic testing has revealed that a patient's hepatocellular carcinoma (HCC) is limited to one lobe. The nurse should anticipate that this patient's plan of care will focus on what intervention? A. Cryosurgery B. Liver transplantation C. Lobectomy D. Laser hyperthermia

C. Lobectomy Rationale: Surgical resection is the treatment of choice when HCC is confined to one lobe of the liver and the function of the remaining liver is considered adequate for postoperative recovery. Removal of a lobe of the liver (lobectomy) is the most common surgical procedure for excising a liver tumor. While cryosurgery and liver transplantation are other surgical options for management of liver cancer, these procedures are not performed at the same frequency as a lobectomy. Laser hyperthermia is a nonsurgical treatment for liver cancer.

A patient with liver disease has developed jaundice; the nurse is collaborating with the patient to develop a nutritional plan. The nurse should prioritize which of the following in the patient's plan? A. Increased potassium intake B. Fluid restriction to 2L per day C. Reduction in sodium intake D. High-protein, low-fat diet

C. Reduction in sodium intake Rationale: Patient with ascites require a sharp reduction in sodium intake. Potassium intake should not be correspondingly increased. There is no need for fluid restriction or increased protein intake.

A patient who has undergone liver transplantation is ready to be discharged home. Which outcome of health education should the nurse prioritize? A. The patient will obtain measurement of drainage from the T-tube. B. The patient will exercise three times a week. C. The patient will take immunosuppressive agents as required. D. The patient will monitor for signs of liver dysfunction.

C. The patient will take immunosuppressive agents as required. Rationale: The patient is given written and verbal instructions about immunosuppressive agent doses and dosing schedules. The patient is also instructed on steps to follow to ensure that an adequate supply of medication is available so that there is no change of running out of the medication or skipping a dose. Failure to take medications as instructed may precipitate rejection. The nurse would not teach the patient to measure drainage from a T-tube as the patient would;t go home with a T-tube. The nurse may teach the patient about the need to exercise or what the signs of liver dysfunction are, but the nurse would not stress these topics over the immunosuppressive drug regimen.

A patient presents to the emergency department (ED) complaining of severe right upper quadrant pain. The patient states that his family doctor told him he had gallstones. The ED nurse should recognize what possible complication of gallstones? A) Acute pancreatitis B) Atrophy of the gallbladder C) Gallbladder cancer D) Gangrene of the gallbladder

D

A patient returns to the floor after a laparoscopic cholecystectomy. The nurse should assess the patient for signs and symptoms of what serious potential complication of this surgery? A) Diabetic coma B) Decubitus ulcer C) Wound evisceration D) Bile duct injury

D

A patient has experienced excessive losses of bicarbonate and has subsequently developed an acid-base imbalance. How will this lost bicarbonate be replaced? A) The kidneys will excrete increased quantities of acid. B) Bicarbonate will be released from the adrenal medulla. C) Alveoli in the lungs will synthesize new bicarbonate. D) Renal tubular cells will generate new bicarbonate.

D (Feedback: To replace any lost bicarbonate, the renal tubular cells generate new bicarbonate through a variety of chemical reactions. This newly generated bicarbonate is then reabsorbed by the tubules and returned to the body. The lungs and adrenal glands do not synthesize bicarbonate. Excretion of acid compensates for a lack of bicarbonate, but it does not actively replace it.)

A patient with chronic kidney disease has been hospitalized and is receiving hemodialysis on a scheduled basis. The nurse should include which of the following actions in the plan of care? A) Ensure that the patient moves the extremity with the vascular access site as little as possible. B) Change the dressing over the vascular access site at least every 12 hours. C) Utilize the vascular access site for infusion of IV fluids. D) Assess for a thrill or bruit over the vascular access site each shift.

D) Assess for a thrill or bruit over the vascular access site each shift. The bruit, or thrill, over the venous access site must be evaluated at least every shift. Frequent dressing changes are unnecessary and the patient does not normally need to immobilize the site. The site must not be used for purposes other than dialysis.

A patient has presented with signs and symptoms that are characteristic of acute kidney injury, but preliminary assessment reveals no obvious risk factors for this health problem. The nurse should recognize the need to interview the patient about what topic? A) Typical diet B) Allergy status C) Psychosocial stressors D) Current medication use

D) Current medication use The kidneys are susceptible to the adverse effects of medications because they are repeatedly exposed to substances in the blood. Nephrotoxic medications are a more likely cause of AKI than diet, allergies, or stress.

A nurse is amending a patient's plan of care in light of the fact that the patient has recently developed ascites. What should the nurse include in this patient's care plan? A. Mobilization with assistance at least 4 times daily B. Administration of beta-adrenergic blockers as ordered C. Vitamin B12 injections as ordered D. Administration of diuretics as ordered

D. Administration of diuretics as ordered Rationale: Use of diuretics along with sodium restriction is successful in 90% of patients with ascites. Beta-blockers are not used to treat ascites and bed rest is often more beneficial than increased mobility. Vitamin B23 injections are not necessary.

A nurse is participating in the emergency care of a patient who has just developed variceal bleeding. What intervention should the nurse anticipate? A. Infusion of intravenous heparin B. IV administration of albumin C. STAT administration of vitamin K by the intramuscular route D. IV administration of octreotide (Sandostatin)

D. IV administration of octreotide (Sandostation) Rationale: Octreotide (Sandostatin) a synthetic analog of the hormone somatostatin is effective in decreasing bleeding from esophageal varices, and lacks the vasoconstrictive effects of vasopressin. Because of this safety and efficacy profile, octreotide is considered the preferred treatment regimen for immediate control of variceal bleeding. Vitamin K and albumin are not administered and heparin would exacerbate, not alleviate, bleeding.

nurse is teaching a patient with Guillain-Barré syndrome about the disease. patient asks how he can ever recover if demyelination of his nerves is occurring. the nurse's best response

Guillain-Barré spares the Schwann cell, which allows for remyelination in the recovery phase of the disease."

A 35-year-old woman is diagnosed with a peripheral neuropathy. When making her plan of care, the nurse knows to include what in patient teaching

Inspect the lower extremities for skin breakdown.Footwear needs to be accurately sized. Assistive devices may be needed to reduce the risk of falls.

A patient diagnosed with MS should be encouraged to increase the fiber in his or her diet and void 30 minutes after drinking to help train the bladder.

The patient should participate in daily muscle stretching to help alleviate and relax muscle spasms.

A middle-aged woman has sought care from her primary care provider and undergone diagnostic testing that has resulted in a diagnosis of MS. What sign or symptom is

most likely to have prompted the woman to seek care Difficulty in coordination

Patients that develop trigeminal neuralgia before age 50 should be evaluated for the coexistent of MS because trigeminal neuralgia

occurs in approximately 5% of patients with MS

commonly the second and third branches of the trigeminal nerve. Myasthenia gravis, an autoimmune disorder affecting the myoneural junction, is characterized by varying degrees

of weakness of the voluntary muscles. Guillain-Barré syndrome is an autoimmune attack on the peripheral nerve myelin.

patient with metastatic cancer has developed trigeminal neuralgia and is taking carbamazepine for pain relief. What principle applies to the administration of this med

patient should be monitored for bone marrow depression. Side effects include nausea, dizziness, drowsiness, and aplastic anemia. Carbamazepine should be gradually increased

patient diagnosed with myasthenia gravis has been hospitalized to receive plasmapheresis for a myasthenic exacerbation. nurse knows that the course of treatment for

plasmapheresis in a patient with myasthenia gravis is what Determined by the patient's response

patient with Guillain-Barré syndrome has experienced a sharp decline in vital capacity. What is the nurse's most appropriate action

prepare to assist with intubation.mechanical ventilation is required if the vital capacity falls, making spontaneous breathing impossible and tissue oxygenation inadequate.


Set pelajaran terkait

Managerial Accounting- Ch 10 SmartBook

View Set

Home Inspection - Texas Report Form

View Set

Quiz 1 Dynamic Quizzes Covered Topics

View Set

EMT-Abdominal and Genitourinary Injuries ch 30 qs

View Set

World History Ch 26: World War II

View Set

Combo with "Chemical Bonding" and 15 others

View Set

research 1 study guide for final

View Set